You are on page 1of 55

Public Land V Govnt Land – Confirmation of Imperfect or incomplete title: Natural Resources Jennica Gyrl

Delfin

E. PUBLIC LAND VS. GOVERNMENT LAND The Court of Land Registration in its decision of
[G.R. No. 3714. January 26, 1909. ] December 1, 1906, dismissed the said oppositions
without costs, and decreed, after a general entry by
ISABELO MONTANO Y MARCIAL, Petitioner-Appellee, v. THE default, the adjudication and registration of the
INSULAR GOVERNMENT, ET AL., Respondents. — THE INSULAR property described in the petition, in favor of Isabelo
GOVERNMENT, Appellant. Montano y Marcial.

Attorney-General Araneta, for Appellant. From this decision only counsel for the Director of
Public Lands appealed to this court. It is a kindred
F. Buencamino, for Appellee. case to Cirilo Mapa v. The Insular Government, decided
by this court on February 19,1908, reported in 10 Phil.
SYLLABUS Rep., 175.
1. PUBLIC LANDS. — In Acts of the Congress of the
United States the term "public lands" is uniformly used As some discussion has arisen as to the scope of that
to describe so much of the national domain under the decision, it appears opportune to reaffirm the
legislative power of the Congress as has not been principle there laid down. The issue was, whether lands
subjected to private right or devoted to public use. used as a fishery, for the growth of nipa, and as salt
deposits, inland some distance from the sea, and
2. TIDEWATER LANDS. — Lands under the ebb and flow of asserted, though not clearly proved to be overflowed
the tide being reserved for public uses of navigation at high tide, could be registered as private property
and fishery and subject to Congressional regulation, on the strength of ten years’ occupation, under
pursuant to its power over commerce, are not understood paragraph 6 of section 54 of Act No. 926 of the
as included in the term "public lands" when used in Philippine Commission. The point decided was that such
general laws authorizing private appropriation thereof land within the meaning of the Act of Congress of July
as homesteads or otherwise. 1, 1902, was agricultural? the reasoning leading up to
that conclusion being that Congress having divided all
3. SWAMPS AND MARSHES. — Swamps and marshes not the public lands of the Islands into three classes it
available for the purpose of navigation or public uses must be included in one of the three, and being clearly
may be subjected to private appropriation although neither forest nor mineral, it must of necessity fall
covered by the tides. into the division of agricultural land. In the
concurring opinion, in order to avoid misapprehension
4, "MANGLARES." — Of this character are the manglar or on the part of those not familiar with United States
mangrove swamps of the Philippine Islands in which grow land legislation and a misunderstanding of the reach
aquatic trees cultivated and in common use for domestic of the doctrine, it was pointed out that under the
or commercial purposes. Such manglares when converted decisions of the Supreme Court of the United States
by man into fisheries and used as such for the the phrase "public lands" is held to be equivalent to
statutory period are the subject of private ownership "public domain," and does not by any means include all
under the Act of Congress of July 1, 1902, and Act No, lands of Government ownership, but only so much of said
926 of the Philippine Commission. lands as are thrown open to. private appropriation and
settlement by homestead and other like general laws.
The status of such lands at the time of the change of Accordingly, "government land" and "public land" are
sovereignty was not authoritatively determined under not synonymous terms; the first includes-not only the
the Spanish law and they are open to the benefit of second, but also other lands of the Government already
these statutes. reserved or devoted to public use or subject to private
right. In other words, the Government owns real estate
This is so even if the words "public lands" used in which is part of the "public lands" and other real
the Act of Congress be not given their otherwise estate which is not a part thereof.
uniform meaning but be interpreted as referring to such
lands as defined in the Spanish law theretofore This meaning attached to the phrase "public lands" by
prevailing in the Philippine Islands. Congress in its land legislation is settled by usage
and adjudication beyond a doubt, and without variation.
The case of Mapa v. The Insular Government (10 Phil, It is therefore doing the utmost violence to all rules
Rep., 176), considered and commented upon and the of construction to contend that in this law, dealing
Spanish Law of Waters of 1868 and Congressional with the same subject-matter in connection with these
legislation on the same subject reviewed. Islands, a different meaning had, without indication
or motive, been imported into the words. They can not
have one meaning in every other statute and a different
D E C I S I O N and conflicting meaning in this statute. Where property
in general is referred to therein, other and apt
phrases are used in order to include it; for instance,
TRACEY, J. : section 12 provides "that all the property and, rights
which may have been acquired in the Philippine Islands
by the United States . . . are hereby placed under the
Isabelo Montano presented a petition to the Court of control of the Government of the said Islands."
Land Registration for the inscription of a piece of Therefore, there is much real property belonging to
land in the barrio of Libis, municipality of Caloocan, the Government which is not affected by statutes for
used as a fishery, having a superficial area of 10,805 the settlement, prescription or sale of public lands.
square meters, and bounded as set out in the petition; Examples in point are properties occupied by public
its value according to the last assessment being buildings or devoted to municipal or other governmental
$505.05, United States currency. uses.

This petition was opposed by the Solicitor-General in Among the authorities cited in the Mapa case are two,
behalf of the Director of Lands, and by the entity Shively v. Bowlby (152 U. S., 1), and Mann v. Tacoma
known as Obras Pias de la Sagrada Mitra, the former on Land Co. (153 U. S., 273), in which it was held that
the ground that the land in question belonged to the general public land laws did not apply to land over
Government of the United States, and the latter, that which. the tide ebbs and flows. Mr. Justice Gray, in
it was the absolute owner of all the dry land along Shively v. Bowlby, which is in itself an epitome of
the eastern boundary of the said fishery. the American Law of Waters, speaking of tide lands,

1
Public Land V Govnt Land – Confirmation of Imperfect or incomplete title: Natural Resources Jennica Gyrl
Delfin
said:jgc:chanrobles.com.ph The interest of the people in the navigation of the
waters and in commerce over them may be improved in
"But Congress has never undertaken by general laws to many instances by the erection of wharves, docks, and
dispose of such lands. . . . piers therein, for which purpose the State may grant
parcels of the submerged lands; and, so long as their
"The Congress of the United States, in disposing of disposition is made for such purposes, no valid
the public lands, has constantly acted upon the theory objections can be made to the grants . . . . The control
that those lands, whether in the interior, or on the of the State for the purposes of the trust can never
coast, above high-water mark, may be taken up by actual be lost, except as to such parcels as are used in
occupants, in order to encourage the settlement of the promoting the interests of the public therein, or can
country, but that the navigable waters and the soils be disposed of without any substantial impairment of
under them, whether within or above the ebb and flow the public interest in the lands and waters remaining
of the tide, shall be and remain public highways; and, . . . . The State can no more abdicate its trust over
being chiefly valuable for the public purposes of property in which the whole people are interested, like
commerce, navigation, and fishery, and for the navigable waters and soils under them, so as to leave
improvements necessary to secure and promote those them entirely under the use and control of private
purposes, shall not be granted away during the period parties, except in the instance of parcels mentioned
of territorial government." (Pp. 48 and 49.) for the improvement of the navigation and use of the
waters, or when parcels can be disposed of without
The conclusions of the court are in part stated as impairment of the public interest in what remains, than
follows:jgc:chanrobles.com.ph it can abdicate its police powers in the administration
of government and the preservation of the peace . . .
"Lands under tide waters are incapable of cultivation . So with trusts connected with public property, or
or improvement in the manner of lands above high-water property of a special character, like lands under
mark. They are of great value to the public for the navigable waters, they can not be placed entirely
purposes of commerce, navigation, and fishery. Their beyond the direction and control of the State.
improvement by individuals, when permitted, is
incidental or subordinate to the public use and right. "The ownership of the navigable waters of the harbor
Therefore the title and the control of them are vested and of the lands under them is a subject of public
in the sovereign for the benefit of the whole people concern to the whole people of the State. The trust
. . . . with which they are held, therefore, is governmental
and can not be alienated, except in those instances
"Upon the acquisition of a territory by the United mentioned of parcels used in the improvement of the
States, whether by cession from one of the States, or interest thus held, or when parcels can be disposed of
by treaty with a foreign country, or by discovery and without detriment to the public interest in the lands
settlement, the same title and dominion passed to the and waters remaining. . . . ." (Pp. 452-455.)
United States, for the benefit of the whole people,
and in trust for the several States to be ultimately Mr. Justice Field quotes from an opinion by Mr. Justice
created out of the territory . . . Bradley, delivered in a case in the Circuit Court,
speaking of lands under water, as follows (p.
"The United States, while they hold the country as a 457):jgc:chanrobles.com.ph
territory, having all the powers both of national and
municipal government, may grant, for appropriate "Being subject to this trust, they were publici juris;
purposes, titles or rights in the soil below high- in other words, they were held for the use of the
water mark of tide waters. But they have never done so people at large. It is true that to utilize the
by general laws." (Pp. 57 and 58.) fisheries, especially those of shellfish, it was
necessary to parcel them out to particular operators,
In Mann v. Tacoma Land Co., it was said by Mr. Justice and employ the rent or consideration for the benefit
Brewer (p. 284):jgc:chanrobles.com.ph of the whole people; but this did not alter the
character of the title. The land remained subject to
"It is settled that the general legislation of Congress all other public uses as before, especially to those
in respect to public lands does not extend to tide of navigation and commerce, which are always paramount
lands . . . . It provided that the scrip might be to those of public fisheries. It is also true that
located on the unoccupied and unappropriated public portions of the submerged shoals and flats, which
lands, but the term ’public lands’ does not include really interfered with navigation, and could better
tide lands. As said in Newhall v. Sanger (92 U. S., subserve the purposes of commerce by being filled up
761, 763.) ’The words "public lands" are habitually and reclaimed, were disposed of to individuals for that
used in our legislation to describe such as are subject purpose. But neither did these dispositions of useless
to sale or other disposal under general laws.’" parts affect the character of the title to the
remainder."cralaw virtua1aw library
In Illinois Central R. R. Company v. Illinois (146 U.
S., 387) Mr. Justice Field, delivering the opinion of These citations are thus given at length in order to
the court, said:jgc:chanrobles.com.ph make clear, first, that lands under the ebb and flow
of the tide of navigable waters are not in America
"That the State holds the title to the lands under the understood to be included in the phrase "public lands"
navigable waters of Lake Michigan within its limits, in Acts of Congress of the United States; nor,
in the same manner that the State hold title to soils perforce, can they be so understood in laws of the
under tide water, by the common law, we have already Philippine Commission drawn immediately under the
shown, and that title necessarily carries with it sanction of those Acts; and, second, that such lands
control over the waters above them whenever the lands are not under existing Congressional legislation the
are subjected to use. But it is a title different in subject of private ownership, any occupation thereof
character from that which the State holds in lands being subordinate to the public purposes of navigation
intended for sale. It is different from the title which and fishery. While as well in the original thirteen
the United States hold in the public lands which are States in which there was never a national public
open to preemption and sale. It is a title held in domain to which the land laws of Congress could apply
trust for the people of the State that they may enjoy as in States more recently created out of that domain
the navigation of the waters, carry on commerce over and which upon their formation became masters of their
them, and have liberty of fishing therein freed from own land policy, the local laws govern riparian and
the obstruction or interference of private parties. littoral rights, subject only to Congressional control

2
Public Land V Govnt Land – Confirmation of Imperfect or incomplete title: Natural Resources Jennica Gyrl
Delfin
in matters of foreign and interstate commerce (U. S. interesting historical and graphic citations he
v. Mission Rock Co., 189 U. S., 391), yet, as to the describes that part of the marginal seashore of the
unappropriated public lands constituting the public Philippine Islands known as manglares, with their
domain the sole power of legislation is vested in characteristic vegetation. In brief, it may be said
Congress, which has uniformly and consistently that they are mud flats, alternately washed and exposed
declined to assume the function of authorizing or by the tide, in which grow various kindred plants which
regulating private appropriation of such rights. will not live except when watered by the sea, extending
Therefore, in the absence of specific Congressional their roots deep into the mud and casting their seeds,
legislation, it is impossible for individuals to which also germinate there. These constitute the
acquire title under the ten years’ provision of Act mangrove flats of the tropics, which exist naturally,
No. 926 or even through a definite grant from the local but which are also, to some extent, cultivated by man
legislature of land beneath navigable waters in which for the sake of the combustible wood of the mangrove
the tide ebbs and flows, except for wharfage or other and like trees as well as for the useful nipa palm
purposes auxiliary to navigation or other public uses, propagated thereon. Although these flats are literally
unless in conformity with the preexisting local law of tidal lands, yet we are of the opinion that they can
the Archipelago. not be so regarded in the sense in which that term is
used in the cases cited or in general American
The matter is dwelt upon for the reason that the late jurisprudence. The waters flowing over them are not
Attorney-General in his very able brief calls attention available for purpose of navigation, and they "may be
to the effect apprehended from the extension of the disposed of without impairment of the public interest
words "agricultural lands" as used in Act No. 926 to in what remains." Mr. Justice Bradley, in the passage
include all public lands not forest or mineral in quoted by Mr. Justice Field, makes an exception of
character, specifying two Acts of the Philippine submerged shoals and flats. In Railroad Company v.
Commission, the validity of which he fears might Schurmeir (74 U. S., 272), a Government patent of
thereby be called into question. The first of these, public land bordering upon a river was held to include
Act No. 1039, dedicates to the use of the Navy a parcel submerged at very high water and separated
Department of the United States Government certain from the mainland by a slough in which the water ran
ground and buildings in Cavite, while the other, Act when ordinarily high. In Mobile v. Hallett (41 U. S.,
No. 1654, is a fore-shore law regulating the control 260), at page 266, Mr. Justice Catron remarked in his
and disposal of filled Government lands. If the term dissenting opinion:jgc:chanrobles.com.ph
"agricultural lands" be held to include all government
property not forest or mineral in character, he ". . . and that a mud flat, flowed by tide water is
suggests that these Acts, not being in conformity with the subject of grant by the Government to an
the procedure of Act No. 926, as approved by Congress, individual, I think can not well be doubted by anyone
would be invalid, and moreover, that the Philippine acquainted with the southern country; when such
Government would be seriously tied up in the management valuable portions of it are mud flats, in the constant
and disposition of other lands owned by it. course of reclamation."cralaw virtua1aw library

Without finally passing on this question in relation In several of the older States along the Atlantic coast
to lands the owners of which are not before us as such flats, either by force of ordinance, custom,
parties to this action, it is appropriate, in answering judicial construction, or local laws are held to pass
the argument of the law officer of the State, to point under private grants as appurtenant to the uplands.
out that this consequence appears to be avoided by the (Winslow v. Patten, 34 Maine, 25; Litchfield v.
restricted sense given to the words "public land" or Scituate, 135 Mass., 39; People v. New York and Staten
"public domain" in the Act of Congress and in Act No. Island Ferry Co., 68 N. Y., 71; Stevens v. P. & N.
926, as hereinbefore noted. Neither the property Railroad, 5 Vroom, 34 N. J. Law, 532.) There is even
affected by Act No. 1039, already in use by the Navy stronger reason for excepting mud flats from the rule
Department of the United States, nor the fore-shore of tide lands in these Islands, owing to the
land mentioned in Act No. 1654, which is under the ebb peculiarities of their configuration and to the nature
and flow of the tide, was, in so far as appears in the of the tropical growth thereon, and whatever may be
Acts before us, part of the public domain to be the action of the tide, we do not think that in the
disposed of under sections 13, 14, 15, and 16 of the Philippines such of the shoals covered by this
Act of Congress of July 1, 1902, and for that reason vegetation, whether spontaneously or by cultivation,
it is not included in any of the three subdivisions of as are not available for free navigation, or required
"public lands" as agricultural or otherwise, although for any other purpose of general benefit, can be
it was part of the property acquired in the Philippine considered tidal land reserved for public use alone,
Islands by the United States by the treaty of peace under the governmental trust for commerce and public
with Spain, which by section 12 of that Act was "placed fishery, but, on the contrary, we regard them as public
under the control of the Government of said Islands, property, susceptible of a sort of cultivation and of
to be administered for the benefit of the inhabitants improvement, and as such, subject to occupation under
thereof." It would seem that the validity of the Cavite paragraph 6 of section 54 of the Land Law. Instances
Act can not be successfully assailed on this ground, may hereafter arise of fisheries unduly established in
while it may well be that The Fore-shore Act on what are clearly navigable waters which would
examination will be found to fall, as to its general constitute a nuisance, and not be the subject of
purpose, within the authorization of section 11 of the prescription or of grant. A brief reference to the five
Act of Congress, whereby the duty is imposed upon the cases under consideration in this court, however, will
Island Government of improving the harbors and serve to show that they all fairly fall within the
navigable waters in the interest of commerce. benefits of the law. In the Mapa case 1 the property
was far from the sea, partly occupied as a fish pond,
As a consequence, it follows that The Public Land Act as nipa land, and as a salt pit. It does not appear
did not apply to the fisheries in the Mapa case, if whether it was connected with the sea by nature or by
they are to be regarded as constituting, in a general art, or whether the tide ebbed or flowed upon it, or
sense, land under tidal waters. It becomes necessary, whether the salt was sufficient to impart to any
therefore, to refer to the character of the lands. portion of it a mineral character. In the Santiago case
2 there was a fishery about two thousand yards from
Although argued at different times, five of these cases the sea, with which it communicated by a river, and a
have been presented substantially together, all being portion of the inclosure was dedicated to growing the
covered by one brief of the late Attorney-General in aquatic tree called bacawan. The fishery had been
behalf of the Government in which, with many constructed by man, upon land heretofore sown with this

3
Public Land V Govnt Land – Confirmation of Imperfect or incomplete title: Natural Resources Jennica Gyrl
Delfin
tree. In the Gutierrez case 1 it was shown that the presented itself as to what constituted seashore, which
land was partly highland, growing fruit trees, and was of public use and trust and therefore not
partly low land, converted by the occupant of the alienable. This question can not be said to have been
upland into a fishery by his labor. In the Baello case, settled by official ruling at the time of the American
2 the river running to the sea was a hundred meters occupation. From the official records it appears that
away, the salt water therefrom reaching the lowland by there were then pending for registration a great number
means of an artificial canal cut by the owner of the of possessory expedientes, twenty-two of which, made
land when he gave up cultivating bacawan thereon, and before April 17, 1895, were from the Province of
made it into a fishery. In the Montano case, although Pampanga alone, in which the land was described as
there was a considerable depth of water over the soil, manglares. Under the royal decree of 1894 such
yet before the fishery was made, some thirty years manglares appear at the outset to have been registered
before the trial, bacawan had been sown and propagated and considered alienable and numbers of them were
in the mud by the owner who finally sold the entire conceded by adjustment, including considerable tracts
cut when he built the dikes. in the town of Sexmoan and Lubao in Pampanga. Claims
having been made that on account of the trees growing
All these lots, in their original state, whether near thereon they formed part of the forest reserve and also
the sea or at a distance from it inland, and whether because, being covered and uncovered by the tide, they
bare or washed by the tides, were not covered by waters were part of the shore, and in either case were
practically navigable and were filled, whether inalienable, the engineer in chief of the forestry
naturally or artificially, with vegetation sometimes district of the center of Luzon addressed, on January
cultivated and in common use for fuel and for building 7, 1893, a communication to the inspector general de
purposes, and they were all adapted to fisheries or montes (Forestry Department) in which he expressed an
fish hatcheries by the labor of man introducing or opinion that as part of the shore they were not subject
regulating the access of salt water thereto. It is to private ownership and asked for an early decision
obvious that all five cases are of the same general of the question. On November 26, 1893, the acting
nature and that one rule must be applied to them all. inspector-general notified the chief of the district
of the Visayas in Mindanao that his excellency, the
In this discussion of the meaning which the Congress governor-general, had that day ordered all action
of the United States attached to the phrase "public suspended on expedientes of manglar and nipa lands and
lands" in the Philippine Bill, we have assumed that it salt marshes until the questions involved in regard
was used in the same sense as in other laws enacted by thereto should be determined. In this condition the
that body. If, however, it can be considered as matter remained until the expiration of the Spanish
employed with reference to the peculiar conditions of sovereignty.
the territory to which it was to be applied and to the
local law or usage prevailing therein, the result would By article 14 of the Law of Waters the right of shore
not be different. In many of its general features the fishery was declared public, but by article 23
Spanish law of public lands in the Philippines authority might be granted individuals to establish
resembled the American. Government property was of two shore hatcheries for fish and shellfish, and by article
kinds — first, that of public use or service, said to 15 salt-water ponds on private ground not communicating
be of public ownership, and second, that having a with the sea by water navigable by boats were
private character or use. (Civil Code, arts. 339 and recognized as private property, while chapter 10
340.) Lands of the first class, while they retain their permitted and regulated the draining of swamps and
public character are inalienable; those of the second marshes, both of private and of public ownership.
are not.
Under this uncertain and somewhat unsatisfactory
By the royal decree of February 13, 1894, it was condition of the law the custom had grown up of
enacted that all "the land, soil, ground not under converting manglares and nipa lands into fisheries
cultivation, and forests in the Philippine Islands which became a common feature of settlements along the
should be considered saleable crown lands," which were coast and at the time of the change of sovereignty
not included in four exceptions stated, among which constituted one of the most productive industries of
were "those which belonged to forest zones which the the Islands, the abrogation of which would destroy
State desires to hold for the Commonwealth." This vested interests and prove a public disaster. In our
corresponds in the main to the American classification opinion it was the object of Congress not to work such
into Government property, public lands, and forest a result but, on the contrary, in furtherance of the
reserve. Mineral lands are elsewhere defined. It is to purposes of the treaty of Paris, to recognize and
be noted, however, that in the two languages terms safeguard such property. Therefore the judgment of the
ordinarily equivalent are not in this relation employed Court of Land Registration is affirmed, without cost.
in the same sense and that lands de dominio publico
signify quite a different thing from the arbitrary Torres, Mapa and Carson, JJ., concur.
English phrases "public lands" or "public
domain."cralaw virtua1aw library MODES OF DISPOSITION
The Law of Waters of 1866, which was the latest Spanish G.R. No. L-30389 December 27, 1972
Law of Waters extended to these Islands, provides that PEDRO LEE HONG HOK, SIMEON LEE HONG HOK, ROSITA LEE
private property can not be acquired in lands HONG HOK and LEONCIO LEE HONG HOK, petitioners,
preserving the character of public ownership (title 1, vs.
art. 1, par. 29), and among the lands declared of ANIANO DAVID, THE HON. SECRETARY OF AGRICULTURE AND
public ownership and use by article 1 of chapter 1 of NATURAL RESOURCES, THE DIRECTOR OF LANDS and COURT OF
title 5 of the same law are:jgc:chanrobles.com.ph APPEALS, respondents.
Augusto A. Pardalis for petitioners.
"The seashore. —By shore is understood the land Luis General, Jr. for respondent Aniano David.
alternately covered and uncovered by the sea in its Office of the Solicitor General for other respondents.
tidal movement. Its interior, or land limit, is the
point reached by the highest and equinoctial tides. At FERNANDO, J.:p
those places not affected by tides, the land limit is Petitioners 1 in this appeal by certiorari would have
the highest point reached by sea water in ordinary us reverse a decision of respondent Court of Appeals
storms or hurricanes." (Par. 3.) affirming a lower court judgment dismissing their
complaint to have the Torrens Title 2 of respondent
So that under this legislation the same question also Aniano David declared null and void. What makes the

4
Public Land V Govnt Land – Confirmation of Imperfect or incomplete title: Natural Resources Jennica Gyrl
Delfin
task for petitioners quite difficult is that their its holding that authoritative doctrines preclude a
factual support for their pretension to ownership of party other than the government to dispute the validity
such disputed lot through accretion was rejected by of a grant and the recognition of the indefeasible
respondent Court of Appeals. Without such character of a public land patent after one year, is
underpinning, they must perforce rely on a legal possessed of merit. Consequently, as set forth at the
theory, which, to put it mildly, is distinguished by outset, there is no justification for reversal.
unorthodoxy and is therefore far from persuasive. A 1. More specifically, the shaft of criticism was let
grant by the government through the appropriate public loose by petitioner aimed at this legal proposition
officials 3 exercising the competence duly vested in set forth in the exhaustive opinion of then Justice
them by law is not to be set at naught on the premise, Salvador Esguerra of the Court of Appeals, now a member
unexpressed but implied, that land not otherwise of this Court: "There is, furthermore, a fatal defect
passing into private ownership may not be disposed of of parties to this action. Only the Government,
by the state. Such an assumption is at war with settled represented by the Director of Lands, or the Secretary
principles of constitutional law. It cannot receive of Agriculture and Natural Resources, can bring an
our assent. We affirm. action to cancel a void certificate of title issued
The decision of respondent Court of Appeals following pursuant to a void patent (Lucas vs. Durian, 102 Phil.
that of the lower court makes clear that there is no 1157; Director of Lands vs. Heirs of Ciriaco Carlo,
legal justification for nullifying the right of G.R. No. L-12485, July 31, 1959). This was not done by
respondent Aniano David to the disputed lot arising said officers but by private parties like the
from the grant made in his favor by respondent plaintiffs, who cannot claim that the patent and title
officials. As noted in the decision under review, he issued for the land involved are void since they are
"acquired lawful title thereby pursuant to his not the registered owners thereof nor had they been
miscellaneous sales application in accordance with declared as owners in the cadastral proceedings of Naga
which an order of award and for issuance of a sales Cadastre after claiming it as their private property.
patent was made by the Director of Lands on June 18, The cases cited by appellants are not in point as they
1958, covering Lot 2892 containing an area of 226 refer to private registered lands or public lands over
square meters, which is a portion of Lot 2863 of the which vested rights have been acquired but
Naga Cadastre. On the basis of the order of award of notwithstanding such fact the Land Department
the Director of Lands the Undersecretary of Agriculture subsequently granted patents to public land
and Natural Resources issued on August 26, 1959, applicants."5 Petitioner ought to have known better.
Miscellaneous Sales Patent No. V-1209 pursuant to which The above excerpt is invulnerable to attack. It is a
OCT No. 510 was issued by the Register of Deeds of Naga restatement of a principle that dates back to Maninang
City to defendant-appellee Aniano David on October 21, v. Consolacion, 6 a 1908 decision. As was there
1959. According to the Stipulation of Facts, since the categorically stated: "The fact that the grant was made
filing of the sales application of Aniano David and by the government is undisputed. Whether the grant was
during all the proceedings in connection with said in conformity with the law or not is a question which
application, up to the actual issuance of the sales the government may raise, but until it is raised by
patent in his favor, the plaintiffs-appellants did not the government and set aside, the defendant can not
put up any opposition or adverse claim thereto. This question it. The legality of the grant is a question
is fatal to them because after the registration and between the grantee and the government."7 The above
issuance of the certificate and duplicate certificate citation was repeated ipsissimis verbis in Salazar v.
of title based on a public land patent, the land Court of Appeals.8 Bereft as petitioners were of the
covered thereby automatically comes under the right of ownership in accordance with the findings of
operation of Republic Act 496 subject to all the the Court of Appeals, they cannot, in the language
safeguards provided therein.... Under Section 38 of of Reyes v. Rodriguez, 9 "question the [title] legally
Act 496 any question concerning the validity of the issued." 10 The second assignment of error is thus
certificate of title based on fraud should be raised disposed of.
within one year from the date of the issuance of the 2. As there are overtones indicative of skepticism, if
patent. Thereafter the certificate of title based not of outright rejection, of the well-known
thereon becomes indefeasible.... In this case the land distinction in public law between the government
in question is not a private property as the Director authority possessed by the state which is appropriately
of Lands and the Secretary of Agriculture and Natural embraced in the concept of sovereignty, and its
Resources have always sustained the public character capacity to own or acquire property, it is not
thereof for having been formed by reclamation.... The inappropriate to pursue the matter further. The former
only remedy therefore, available to the appellants is comes under the heading of imperium and the latter
an action for reconveyance on the ground of fraud. In of dominium. The use of this term is appropriate with
this case we do not see any fraud committed by reference to lands held by the state in its proprietary
defendant-appellant Aniano David in applying for the character. In such capacity, it may provide for the
purchase of the land involved through his Miscellaneous exploitation and use of lands and other natural
Sales Application No. MSA-V-26747, entered in the resources, including their disposition, except as
records of the Bureau of Lands [Miscellaneous Sales] limited by the Constitution. Dean Pound did speak of
Entry No. V-9033, because everything was done in the the confusion that existed during the medieval era
open. The notices regarding the auction sale of the between such two concepts, but did note the existence
land were published, the actual sale and award thereof of res publicae as a corollary to dominium." 11 As far
to Aniano David were not clandestine but open and as the Philippines was concerned, there was a
public official acts of an officer of the Government. recognition by Justice Holmes in Cariño v. Insular
The application was merely a renewal of his deceased Government, 12 a case of Philippine origin, that "Spain
wife's application, and the said deceased occupied the in its earlier decrees embodied the universal feudal
land since 1938." 4 theory that all lands were held from the
On such finding of facts, the attempt of petitioners Crown...." 13 That was a manifestation of the concept
to elicit a different conclusion is likely to be of jura regalia, 14 which was adopted by the present
attended with frustration. The first error assigned Constitution, ownership however being vested in the
predicated an accretion having taken place, state as such rather than the head thereof. What was
notwithstanding its rejection by respondent Court of stated by Holmes served to confirm a much more
Appeals, would seek to disregard what was accepted by extensive discussion of the matter in the leading case
respondent Court as to how the disputed lot came into of Valenton v. Murciano, 15 decided in 1904. One of the
being, namely by reclamation. It does not therefore royal decrees cited was incorporated in the
call for any further consideration. Neither of the Recopilacion de Leyes de las Indias 16 in these words:
other two errors imputed to respondent Court, as to "We having acquired full sovereignty over the Indies

5
Public Land V Govnt Land – Confirmation of Imperfect or incomplete title: Natural Resources Jennica Gyrl
Delfin
and all lands, territories, and possessions not proceedings, and for both the decree of registration
heretofore ceded away by our royal predecessors, or by issued is conclusive and final." 30Such a view has been
us, or in our name, still pertaining to the royal crown followed since then. 31 The latest case in point
and patrimony, it is our will that all lands which are is Cabacug v. Lao. 32 There is this revealing excerpt
held without proper and true deeds of grant be restored appearing in that decision: "It is said, and with
to us according as they belong to us, in order that reason, that a holder of a land acquired under a free
after reserving before all what to us or to our patent is more favorably situated than that of an owner
viceroys audiences, and governors may seem necessary of registered property. Not only does a free patent
for public squares, ways, pastures, and commons in have a force and effect of a Torrens Title, but in
those places which are peopled, taking into addition the person to whom it is granted has likewise
consideration not only their present condition, but in his favor the right to repurchase within a period
also their future and their probable increase, and of five years." 33 It is quite apparent, therefore,
after distributing to the natives what may be necessary that petitioners' stand is legally indefensible.
for tillage and pasturage, confirming them in what they WHEREFORE, the decision of respondent Court of Appeals
now have and giving them more if necessary, all the of January 31, 1969 and its resolution of March 14,
rest of said lands may remain free and unencumbered 1969 are affirmed. With costs against petitioners-
for us to dispose of as we may wish." 17 appellants.
It could therefore be affirmed in Montano v. Insular
Government" 18 that "as to the unappropriated public G.R. No. L-58867 June 22, 1984
lands constituting the public domain the sole power of DIRECTOR OF LANDS and DIRECTOR OF FOREST
legislation is vested in Congress, ..." 19 They DEVELOPMENT, petitioners,
continue to possess that character until severed vs.
therefrom by state grant. 20 Where, as in this case, it HON. COURT OF APPEALS and ANTONIO VALERIANO, GABRIELA
was found by the Court of Appeals that the disputed VALERIANO VDA. DE LA CRUZ, LETICIA A. VALERIANO and
lot was the result of reclamation, its being correctly MARISSA VALERIANO DE LA ROSA, respondents.
categorized as public land is undeniable. 21What was The Solicitor General for petitioners.
held in Heirs of Datu Pendatun v. Director of Carlos C. Serapio for private respondents.
Lands 22 finds application. Thus: "There being no
evidence whatever that the property in question was MELENCIO-HERRERA, J.:
ever acquired by the applicants or their ancestors Petitioners-public officials, through the Solicitor
either by composition title from the Spanish Government General, seek a review of the Decision and Resolution
or by possessory information title or by any other of the then Court of Appeals affirming the judgment of
means for the acquisition of public lands, the property the former Court of First Instance of Bulacan, Branch
must be held to be public domain." 23 For it is well- III, decreeing registration of a parcel of land in
settled "that no public land can be acquired by private private respondents' favor. The land in question,
persons without any grant, express or implied, from Identified as Lot 2347, Cad-302-D, Case 3, Obando
the government." 24 It is indispensable then that there Cadastre, under Plan Ap-03-000535, is situated in
be a showing of a title from the state or any other Obando, Bulacan, and has an area of approximately 9.3
mode of acquisition recognized by law. 25 The most hectares. It adjoins the Kailogan River and private
recent restatement of the doctrine, found in an opinion respondents have converted it into a fishpond.
of Justice J.B.L. Reyes, follows: 26 "The applicant, In their application for registration filed on May 10,
having failed to establish his right or title over the 1976, private respondents (Applicants, for brevity)
northern portion of Lot No. 463 involved in the present claimed that they are the co-owners in fee simple of
controversy, and there being no showing that the same the land applied for partly through inheritance in 1918
has been acquired by any private person from the and partly by purchase on May 2, 1958; that it is not
Government, either by purchase or by grant, the within any forest zone or military reservation; and
property is and remains part of the public that the same is assessed for taxation purposes in
domain." 27 To repeat, the second assignment of error their names.
is devoid of merit. The Republic of the Philippines, represented by the
3. The last error assigned would take issue with this Director of the Bureau of Forest Development opposed
portion of the opinion of Justice Esguerra: "According the application on the principal ground that the land
to the Stipulation of Facts, since the filing of the applied for is within the unclassified region of
sales application of Aniano David and during all the Obando, Bulacan, per BF Map LC No. 637 dated March 1,
proceedings in connection with said application, up to 1927; and that areas within the unclassified region
the actual issuance of the sales patent in his favor, are denominated as forest lands and do not form part
the of the disposable and alienable portion of the public
plaintiffs-appellants did not put up any opposition or domain.
adverse claim thereto. This is fatal to them because After hearing, the Trial Court ordered registration of
after the registration and issuance of the certificate the subject land in favor of the Applicants. This was
and duplicate certificate of title based on a public affirmed on appeal by respondent Appellate Court, which
land patent, the land covered thereby automatically found that "through indubitable evidence (Applicants)
comes under the operation of Republic Act 496 subject and their predecessors-in-interest have been in open,
to all the safeguards provided therein ... Under public, continuous, peaceful and adverse possession of
Section 38 of Act 496 any question concerning the the subject parcel of land under a bona fide claim of
validity of the certificate of title based on fraud ownership for more than 30 years prior to the filing
should be raised within one year from the date of the of the application" and are, therefore, entitled to
issuance of the patent. Thereafter the certificate of registration. It further opined that "since the subject
title based thereon becomes indefeasible property is entirely devoted to fishpond purposes, it
..." 28 Petitioners cannot reconcile themselves to the cannot be categorized as part of forest lands. "
view that respondent David's title is impressed with Before this instance, the principal issues posed are:
the quality of indefeasibility. In thus manifesting (1) whether or not Courts can reclassify the subject
such an attitude, they railed to accord deference to public land; and (2) whether or not applicants are
controlling precedents. As far back as 1919, in Aquino entitled to judicial confirmation of title.
v. Director of The parties, through their respective counsel,
Lands, 29 Justice Malcolm, speaking for the Court, stipulated that the land is within an unclassified
stated: "The proceedings under the Land Registration region of Obando, Bulacan, as shown by BF Map LC No.
Law and under the provisions of Chapter VI of the 637, dated March 1, 1927. 1 No evidence has been
Public Land Law are the same in that both are against submitted that the land has been released or
the whole world, both take the nature of judicial subsequently classified despite an Indorsement, dated

6
Public Land V Govnt Land – Confirmation of Imperfect or incomplete title: Natural Resources Jennica Gyrl
Delfin
November 17, 1976, of the District Forester, to the MENGUITO, petitioners, vs. REPUBLIC OF THE
Director of Forest Development, containing the PHILIPPINES, respondent.
following recommendation: D E C I S I O N
Subject area requested for release was verified and PANGANIBAN, J.:
found to be within the Unclassified Region of Obando, Unless a piece of public land is shown to have been
Bulacan per BF LC Map No. 637, certified March 1, 1927. classified as alienable and disposable, it remains part
However, on-the-spot inspection conducted by a of the inalienable public domain. Even assuming that
representative of this Office, it disclosed that the such land has been classified as alienable, title
same was devoid of any forest growth and forms part of thereto can be registered only upon presentation of
a well-developed and 100 percent producing fishponds. incontrovertible proof of adverse, notorious and open
Two houses of light materials were erected within the possession in the concept of owner for a period of
area for the caretakers temporary dwelling. thirty years.
In view thereof, and in fairness to the applicant The Case

considering the investment introduced therein this Before us is a Petition for Review under Rule 45 of
Office believes that the release is in order, the Rules of Court assailing the September 30, 1997
Recommended for approval and be disposed of in Decision[1] and the June 23, 1998 Resolution[2] of the
accordance with the Public Land Law.2 Court of Appeals (CA) in CA-GR CV No. 39638. The
The Government's case is meritorious. decretal portion of said Decision reads as follows:
In effect, what the Courts a quo have done is to WHEREFORE, the decision appealed from is
release the subject property from the unclassified hereby REVERSED and SET ASIDE. Accordingly, the
category, which is beyond their competence and appellees application for registration is
jurisdiction. The classification of public lands is an hereby DISMISSED.[3]
exclusive prerogative of the Executive Department of The Decision of the Regional Trial Court (RTC) of Pasig
the Government and not of the Courts. In the absence City (Branch 157),[4] which was reversed by the
of such classification, the land remains as appellate court, granted petitioners application for
unclassified land until it is released therefrom and registration in this wise:[5]
rendered open to disposition. 3 This should be so under WHEREFORE, the order of general default against the
time-honored Constitutional precepts. This is also in whole world heretofore entered in this case is
consonance with the Regalian doctrine that all lands affirmed, and judgment is hereby rendered confirming
of the public domain belong to the State, 4 and that the registerable title of the applicants to the land
the State is the source of any asserted right to described in their application under plan Swo-13-
ownership in land and charged with the conservation of 000227 and its technical descriptions, situated in the
such patrimony. 5 Barrio of Ususan, Municipality of Taguig, Metro Manila,
The recommendation of the District Forester for release and containing an aggregate area of 2,112 square
of subject property from the unclassified region is meters; and individual and separate certificates of
not the ultimate word on the matter. And the fact that titles to the lots comprising the said land are hereby
BF Map LC No. 637 dated March 1, 1927 showing subject ordered registered in the names of the applicants, as
property to be within the unclassified region was not follows:
presented in evidence will not operate against the 1. For lots 6045-A, 6045-B, 6045-C, and 6045-D in the
State considering the stipulation between the parties name of Susana Menguito, of legal age, widow, Filipino
and under the well-settled rule that the State cannot citizen, with residence and postal address at T. Sulit,
be estopped by the omission, mistake or error of its St., Pater[o]s, Metro Manila;
officials or agents, 6 if omission there was, in fact. 2. For Lot 6045-E -- in the name of Renato Menguito,
While it may be that the Municipality of Obando has of legal age, married to Irene Toledo, Filipino
been cadastrally surveyed in 1961, it does not follow citizen, with residence and postal address at T. Sulit,
that an lands comprised therein are automatically St., Pateros, Metro Manila;
released as alienable. A survey made in a cadastral 3. For Lot 6045-F -- in the name of Bersamin Menguito,
proceeding merely Identifies each lot preparatory to a of legal age, Filipino citizen, single, with residence
judicial proceeding for adjudication of title to any and postal address at T. Sulit, St., Pateros, Metro
of the lands upon claim of interested parties. Besides, Manila;
if land is within the jurisdiction of the Bureau of 4. For Lot 6045-G -- in the name of Generoso Menguito,
Forest Development, it would be beyond the jurisdiction of legal age, Filipino citizen, single, with residence
of the Cadastral Court to register it under the Torrens and postal address at T. Sulit, St., Pateros, Metro
System. Manila;
Since the subject property is still unclassified, 5. For Lot 6045-H -- in the name of Helen Marta
whatever possession Applicants may have had, and, Menguito, of legal age, Filipino citizen, single, with
however long, cannot ripen into private ownership. 7
residence and postal address at T. Sulit, St., Pateros,
The conversion of subject property into a fishpond by Metro Manila;
Applicants, or the alleged titling of properties around 6. For Lot 6046-I -- in the name of Froilan Menguito,
it, does not automatically render the property as of legal age, Filipino citizen, married to Zenaida
alienable and disposable. Applicants' remedy lies in Carag, with residence and postal address at T.Sulit
the release of the property from its present St., Pateros, Metro Manila;
classification. In fairness to Applicants, and it 7. For Lot 6045-J -- in the name of Emelita Menguito,
appearing that there are titled lands around the of legal age, Filipino citizen, married to Luciano
subject property, petitioners-officials should give Manalili, with residence and postal address at T.
serious consideration to the matter of classification Sulit, St., Pateros, Metro Manila; and
of the land in question. 8. For Lot 6045-K -- in the name of Generoso Menguito,
WHEREFORE, the appealed Decision is reversed and the of legal age, Filipino citizen, married to Luciano
application for registration in Land Registration Case Manalili; and Froilan Menguito, of legal age, Filipino
No. N299-V-76 of the former Court of First Instance of citizen, married to Zenaida Carag, all with residence
Bulacan, Branch III, is hereby dismissed, without and postal address at T. Sulit St., Pateros, Metro
prejudice to the availment by the applicants of the Manila.
proper administrative remedy. No costs. Upon the finality of this Decision, let an Order be
SO ORDERED. issued to the Commissioner of Land Registration
Authority for the issuance of the decree of
[G.R. No. 134308. December 14, 2000] registration and the corresponding certificates of
SUSANA MENGUITO, EMELITA MENGUITO-MANALILI, HELEN title in favor of the applicants pursuant to Section
MARTA MENGUITO-LUNA, RENATO MENGUITO, BERSAMIN 39 of PD No. 1529.
MENGUITO, FROILAN MENGUITO and GENEROSO SO ORDERED.

7
Public Land V Govnt Land – Confirmation of Imperfect or incomplete title: Natural Resources Jennica Gyrl
Delfin
The Facts 9. That the following documents are attached hereto
The antecedents of the case are adequately summarized and made part hereof:
by the Court of Appeals as follows: (a) Tracing cloth plan of Swo-13-000227
On November 10, 1987, in the Regional Trial Court at (b) Two (2) print copies of said plan Swo-13-000227
Pasig, Metro Manila an Application for Registration of (c) Three (3) copies each of the Technical Description
Title was filed by the following successors-in- of:
interest of the deceased spouses Cirilo Menguito and Lot 6045-A
Juana Manalo-Menguito, namely: SUSANA MENGUITO, Lot 6045-B
EMELITA MENGUITO-MANALILI, HELEN MARTA MENGUITO-LUNA, Lot 6045-C
RENATO MENGUITO, BERSAMIN MENGUITO, FROILAN MENGUITO Lot 6045-D
and GENEROSO MENGUITO. Docketed in the said court as Lot 6045-E
LRC Case No. N-10938, the application reads: Lot 6045-F
APPLICATION FOR REGISTRATION OF TITLE Lot 6045-G
The above-named applicants hereby apply to have the Lot 6045-H
land hereinafter described brought under the operation Lot 6045-I
of the Land Registration Act as amended by the Property Lot 6045-J
Registration Decree No. 1529 and to have their title Lot 6045-K
thereto registered and confirmed, (d) Three (3) copies of Engineers Certificate
AND DECLARE: (e) Four (4) copies of Tax Declaration No. B-011-01351
1. That the applicants are the owners in fee simple of x x x x x x x x x
eleven (11) parcels of land situated in the Barrio of (Amended Record on Appeal, pp. 1-5).
Ususan, Municipality of Taguig, Metro Manila, and are Acting on the foregoing application, the lower court
bounded and described as shown on plan Swo-13-000227 issued a Notice of Initial Hearing addressed to: the
(lot Nos. 6045-A, 6045-B, 6045-C, 6045-D, 6045-E, 6045- Solicitor General, the Director of the Land Management
F, 6045-G, 6045-H, 6045-I, 6045-J and 6045-K) and Bureau, the Secretary of the Department of Public Works
corresponding technical descriptions, x x x; and Highways, the Secretary of the Department of
2. That said parcels of land are assessed for taxation Agrarian Reform, the Director of the Bureau of Forest
for the current year at P5,910.00 as per Tax Development, and the owners of the adjacent properties
Declaration No. B-11-01351 of the land record of as mentioned in the application, informing them that
Taguig, Metro Manila; the application is scheduled for initial hearing on
3. That to the best of applicants knowledge and belief, April 25, 1989. The addressees were then ordered to
there is no mortgage or encumbrance of any kind present such claims as you may have to said lands or
whatsoever affecting the said land nor any other any portion thereof, and to submit evidence in support
persons having any estate or interest therein, legal of such claims and unless you appear at said court at
or equitable, in possession, remainder, reversion or the time and place aforesaid, your default will be
expectancy; recorded and the title to the lands will be adjudicated
4. That the applicants acquired the said parcels of and determined in accordance with law and the evidence
land by inheritance; before the Court, and thereafter, you will forever be
5. That said parcels of land are occupied by the barred from contesting said application or any decree
applicants and their predecessors-in-interest have entered thereon (Exhibit A).
been in actual, open, peaceful, continuous, and adverse Said notice of initial hearing was published in the
possession, in the concept of owners, of said parcels April 5, 1989 issue of Abante, a daily tabloid (Exhs.
of land for more than thirty years; C, C-1, C-1-A).
6. That the names in full and addresses as far known Earlier, or on March 30, 1989, the Republic of the
to the undersigned, of the owners of all adjoining Philippines, through the Solicitor General, filed its
properties are as follows: Opposition to the application for registration
(a) Pilar Menguito contending:
Pateros-Taguig Road 1. That neither the applicant nor his predecessors-in-
Ususan, Taguig interest have been in open, continuous, exclusive and
Metro Manila notorious possession and occupation of the land in
b) Andres Filemon question since June 12, 1945 or prior thereto (Sec. 48
Pateros-Taguig Road [b], C.A. 141, as amended by P.D. 1073).
Ususan, Taguig 2. That the muniments of title and tax payment receipts
Metro Manila of applicant, if any, attached to or alleged in the
c) Beatriz Dumagat application, do not constitute competent and
Pateros-Taguig Road sufficient evidence of a bona fideacquisition of the
Ususan, Taguig lands applied for or his open, continuous, exclusive
Metro Manila and notorious possession and occupation thereof in the
d) Maura Cabanatan concept of owner, since June 12, 1945, or prior
Pateros-Taguig Road thereto. Said muniments of title do not appear to be
Ususan, Taguig genuine and indicate the pretended possession of
Metro Manila applicant to be of recent vintage.
e) Pateros-Taguig Road 3. That the claim of ownership in fee simple on the
c/o The District Engineer basis of Spanish title or grant can no longer be
Pasig, Metro Manila availed of by the applicant who has failed to file an
7. That the applicants full name, age, citizenship, appropriate application for registration within the
residence, and postal address, are as follows: period of six (6) months from February 16, 1976 as
SUSAN MENGUITO, widow; EMELITA M. MANALILI, married to required by Presidential Decree No. 892. From the
Luciano Manalili; HELEN MARTA M. LUNA, married to records, it appears that the instant application was
Benjamin Luna, Jr.; RENATO MENGUITO, married to Irene filed on July 31, 1990.
Toledo; BERSAMIN MENGUITO, married to Elvira 4. That the parcel applied is part of the public domain
Salvacion; FROILAN MENGUITO, married to Zenaida Carag; belonging to the Republic of the Philippines not
and GENEROSO MENGUITO, single; all of legal age, subject to private appropriation. (Amended Record on
Filipinos, and with residence and postal address at T. Appeal, pp. 5-6).
Sulit St., Pateros, Metro Manila. The Solicitor General therefore prayed for the denial
8. That should the Land Registration Act invoked be of the application for registration and for the
not applicable in the instant case, the applicants declaration of the properties subject thereof as part
hereby apply for the benefit of Chapter VIII of of the public domain belonging to the Republic of the
Commonwealth Act No. 141 as amended; Philippines.

8
Public Land V Govnt Land – Confirmation of Imperfect or incomplete title: Natural Resources Jennica Gyrl
Delfin
At the scheduled initial hearing of the case on April On July 8, 1991, the lower court issued an order
25, 1989, a certain Jose Tangco, Jr. appeared and denying the motion for reconsideration for lack of
registered a verbal opposition to the application. On merit.[6]
motion of counsel for the applicants, the court issued Ruling of the Court of Appeals

an Order of General Default against the whole world, The Court of Appeals agreed with respondent that the
except as against the oppositors Republic of the lower court had failed to consider the legal
Philippines and Jose Tangco, Jr., who was directed to requirements for registration of imperfect titles;
file his written opposition but never did. Thereafter, namely: (1) the land is alienable and disposable; and
trial on the merits ensued. (2) the applicants and their predecessors-in-interest
On June 13, 1990, the applicants filed their Formal have occupied and possessed the land openly,
Offer of Evidence, submitting therewith the following continuously, exclusively, and adversely since June
documentary exhibits: (1) Plan Swo-13-000227 (Exh. F); 12, 1945. It was not convinced that the land in
(2) technical descriptions of Lot Nos. 6045-A to 6045- question had been classified as alienable or disposable
J, inclusive (Exhs. F to F-10, inclusive); (3) and that petitioners or their predecessors-in-interest
Engineers Certificate (Exh. G); (4) Extra-judicial had been in possession of it since June 12, 1945.
Settlement and Partition executed by the applicants Hence, this Petition.[7]
dated December 12, 1985 (Exh. H); (5) description of The Issue

the land and the apportionment thereof among the In their Memorandum, petitioners submit a single issue
applicants (Exhs.H-1 and H-2, respectively); (6) Tax for our consideration:
Declarations (Exhs. I, J, K, L, M, N and O) (7) Tax Whether or not the court a quo erred in reversing the
Receipts (Exhs. O, O-1, P. P-1, Q and R); (8) Kasulatan findings of facts of the trial court.[8]
ng Pagkakaloob dated May 7, 1969 executed by Cirilo In fine, the Court will resolve whether the CA erred
Menguito in favor of Pedro Menguito (Exh. S); and (9) in rejecting petitioners application for the
Deed of Partition dated November 7, 1990 executed by registration of their respective titles.
the applicants (Exh. T). The Courts Ruling

On September 12, 1990, the oppositor Republic filed The Petition is devoid of merit.
its Manifestation and Opposition to applicants formal Sole Issue: Registration of Petitioners Titles

offer of evidence. The said manifestation reads: Section 48 of Commonwealth Act (CA) No. 141,[9] as
It interposes no objection to the admission of Exhibits amended, provides for the registration of imperfect
A, B, C, D, relative to jurisdictional requirements. titles to lands of the public domain in this wise:
It has no objection to Exhibits E, F, F-1, to F-10 "SECTION 48. The following described citizens of the
relating to the plan and the technical description of Philippines, occupying lands of public domain or
the lots being applied for and Exhibit G which is the claiming to own any such lands or an interest thereon,
Engineers certificate. but whose titles have not been perfected or completed,
It objects to Exhibits H, H-1 to H-2 the extrajudicial may apply to the Court of First Instance of the
settlement and partition dated December 12, 1985 for province where the land is located for confirmation of
being self serving. It objects to Exhibits I, J, K, L, their claims, and the issuance of a certificate of
M and N for being incompetent and insufficient proof title therefor, under the Land Registration Act, to
of possession of the lot in question by applicants or wit:
their predecessors-in interest. In fact the said tax x x x x x x x x x
declarations do not date back to at least June 12, (b) those who by themselves or through their
1945. It objects to Exhibits O, P, Q, and R, the same predecessor in-interest have been in open, continuous,
being incompetent and insufficient to prove possession exclusive and notorious possession and occupation of
since June 12, 1945. It objects to Exhibits O, P, Q, agricultural lands of the public domain, under a bona
and R, the same being incompetent and insufficient to fide claim of acquisition or ownership, for at least
prove possession since June 12, 1945. It objects to thirty years immediately preceding the filing of the
Exhibit S as being self-serving being a mere photocopy application for confirmation of title except when
of the alleged Kasulatan ng Pagkakaloob dated May 7, prevented by war or force majeure. They shall be
1989 executed by Cirilo Menguito the same cannot be conclusively presumed to have performed all the
accepted in evidence, applicants not having first laid conditions essential to a Government grant and shall
the basis for the presentation of secondary be entitled to a certificate of title under the
evidence. It objects to the first page of Exhibit T, provisions of this Chapter.
being self-serving and a mere photocopy. Furthermore, Presidential Decree (PD) No. 1073[10] clarified
page 2 of said exhibit, where the supposed paragraph b of the said provision by specifically
acknowledgment of the instrument appears, refers to declaring that it applied only to alienable and
different parcels of land other than those being disposable lands of the public domain.[11]
applied for. Hence, as observed by the appellate court, petitioners
WHEREFORE, considering that the applicants have failed were duty-bound to prove two legal requirements: (1)
to prove their title to the lands applied for, it is the land applied for was alienable and disposable; and
respectfully prayed that the application for (2) the applicants and their predecessors-in-interest
registration be denied and that the land applied for had occupied and possessed the land openly,
be declared as part of the public domain belonging to continuously, exclusively, and adversely since June
the Republic of the Philippines. 12, 1945.
Considering the above, oppositor respectfully The records show that petitioners failed to establish
manifests that there is no need for it to submit these two requisites.
Classification of the Land
evidence in support of its opposition. (Amended Record
on Appeal, pp. 11-13). To prove that the land in question formed part of the
On May 15, 1991, the lower court rendered its decision alienable and disposable lands of the public domain,
disposing as follows: petitioners relied on the printed words which
WHEREFORE, the order of general default against the read: This survey plan is inside Alienable and
whole world heretofore entered in this case is Disposable Land Area, Project No. 27-B as per L.C. Map
affirmed, and judgment is hereby rendered confirming No. 2623, certified by the Bureau of Forestry on
the registerable title of theapplicants x x x January 3, 1968, appearing on Exhibit E (Survey Plan
On June 11, 1991, the oppositor Republic, through the No. Swo-13-000227).
Solicitor General, moved for a reconsideration of the This proof is not sufficient. Section 2, Article XII
afore-quoted decision, to which a written opposition of the 1987 Constitution, provides: All lands of the
was interposed by the applicants. public domain, waters, minerals, coal, petroleum, and
other mineral oils, all forces of potential energy,
fisheries, forests or timber, wildlife, flora and

9
Public Land V Govnt Land – Confirmation of Imperfect or incomplete title: Natural Resources Jennica Gyrl
Delfin
fauna, and other natural resources are owned by the receipts (Exhs. O. O1, P, and P-1) and the Municipal
State. x x x. (Emphasis supplied.) Treasurers certifications of tax payments (Exhs. Q and
For the original registration of title, the applicant R) presented in evidence are incompetent and
(petitioners in this case) must overcome the insufficient to prove petitioners and their
presumption that the land sought to be registered forms predecessors-in-interests possession of the lots in
part of the public domain.[12] Unless public land is question.
shown to have been reclassified or alienated to a Because the factual findings of the trial and the
private person by the State, it remains part of the appellate courts were contrary to each other, we waded
inalienable public domain. Indeed, occupation thereof into the records,[18] but found no reason to modify the
in the concept of owner, no matter how long, cannot assailed CA Decision.Much as we want to conform to the
ripen into ownership and be registered as a States policy of encouraging and promoting the
title.[13] To overcome such presumption, distribution of alienable public lands to spur economic
incontrovertible evidence must be shown by the growth and remain true to the ideal of social justice,
applicant.[14] Absent such evidence, the land sought to our hands are tied by the laws stringent safeguards
be registered remains inalienable. against registering imperfect titles. In this case, we
In the present case, petitioners cite a surveyor- agree with the CA that petitioners have not presented
geodetic engineers notation in Exhibit E indicating sufficient proof of their compliance with the legal
that the survey was inside alienable and disposable requirements for registration of imperfect titles.
land. Such notation does not constitute a positive WHEREFORE, the Petition is DENIED and the assailed
government act validly changing the classification of Decision AFFIRMED. Costs against petitioners.
the land in question. Verily, a mere surveyor has no SO ORDERED.
authority to reclassify lands of the public domain. By
relying solely on the said surveyors assertion, A. HOMESTEAD SETTLEMENT
petitioners have not sufficiently proven that the land
in question has been declared alienable. G.R. No. L-14722 May 25, 1960
Period of Possession
IGNACIO MESINA, plaintiff-appellant,
Even assuming arguendo that petitioners have been able vs.
to prove that the land is alienable, their Petition EULALIA PINEDA VDA. DE SONZA, ET AL., defendants.
for confirmation of their imperfect titles and EULALIA PINEDA VDA. DE SONZA, defendant-appellee.
registration thereof under the law will still be Agustin C. Bagasao for appellant.
denied. The reason is that they have failed to Luis Manalang and Associates for appellee.
establish possession of the lots in question -- openly, BAUTISTA ANGELO, J.:
continuously, exclusively and adversely -- in the Plaintiff brought this action before the Court of First
concept of owner for at least 30 years, since June 12, Instance of Nueva Ecija praying that Original
1945. Certificate of Title No. P-1137 of the Register of
Petitioners do not claim that they are the original Deeds of Nueva Ecija be ordered cancelled and that the
possessors of the lots in question, which had allegedly registration case pending before the same court
belonged to Cirilo Menguito before he donated it to covering the property described therein be given due
his son Pedro. When Pedro died in 1978, these lots course and that defendants be ordered to pay plaintiff
allegedly passed down to petitioners. P1,000.00 as attorney's fees and costs.
Although petitioners can trace their possession of the Defendants filed a motion to dismiss on the ground that
land from as far back as 1968 only, they would tack it plaintiff's action is already barred by the statute of
to that of their predecessors, who had supposedly been limitations. The reasons advanced are: the complaint
in possession thereof even before the Second World was filed on March 25, 1958. The decree of registration
War. There is not enough convincing proof, however, to or issuance of patent over the property was issued
support such claim. "sometime on September 12, 1953 or thereabout", while
Petitioners presented evidence that they had been the transfer certificate of title covering the same
paying real estate taxes since 1974.[15] Their was issued on September 16, 1953. The present action
predecessors-in-interest, they claimed, have also been which calls for the cancellation of said decree and
paying taxes on the land for several years before them, title has, therefore, been filed after the elapse of
and Cirilo Menguito had declared the land for tax more than four years, which cannot be done, because
purposes in 1943.[16] However, they did not present any the title has already become indefeasible and
documents or any other satisfactory proof to incontrovertible. The court sustained this motion and
substantiate this claim. General statements, which are dismissed the complaint. Hence the present appeal.
mere conclusions of law and not proofs of possession, Plaintiff claims that he is the owner in fee simple of
are unavailing and cannot suffice.[17] Lot No. 3259, with improvements thereon, situated in
Cirilos six children were not presented as witnesses San Antonio, Nueva Ecija; that he has been in actual
by petitioners during the hearing of their application possession thereof since 1914, publicly, openly,
for registration of the lots in question. In fact, of peacefully and against the whole world and up to the
the six children, only Pilar Menguito was personally present time he is the only one who benefits from the
informed of petitioners application. Still, she was produce thereof; that said lot is at present the
not presented as a witness. subject of registration proceedings pending in the same
There can be no question that Cirilos children were court known as Registration Case No. N-372, L.R.C. Cad.
the best witnesses, because they could have Record No. N-12238; that sometime in September 12,
substantiated petitioners claim that indeed the lots 1953, the Director of Lands, without exercising due
in question had been donated to Pedro care, and in spite of his knowledge that defendants
Menguito. Moreover, they may even have in their had not complied with the knowledge that defendants
possession documents that can adequately support their had not complied with the requirements of Commonwealth
supposed claim. Instead, petitioners presented only Act No. 141, issued a homestead patent in their favor
Raymunda Bautista, the alleged tenant of Cirilo as a consequence of which a certificate of title was
Menguito, who had tilled the land before petitioners issued in their name by the register of deeds; that
built their houses thereon. Neither Cirilos children said title was procured by defendants through frauds,
nor the documents that they might have had in their deception and misrepresentation since they knew that
possession were presented. the lot belonged to the plaintiff; and that the
Furthermore, serious doubts are cast on petitioners Director of Lands has no authority nor jurisdiction to
claim that their predecessors-in-interest have been in issue a patent covering said land because it is a
open, continuous, exclusive and adverse possession and private property of plaintiff. For these reasons,
occupation of the land. Because they are of recent plaintiff prays that said decree and title be
vintage, the tax declarations (Exhs. I to N), tax cancelled.

10
Public Land V Govnt Land – Confirmation of Imperfect or incomplete title: Natural Resources Jennica Gyrl
Delfin
Republic Act No. 1942, which took effect on June 22, thereof since 1914, publicly, openly, peacefully and
1957 (amending Section 48-b of Commonwealth Act 141), against the whole world, and that up to the present
provides: time he is the only one who benefits from the produce
(b) Those who by themselves or through their thereof. He further claims that said lot is present
predecessors in interest have been in open, continuous, the subject of a registration proceeding pending in
exclusive and notorious possession and occupation of the same court, known as Registration Case No. N-372,
agricultural lands of the public domain, under a bona L.R.C. Cad. Record No. N-12238. If by legal fiction,
fide claim of acquisition of ownership, for at least as stated in the Susi case, plaintiff is deemed to have
thirty years immediately preceeding the filing of the acquired the lot by a grant of the State, it follows
application for confirmation of title except when that the same had ceased to be part of the public
prevented by war or force majeure. These shall be domain and had become private property and, therefore,
conclusively presumed to have performed all the is beyond the control of the Director of Lands.
conditions essential to a Government grant and shall Consequently, the homestead patent and the original
be entitled to a certificate of title under the certificate of title covering said lot issued by the
provisions of this chapter. Director of Lands in favor of the defendants can be
In the case of Susi vs. Razon, et al., 48 Phil., 424, said to be null and void, for having been issued
it was observed that where all the necessary through fraud, deceit and misrepresentation.
requirements for a grant by the Government are complied Considering that this case was dismissed by the trial
with through actual physical possession openly, court merely on a motion to dismiss on the ground that
continuously, and publicly, with a right to a plaintiff's action is already barred by the statute of
certificate of title to said land under the provisions limitations, which apparently is predicated on the
of Chapter VIII of Act No. 2874, amending Act No. 926 theory that a decree of registration can no longer be
(carried over as Chapter VIII of Commonwealth Act No. impugned on the ground of fraud one year after the
141), the possessor is deemed to have already acquired issuance and entry of the decree,1 which theory does
by operation of law not only a right to a grant, but not apply here because the property involved is
a grant of the Government, for it is not necessary that allegedly private in nature and has ceased to be part
a certificate of title be issued in order that said of the public domain, we are of the opinion that the
grant may be sanctioned by the court — an application trial court erred in dismissing the
therefor being sufficient under the provisions of case outright without giving plaintiff a chance to
Section 47 of Act No. 2874 (reproduced as Section 50, prove his claim. It would have been more proper for
Commonwealth Act No. 141). Thus, the following is what the court to deny the motion on the ground that its
this Court said on the matter: object does not appear to be indubitable, rather than
It clearly appears from the evidence that Valentin Susi to have dismissed it, as was done by the trial court.
has been in possession of the land in question openly, Wherefore, the order appealed from is set aside. The
continuously, adversely and publicly, personally and case is remanded to the trial court for further
through his predecessors, since the year 1880, that proceedings. No costs.
is, for about forty-five years. ... When on August 15, Paras, Bengzon, C.J., Padilla, Labrador, Concepcion,
1914, Angela Razon applied for the purchase of said Barrera, and Gutierrez David, JJ., concur.
land, Valentin Susi had already been in possession
thereof personally and through his predecessors for G.R. No. L-24066 December 9, 1925
thirty-forty years. And if it is taken into account VALENTIN SUSI, plaintiff-appellee,
that Nemesio Pinlac had already made said land a fish vs.
pond when he sold it on December 13, 1880, it can ANGELA RAZON and THE DIRECTOR OF LANDS, defendants.
hardly be estimated when he began to possess and occupy THE DIRECTOR OF LANDS, appellant.
it, the period of time being so long that it is beyond Acting Attorney-General Reyes for appellant.
the reach of memory. ... In favor of Valentin Susi, Monico R. Mercado for appellee.
there is, moreover the presumption juris et de
jure established paragraph (b) of section 45 of Act
No. 2874, amending Act No. 926, that all the necessary VILLA-REAL, J.:
requirements for a grant by the Government were This action was commenced in the Court of First
complied with, for he has been in actual and physical Instance of Pampanga by a complaint filed by Valentin
possession, personally and through his predecessors, Susi against Angela Razon and the Director of Lands,
of an agricultural land of the public domain openly, praying for judgment: (a) Declaring plaintiff the sole
continuously, exclusively and publicly since July 26, and absolute owner of the parcel of land described in
1894, with a right to a certificate of title to said the second paragraph of the complaint; (b) annulling
land under the provisions of Chapter VIII of said Act. the sale made by the Director of Lands in favor of
So that when Angela Razon applied for the grant in her Angela Razon, on the ground that the land is a private
favor, Valentin Susi had already acquired, by operation property; (c) ordering the cancellation of the
of law, not only a right to grant, but a grant of the certificate of title issued to said Angela Razon; and
Government, for it is not necessary that certificate (d) sentencing the latter to pay plaintiff the sum of
of title should be issued in order that said grant may P500 as damages, with the costs.
be sanctioned by the courts, an application therefor For his answer to the complaint, the Director of Lands
is sufficient, under the provisions of section 47 of denied each and every allegation contained therein and,
Act No. 2874. If by a legal fiction, Valentin Susi had as special defense, alleged that the land in question
acquired the land in question by a grant of the State, was a property of the Government of the United States
it had already ceased to be of the public domain and under the administration and control of the Philippine
had become private property, at least by presumption, Islands before its sale to Angela Razon, which was made
of Valentin Susi, beyond the control, of the Director in accordance with law.
of Lands. Consequently, in selling the land in question After trial, whereat evidence was introduced by both
to Angela Razon, the Director of Lands disposed of a parties, the Court of First Instance of Pampanga
land over which he had no longer any title or control, rendered judgment declaring the plaintiff entitled to
and the sale thus made was void and of no effect, and the possession of the land, annulling the sale made by
Angela Razon did not thereby acquire any right. the Director of Lands in favor of Angela Razon, and
(Emphasis supplied) ordering the cancellation of the certificate of title
Such is the situation in which the plaintiff claims to issued to her, with the costs against Angela Razon.
be in his complaint. He alleges that he is the owner From this judgment the Director of Lands took this
in fee simple of the lot in question, with the appeal, assigning thereto the following errors, to wit:
improvements thereon, situated in San Antonio, Nueva (1) The holding that the judgment rendered in a prior
Ecija, and that he has been in actual possession case between the plaintiff and defendant Angela Razon

11
Public Land V Govnt Land – Confirmation of Imperfect or incomplete title: Natural Resources Jennica Gyrl
Delfin
on the parcel of land in question is controlling in Susi, there is, moreover, the presumption juris et de
this action; (2) the holding that plaintiff is entitled jure established in paragraph (b) of section 45 of Act
to recover the possession of said parcel of land; the No. 2874, amending Act No. 926, that all the necessary
annulment of the sale made by the Director of Lands to requirements for a grant by the Government were
Angela Razon; and the ordering that the certificate of complied with, for he has been in actual and physical
title issued by the register of deeds of the Province possession, personally and through his predecessors,
of Pampanga to Angela Razon by virtue of said sale be of an agricultural land of the public domain openly,
cancelled; and (3) the denial of the motion for new continuously, exclusively and publicly since July 26,
trial filed by the Director of Lands. 1894, with a right to a certificate of title to said
The evidence shows that on December 18, 1880, Nemesio land under the provisions of Chapter VIII of said Act.
Pinlac sold the land in question, then a fish pond, So that when Angela Razon applied for the grant in her
tho Apolonio Garcia and Basilio Mendoza for the sum of favor, Valentin Susi had already acquired, by operation
P12, reserving the right to repurchase the same of law, not only a right to a grant, but a grant of
(Exhibit B). After having been in possession thereof the Government, for it is not necessary that
for about eight years, and the fish pond having been certificate of title should be issued in order that
destroyed, Apolonio Garcia and Basilio Mendoza, on said grant may be sanctioned by the courts, an
September 5, 1899, sold it to Valentin Susi for the application therefore is sufficient, under the
sum of P12, reserving the right to repurchase it provisions of section 47 of Act No. 2874. If by a legal
(Exhibit A). Before the execution of the deed of sale, fiction, Valentin Susi had acquired the land in
Valentin Susi had already paid its price and sown question by a grant of the State, it had already ceased
"bacawan" on said land, availing himself of the to be the public domain and had become private
firewood gathered thereon, with the proceeds of the property, at least by presumption, of Valentin Susi,
sale of which he had paid the price of the property. beyond the control of the Director of Lands.
The possession and occupation of the land in question, Consequently, in selling the land in question to Angela
first, by Apolonio Garcia and Basilio Mendoza, and then Razon, the Director of Lands disposed of a land over
by Valentin Susi has been open, continuous, adverse which he had no longer any title or control, and the
and public, without any interruption, except during sale thus made was void and of no effect, and Angela
the revolution, or disturbance, except when Angela Razon did not thereby acquire any right.
Razon, on September 13, 1913, commenced an action in The Director of Lands contends that the land in
the Court of First Instance of Pampanga to recover the question being of the public domain, the plaintiff-
possession of said land (Exhibit C), wherein after appellee cannot maintain an action to recover
considering the evidence introduced at the trial, the possession thereof.lawphi1.net
court rendered judgment in favor of Valentin Susi and If, as above stated, the land, the possession of which
against Angela Razon, dismissing the complaint is in dispute, had already become, by operation of law,
(Exhibit E). Having failed in her attempt to obtain private property of the plaintiff, there lacking only
possession of the land in question through the court, the judicial sanction of his title, Valentin Susi has
Angela Razon applied to the Director of Lands for the the right to bring an action to recover possession
purchase thereof on August 15, 1914 (Exhibit C). Having thereof and hold it.
learned of said application, Valentin Susi filed and For the foregoing, and no error having been found in
opposition thereto on December 6, 1915, asserting his the judgment appealed from, the same is hereby affirmed
possession of the land for twenty-five years (Exhibit in all its parts, without special pronouncement as to
P). After making the proper administrative costs. So ordered.
investigation, the Director of Lands overruled the Avanceña, C.J., Malcolm, Street, Villamor, Ostrand,
opposition of Valentin Susi and sold the land to Angela Johns, and Romualdez, JJ., concur.
Razon. By virtue of said grant the register of deeds Johnson, J., took no part.
of Pampanga, on August 31, 1921, issued the proper
certificate of title to Angela Razon. Armed with said
document, Angela Razon required Valentin Susi to vacate [G.R. No. 127827. March 5, 2003]
the land in question, and as he refused to do so, she ELEUTERIO, ANATALIA, JOSELITO, ROGELIO, EVANGELINE,
brought and action for forcible entry and detainer in NOEL, GUILLERMO, LORENZO, DOMINGO, AMADO, and
the justice of the peace court of Guagua, Pampanga, VICTORIA, all surnamed LOPEZ, petitioners, vs. THE
which was dismissed for lack of jurisdiction, the case HONORABLE COURT OF APPEALS, and spouses MARCELINO and
being one of title to real property (Exhibit F and M). CRISTINA S. LOPEZ, FELISA LOPEZ and RAMON CORTEZ, ZOILO
Valentin Susi then brought this action. LOPEZ, LEONARDO LOPEZ and LEONILA LOPEZ and spouses
With these facts in view, we shall proceed to consider ROGELIO M. AMURAO and NOAMI T. AMURAO, respondents.
the questions raised by the appellant in his D E C I S I O N
assignments of error.lawphi1.net PUNO, J.:
It clearly appears from the evidence that Valentin Susi Before us is a petition for review on certiorari of
has been in possession of the land in question openly, the Decision[1] dated September 30, 1996 of the Court
continuously, adversely, and publicly, personally and of Appeals in C.A.-G.R. CV No. 43837, which affirmed
through his predecessors, since the year 1880, that with modification the Decision dated March 30, 1993 of
is, for about forty-five years. While the judgment of the Regional Trial Court of Antipolo, Rizal, Branch
the Court of First Instance of Pampanga against Angela 71, in Civil Case No. 677-A.
Razon in the forcible entry case does not affect the The evidence shows that in 1920, Fermin Lopez occupied,
Director of Lands, yet it is controlling as to Angela possessed, and declared for taxation purposes a parcel
Razon and rebuts her claim that she had been in of public land containing an area of 19 hectares, 48
possession thereof. When on August 15, 1914, Angela ares, 88 centares, more or less, situated in
Razon applied for the purchase of said land, Valentin Makatubong, Barrio De la Paz, Antipolo, Rizal. He filed
Susi had already been in possession thereof personally a homestead application over the land, but his
and through his predecessors for thirty-four years. application was not acted upon until his death in
And if it is taken into account that Nemesio Pinlac 1934. When he died, he was survived by the following:
had already made said land a fish pond when he sold it (1) Hermogenes Lopez, now deceased, leaving his
on December 18, 1880, it can hardly be estimated when children, respondents Marcelino, Felisa, Zoilo, and
he began to possess and occupy it, the period of time Leonardo, all surnamed Lopez, as his heirs; (2)
being so long that it is beyond the reach of memory. petitioner Eleuterio Lopez; (3) Juan Lopez, now
These being the facts, the doctrine laid down by the deceased, leaving his children, Guillermo, Lorenzo,
Supreme Court of the United States in the case of Domingo, Amado, and Victoria, all surnamed Lopez, as
Cariño vs. Government of the Philippine Islands (212 his heirs;[2] and (4) Nazario, now deceased, leaving
U. S., 449 1), is applicable here. In favor of Valentin his wife, petitioner Anatalia, and children,

12
Public Land V Govnt Land – Confirmation of Imperfect or incomplete title: Natural Resources Jennica Gyrl
Delfin
petitioners Joselito, Rogelio, Evangeline and Noel, On May 31, 1985, petitioners Eleuterio, Anatalia,
all surnamed Lopez, as his heirs. Joselito, Rogelio, Evangeline and Noel, all heirs of
Following Fermins death, Hermogenes, being the eldest Nazario Lopez, along with Guillermo, Lorenzo, Domingo,
child, worked and introduced additional improvements Amado, and Victoria, all heirs of Juan Lopez,
on the land. In 1936, he inquired from the Bureau of instituted the present action against the respondents
Lands the status of his late fathers application for a before the RTC of Antipolo, Rizal, Branch 71, docketed
homestead grant. An official[3] of the bureau informed as Civil Case No. 677-A. They prayed, among others,
him that the application remained unacted upon and that they be declared co-owners of the property subject
suggested that he file a new application. Following matter hereof and that private respondents be ordered
the suggestion, Hermogenes filed a homestead to reconvey to them 3/5 thereof as its co-owners, or
application in his own name, which was docketed as No. in the alternative, to pay its value. On June 26, 1985,
138612. After ascertaining that the land was free from respondents filed their Answer with Compulsory
claim of any private person, the Bureau approved his Counterclaim alleging that they are the absolute owners
application. In 1939, Hermogenes submitted his final of the contested land on the basis of the homestead
proof of compliance with the residency and cultivation grant to their predecessor-in-interest, Hermogenes.
requirements of the law. The land was surveyed and a After the pre-trial on November 27, 1987, trial ensued.
resulting plan, H-138612, was approved by the Director In the August 28, 1986 hearing petitioners counsel
of Lands, who thereafter ordered the issuance of the failed to appear, causing the case to be dismissed.
homestead patent. The patent was later transmitted to The dismissal, however, was reconsidered upon motion
the Register of Deeds of Rizal for transcription and of petitioners counsel, and the case was again set for
issuance of the corresponding certificate of title in hearing. In the scheduled hearing of October 17, 1986,
his name. counsel for respondent was absent. Upon proper motion,
Unaware that he has been awarded a homestead patent, petitioners were allowed to present their evidence ex-
Hermogenes executed on February 11, 1956 an Extra- parte on December 5, 1986. Following the presentation
judicial Partition of the disputed land with his of ex-parte evidence, the case was deemed submitted
brothers - petitioner Eleuterio, Juan, and Nazario. On for resolution.
September 12, 1958, however, the three executed a Deed On June 25, 1987, the court a quo rendered a decision
of Absolute Sale of their share in the land in favor in favor of the petitioners ordering the division of
of Hermogenes. The succeeding year, Hermogenes applied the disputed lot in equal portions among the four
with the Land Registration Commission for the children of Fermin or their heirs. Respondents failed
registration of the property in his name. This was to appeal the decision but on September 10, 1987, they
docketed as LRC Case No. 2531. To his surprise, he filed a petition for relief from judgment, alleging
found that the land has been registered in the names that accident/excusable negligence prevented them from
of Fernando Gorospe, Salvador de Tagle, Rosario de attending the trial and that they have a good,
Tagle, Beatriz de Suzuarrequi and Eduardo Santos, who substantial and meritorious defense. On December 28,
collectively opposed his application. 1989, the court a quo set aside its decision dated June
In December 1959, Hermogenes filed a complaint for the 25, 1987 and ordered a pre-trial conference.
annulment of the free patent and title against these On January 30, 1990, respondents filed a Motion to
persons before the Court of First Instance of Admit Amended Answer alleging for the first time that
Rizal,[4] docketed as Civil Case No. 5957. Some of the petitioners have already sold to Hermogenes their
defendants moved for its dismissal alleging that shares in the contested property. Petitioners opposed
Hermogenes was not a real party in interest since he the motion on the ground that the amendments
previously sold his right to the land to one Ambrocio constituted substantial alteration of the theory of
Aguilar on July 31, 1959. The case was dismissed. the defense. On February 13, 1990, the court a
Aguilar instituted on November 18, 1976 a new civil quo allowed respondents to amend the answer. When their
action before the CFI of Rizal,[5] docketed as Civil motion for reconsideration was denied, petitioners
Case No. 24873. It was similar to Civil Case No. 5957 elevated the issue directly to this court via a
except for the change in plaintiff and the addition of Petition for Certiorari. On April 25, 1990, we denied
the Bureau of Lands as co-defendant. On April 15, 1982, the petition for failure to comply with the
the lower court declared Aguilar as the absolute owner requirements of Circular 1-88, with a further
of the land and OCT No. 537 and all subsequent pronouncement that, besides, even if the petition were
certificates of title emanating therefrom as void ab admitted, the same would still be dismissed as the
initio. This decision was affirmed in toto by the Court Court finds that no grave abuse of discretion was
of Appeals. In G.R. No. 90380, we affirmed the decision committed by public respondent. Trial on the merits
of the appellate court in a decision promulgated on once more proceeded in the court a quo.
September 13, 1990.[6] While the case was on trial, complainants therein
After the April 15, 1982 decision of the CFI, and while Guillermo, Lorenzo, Domingo, Amado and Victoria, all
the case was on appeal, respondent Lopezes, as heirs children of Juan Lopez, entered into a compromise
of Hermogenes (who died on August 20, 1982), filed a agreement with the respondent Lopezes, heirs of
complaint against Aguilar before the RTC of Antipolo, Hermogenes, recognizing the latters ownership and
Rizal. The July 14, 1984 complaint was for the possession of the property subject of the case. They
cancellation of the deed of sale executed by Hermogenes confirmed the sale made by their father Juan to
in favor of Aguilar dated July 31, 1959 and/or Hermogenes. On July 20, 1992, the court a quo rendered
reconveyance. It was docketed as Civil Case No. 463- a partial decision approving the compromise
A. On February 5, 1985, the lower court declared the agreement.[8]
deed of absolute sale null and void ab initio and the On March 30, 1993, the court a quo rendered a Decision
respondents as the true and absolute owner of the dismissing the complaint, the dispositive portion of
disputed land. Aguilar sought relief with the Court of which states:
Appeals, which affirmed in toto the decision of the WHEREFORE, judgment is hereby rendered:
RTC in a Decision promulgated on August 18, 1987.[7] In 1. Ordering the dismissal of the case;
G.R. No. 81092, we denied Aguilars petition for review 2. Declaring Hermogenes Lopez as the exclusive owner
in a resolution dated April 6, 1998 for having been of the property in question;
filed late. 3. Ordering the plaintiffs to pay the defendants the
On April 25, 1985, after the RTC of Antipolo rendered amount of P20,000.00 as attorneys fees; and
its February 5, 1987 decision in Civil Case No. 463-A 4. Ordering plaintiffs to pay the costs.
and pending its appeal, respondent Lopezes sold a large SO ORDERED.[9]
portion of the disputed property to respondent spouses Feeling aggrieved, petitioners appealed to the Court
Amurao. of Appeals, which affirmed with modification the above
Decision, thus:

13
Public Land V Govnt Land – Confirmation of Imperfect or incomplete title: Natural Resources Jennica Gyrl
Delfin
Finally, We have to delete and disallow the award of Nor is there any doubt that respondents were able to
attorneys fees for want of factual and legal premise show that they have a good and substantial defense.
in the text of the appealed Decision. They attached to their affidavit of merit the following
IN VIEW OF ALL THE FOREGOING, the decision appealed documents:[14] the decision of the Court of First
from is AFFIRMED with a modification that the award of Instance of Pasig in Civil Case No. 5957 entitled
attorneys fees is deleted. Costs against the Hermogenes Lopez v. Fernando Gorospe, et al.; the
appellants.[10] decision also of the Pasig CFI, in Civil Case No.
Hence, the present course of action where petitioners 24873, entitled Ambrocio Aguilar v. Fernando Gorospe;
contend: the decisions of the lower and appellate courts in the
I. The Honorable Court of Appeals in ruling that the case of Marcelino Lopez, et al. v. Ambrocio Aguilar;
propriety of the grant of respondents petition for the decision of the Municipal Trial Court of Antipolo
relief from judgment has been rendered moot is not in in the case of Ambrocio Aguilar v. Santos; and the Deed
accord with the decisions of this Honorable Supreme of Sale executed by and between Hermogenes and his
Court. brothers - petitioner Eleuterio, Nazario and Juan. The
II. The Court of Appeals ruling that Fermin Lopez, the ruling in the foregoing cases recognized the absolute
common predecessor-in-interest, was not entitled to ownership and possession of respondents predecessor-
the grant of the homestead patent, hence petitioners in-interest, Hermogenes Lopez. The deed showed that
are not co-owners of the disputed property is not in petitioner Eleuterio, Juan and Nazario sold their
accord with the evidence and the decisions of this rights and interests in the contested lot to their
Honorable Supreme Court. brother Hermogenes.
III. The Court of Appeals ruling that the statement or Time and again, we have stressed that the rules of
declarations in the extra-judicial partition (Exh. N); procedure are not to be applied in a very strict and
the special power of attorney (Exh. O); and the letter technical sense. The rules of procedure are used only
dated January 11, 1984 (Exh. Q) were based on a wrong to help secure and not override substantial
assumption that the property is owned by their common justice.[15] If a stringent application of the rules
predecessor-in-interest -- is not in accord with the would hinder rather than serve the demands of
evidence and decisions of this Honorable Supreme Court. substantial justice, the former must yield to the
IV. The Court of Appeals committed reversible error in latter.[16]
ruling that the forged absolute deed of sale dated We now address the substantive issues. The most pivotal
September 12, 1958 has no bearing on the respondents is the petitioners contention that the appellate court
claim over the disputed property. erred in holding that they are not co-owners of the
V. The Court of Appeals in not ruling that the remedy disputed property. They argue that Fermin, their
of partition is available to the petitioners is not in predecessor-in-interest, has complied with all the
accord with law. requirements of the Public Land Act pertaining to a
VI. The Court of Appeals ruling that laches applies to homestead grant, and is therefore entitled to a patent
the herein (sic) who are close relatives is not in as a matter of right. They claim that Fermin filed a
accord with the decisions of this Honorable Supreme homestead application over the land, cultivated at
Court.[11] least one-fifth of it, and resided on it for at least
First, the procedural issue. Petitioners contend that one year. Upon his death, they argue that they became
the grant of relief from judgment is erroneous as the its co-owners through succession.
respondents did not substantiate their allegation of We do not agree. Homestead settlement is one of the
fraud, accident, mistake, or excusable negligence modes by which public lands suitable for agricultural
which unjustly deprived them of a hearing. They add purposes are disposed of.[17] Its object is to provide
that while respondents had ample opportunity to avail a home for each citizen of the state, where his family
of other remedies, such as a motion for reconsideration may shelter and live beyond the reach of financial
or an appeal, from the time they received a copy of misfortune, and to inculcate in individuals those
the decision on July 10, 1987, yet they did not do so. feelings of independence which are essential to the
Rule 38 of the 1997 Rules of Civil Procedure governs maintenance of free institutions.[18]
the petition for relief from judgment. Sections 2 and The record is bereft of any evidence as to when Fermin
3 of the Rules provide: exactly filed his homestead application over the lot
Section 2. Petition for relief from judgment, order or in controversy, but it must have been filed after 1920,
other proceedings. - When a judgment or final order is the year he first occupied and possessed the land, and
entered, or any other proceeding is thereafter taken before 1934, the year he died. During this period, Act
against a party in any court through fraud, accident, No. 2874 was the governing law.[19] Section 12 thereof
mistake or excusable negligence, he may file a petition provides:
in such court and in the same case praying that the Sec. 12. Any citizen of the Philippine Islands or of
judgment, order or proceeding be set aside.[12] the United States, over the age of eighteen years, or
Section 3. Time for filing petition; contents and the head of a family, who does not own more than twenty-
verification. - A petition provided for in either of four hectares of land in said Islands or has not had
the preceding sections of this Rule must be verified, any benefit of any gratuitous allotment of more than
filed within sixty (60) days after the petitioner twenty-four hectares of land since the occupation of
learns of the judgment, final order or other proceeding the Philippine Islands by the United States, may enter
to be set aside, and not more than six (6) months after a homestead of not exceeding twenty-four hectares of
such judgment or final order was entered or such agricultural land of the public domain.[20]
proceeding was taken; and must be accompanied with A person who is legally qualified has to file his
affidavits showing the fraud, accident, mistake or application for a homestead patent with the Bureau of
excusable negligence relied upon, and the facts Lands. If in order, the application shall be approved
constituting the petitioners good and substantial by the Director. The applicant will be authorized to
cause of action or defense, as the case may be.[13] enter the land upon payment of an entry fee of five
We find that respondents were deprived of their right pesos.[21] Within six months after approval of the
to a hearing due to accident. In the October 17, 1986 application, the applicant has to improve and cultivate
hearing, their counsel was absent due to asthma, which the land.[22]He must cultivate at least one-fifth of the
disabled him and made it difficult for him to talk. land for a period of not less than two years nor more
Similarly, when petitioners presented their than five years from the date of approval of the
evidence ex-parte on December 5, 1986, the counsel for application.[23] He must also continuously reside in the
the respondents again failed to appear as he same municipality where the homestead is located, or
experienced another severe asthma attack. On both in an adjacent municipality, for at least one
occasions, his absence is clearly excusable. year.[24] He must finally present his final proof to the

14
Public Land V Govnt Land – Confirmation of Imperfect or incomplete title: Natural Resources Jennica Gyrl
Delfin
Bureau of Lands that he has complied with the still part of alienable public land. As he applied for
cultivation and residency requirements.[25] it in his own name, his application inures to his sole
It bears emphasis that Act No. 2874 requires that for benefit. After complying with the cultivation and
an application to be valid, it must be approved by the residency requirements, he became a grantee of a
Director of Lands. This is expressly mandated by homestead patent over it, thereby making him its
Section 13 of the law, viz: absolute and exclusive owner.[33]
Sec. 13. Upon filing of an application for a homestead, Petitioners, however, claim that Hermogenes and his
the Director of Lands, if he finds that the application heirs, respondent Lopezes, recognized their rights as
should be approved, shall do so and authorize the co-owners of the disputed property, as shown by the
applicant to take possession of the land upon the following documents: an Extra-judicial Partition of
payment of ten pesos, Philippine currency, as entry the real property executed by Hermogenes and his
fee. Within six months from and after the date of the brothers - petitioner Eleuterio, Nazario, and
approval of the application, the applicant shall begin Juan;[34] a Special Power of Attorney to sell the lot
to work the homestead, otherwise he shall lose his in question executed by petitioner Eleuterio, Nazario
prior right to the land.[26] (emphasis supplied) and Juan in favor of Hermogenes;[35] and a letter dated
This provision gives the Director of Lands discretion January 16, 1984, which contains the statement that
to approve or deny an application. He is not a mere petitioners are co-heirs of the property, and which
automaton who must perfunctorily approve an respondent Marcelino Lopez signed.[36] Petitioners
application upon its filing. He is tasked to satisfy argue that respondents are precluded from denying the
himself that, among others, the application papers meet contents of these documents based on the principle of
the requirements of the law, the land is a disposable estoppel by deed. They add that while only Hermogenes
public land, and the land is not subject of a previous applied for a homestead grant, nonetheless, there was
valid application.[27] Only when he finds the an agreement among the brothers that his application
application sufficient in form and substance should he was for and in behalf of all them.
favorably act on it. Otherwise, he should deny it. These arguments fail to impress. Estoppel by deed is a
The application of Fermin unfortunately remained bar which precludes one party from asserting as against
unacted upon up to the time of his death. It was neither the other party and his privies any right or title in
approved nor denied by the Director, as the Bureau derogation of the deed, or from denying the truth of
failed to process it. Hence, he could not have acquired any material facts asserted in it. [37] The principle is
any vested rights as a homestead applicant over the that when a man has entered into a solemn engagement
property because his application was never acted upon. by deed, he shall not be permitted to deny any matter
Reliance on the cases of Davao Grains, Inc. v. which he has asserted therein, for a deed is a solemn
IAC[28] and Balboa v. Farrales[29] by the petitioners is act to any part of which the law gives effect as the
misplaced. Those two had different factual backdrops. deliberate admission of the maker.[38] It promotes the
In both Davao Grains, Inc. and Balboa, the disputed judicious policy of making certain formal documents
lots were subject of valid applications for public land final and conclusive of their contents.[39]
grants. The valid applications became our bases for A void deed, however, will not work, and may not be
ruling that once an applicant has complied with the the basis of, an estoppel.[40] Covenants do not work an
cultivation, residency and other requirements of Act estoppel unless the deed in which they are contained
No. 2874, which entitle him to a patent for a is itself a valid instrument.[41] In the case at bar,
particular tract of land, he is deemed to have already the deed and instruments at issue were void. The extra-
acquired by operation of law not only a right to a judicial partition and the special power of attorney
grant, but a grant of the government for it is not to sell did not have an object certain, which is the
necessary that a certificate of title be issued in subject matter of the deed. The disputed land cannot
order that said grant may be sanctioned by the courts be their object because petitioners do not have any
- an application therefor being sufficient under the right or interest over it. They are not its co-owners
provisions of Section 47 of Act No. 2874.[30] as it is owned absolutely by Hermogenes. Well to note,
A valid application is sadly lacking in the case of the two instruments were executed on the mistaken
Fermin. This circumstance prevented him from acquiring assumption that Hermogenes and his brothers inherited
any vested right over the land and fully owning it at the property from Fermin. Moreover, at the time the
the time of his death. Conformably, his heirs did not documents were made, Hermogenes was unaware that he
inherit any property right from him.[31] was granted a homestead patent. As correctly ruled by
Had the application of Fermin been duly approved, his the appellate court, estoppel does not operate to
heirs would have succeeded him in his rights and confer property rights where there are none.[42]
obligations with respect to the land he has applied Apropos the letter dated January 16, 1984,[43] suffice
for. Sec. 103 of Act No. 2874 covers such a it to state that we agree with the trial courts
contingency, thus: pronouncement that respondent Marcelino Lopez signed
Sec. 103. If at any time the applicant or grantee shall it merely to gain the favors of his uncle Eleuterio
die before the issuance of the patent or the final Lopez and in no way does it constitute an admission
grant of the land, or during the life of the lease, or that the plaintiffs (petitioners herein) are co-owners
while the applicant or grantee still has obligations of the property.[44] Under these circumstances,
pending towards the Government, in accordance with this respondents cannot be held guilty of estoppel by deed.
Act, he shall be succeeded in his rights and The claim of the petitioners that Hermogenes filed the
obligations with respect to the land applied for or application in behalf of all the heirs of Fermin
granted or leased under this Act by his widow, who pursuant to a previous agreement does not hold water.
shall be entitled to have issued to her the patent or There is paucity of evidence in support of this
final concession if she shows that she has complied allegation. Aside from the uncorroborated testimony of
with the requirement therefore, or in case he has left petitioner Eleuterio, petitioners were not able to
no widow or the widow refuses the succession, he shall present other proof of the agreement. Besides, we
be succeeded by the person or persons who are his heirs cannot easily give credence to such a claim considering
by law and who shall be subrogated in all his rights that under Act No. 2874, an applicant must personally
and obligations for the purposes of this Act.[32] comply with the legal requirements for a homestead
The failure of the Bureau of Lands to act on the grant. He must possess the necessary qualifications.
application of Fermin up to the time of his death, He must cultivate the land and reside on it himself.
however, prevented his heirs to be subrogated in all It would be a circumvention of the law if an individual
his rights and obligations with respect to the land were permitted to apply in behalf of another, as the
applied for. latter may be disqualified or might not comply with
Perforce, at the time Hermogenes applied for a the residency and cultivation requirements.
homestead grant over the disputed property, it was

15
Public Land V Govnt Land – Confirmation of Imperfect or incomplete title: Natural Resources Jennica Gyrl
Delfin
In respect of the fourth assigned error, we find that At the crux of the controversy is Lot No. 1-GSS-569
petitioners attack on the authenticity and validity of (subject property), located in San Teodoro, Oriental
the Deed of Absolute Sale dated September 12, 1958, Mindoro, with an area of 6.2820 hectares, and covered
where petitioner Eleuterio, Juan, and Nazario by Original Certificate of Title (OCT) No. P-
allegedly sold their share in the disputed property to 58854 issued on July 21, 1965 by respondent ROD in
Hermogenes, bereft of merit. It did not change the fact respondent Roxas’s name.
that no co-ownership existed among Hermogenes and his The controversy arose from the following facts:
brothers. Hermogenes is the absolute owner of the On February 5, 1941, then President Manuel L. Quezon
disputed property just as his brothers do not own any (Quezon) issued Proclamation No. 678,5 converting
share in it. Hence, they cannot validly sell anything forest land measuring around 928 hectares, situated in
to Hermogenes by virtue of the deed. San Teodoro, Oriental Mindoro, described on Bureau of
Prescinding from the lack of co-ownership, petitioners Forestry Map No. F. R.-110, VICENTE ROXAS AND THE
argument that they are entitled to have the land REGISTER OF DEEDS OF ORIENTAL MINDORO as Matchwood
partition must be rejected. Partition, in general, is Forest Reserve. The Matchwood Forest Reserve was placed
the separation, division and assignment of a thing held under the administration and control of the Bureau of
in common among those to whom it may belong.[45] The Forestry, "which shall have the authority to regulate
purpose of partition is to put an end to co- the use and occupancy of this reserve, and the cutting,
ownership.[46] It seeks a severance of the individual collection and removal of timber and other forest
interests of each co-owner, vesting in each a sole products therein in accordance with the Forest Law and
estate in specific property and giving to each one a Regulations."6 For the foregoing purpose, President
right to enjoy his estate without supervision or Quezon withdrew the 928 hectares of forest land
interference from the other.[47] Not being co-owners of constituting the Matchwood Forest Reserve from entry,
the disputed lot, petitioners cannot demand its sale, or settlement, subject to private rights, if
partition. They do not have any interest or share in there be any.
the property upon which they can base their demand to Petitioner Republic, through the Department of
have it divided. Agriculture and Natural Resources (DANR), entered into
Petitioners last argument that they are not guilty of Matchwood Plantation Lease Agreement No. 1 with
laches in enforcing their rights to the property is petitioner PTFI on May 12, 1965, wherein petitioner
irrelevant. Laches is the negligence or omission to Republic leased the entire Matchwood Forest Reserve to
assert a right within a reasonable time, warranting a petitioner PTFI for a period of 25 years, which would
presumption that the party entitled to assert it has expire on June 30, 1990.
abandoned it or declined to assert it.[48] It does not In the meantime, respondent Roxas filed with the Bureau
involve mere lapse or passage of time, but is of Lands7 on December 29, 1959 Homestead Application
principally an impediment to the assertion or No. 9-5122, covering a parcel of land he initially
enforcement of a right, which has become under the identified as Lot No. 4, SA-22657, located at Paspasin,
circumstances inequitable or unfair to San Teodoro, Oriental Mindoro. Following the report
permit.[49] Petitioners insistence that they are not and recommendation8 of Land Inspector (LI) Domingo Q.
negligent in asserting their right over the property Fernandez (Fernandez), Officer-in-Charge (OIC) Jesus
proceeds from the wrong premise that they have a right B. Toledo (Toledo), for and by the authority of the
to enforce over the disputed property as co-owners. Director of Lands, issued an Order dated September 20,
There can be no delay in asserting a right where the 1961 amending respondent Roxas’s Homestead Application
right does not exist. No. 9-5122, to wit:
IN VIEW WHEREOF, finding no cogent reason to reverse It having been found upon investigation conducted by a
the impugned Decision of the Court of Appeals, the representative of this Office that the land actually
petition is DENIED for lack of merit. occupied by the applicant is Lot No. 1, SA-22657 Amd.,
SO ORDERED. and not Lot No. 4 of the same subdivision as applied
for, and it appearing in the records of this Office
G.R. No. 157988 December 11, 2013 that the land actually occupied is free from claims
REPUBLIC OF THE PHILIPPINES-BUREAU OF FOREST and conflicts, the above-noted application is hereby
DEVELOPMENT, Petitioner, amended to cover Lot No. 1, SA-22657 Amd., and as thus
vs. amended, shall continue to be given due course.9
VICENTE ROXAS AND THE REGISTER OF DEEDS OF ORIENTAL OIC Toledo subsequently issued another Order dated
MINDORO, Respondents. September 27, 1961 which approved respondent Roxas’s
x------------------------------------------------x Homestead Application No. 9-5122 and recorded the same
G.R. No. 160640 as Homestead Entry No. 9-4143.10 Thereafter, respondent
PROVIDENT TREE FARMS, INC., Petitioner, Roxas executed a Notice of Intention to Make Final
vs. Proof, which was posted on September 23,
VICENTE ROXAS AND THE REGISTER OF DEEDS OF ORIENTAL 1963.11Respondent Roxas personally testified before LI
MINDORO, Respondents. Fernandez on October 25, 1963 to finally prove his
D E C I S I O N residence and cultivation of the subject property.
LEONARDO-DE CASTRO, J.: In a letter dated July 12, 1965, Assistant District
Before Us are consolidated Petitions for Review on Forester Luis G. Dacanay (Dacanay), Bureau of Forestry,
Certiorari under Rule 45 of the Rules of Court: (1) DANR, informed the District Land Officer of Calapan,
G.R. No. 157988, filed by petitioner Republic of the Oriental Mindoro, that "the subject-area designated as
Philippines (Republic), represented by the Bureau of Lot No. 1, Gss-569, has been verified to be within the
Forest Development (BFD),1 and (2) G.R. No. 160640, alienable and disposable land of Project 18 of San
filed by petitioner Provident Tree Farms, Inc. (PTFI), Teodoro, Oriental Mindoro, per B.F. Map LC-1110
both against respondents Vicente Roxas (Roxas) and the certified as such on September 30, 1934."12 Assistant
Register of Deeds (ROD) of Oriental Mindoro, assailing District Forester Dacanay further wrote in the same
the joint Decision2 dated April 21, 2003 of the Court letter that "[t]he said land is no longer within the
of Appeals in CA-G.R. CV No.44926, which, in turn, administrative jurisdiction of the Bureau of Forestry,
affirmed the Decision3 dated February 10, 1994 of the so that, its disposition in accordance with the Public
Regional Trial Court (RTC), Branch 39 of Oriental Land Law does not adversely affect forestry interest
Mindoro, in Civil Case No. R-3110. The RTC dismissed anymore."13
the Complaint for Cancellation of Title and/or The Director of Lands issued Homestead Patent No.
Reversion filed by petitioner Republic against 11159814 to respondent Roxas on July 19, 1965, on the
respondents Roxas and the ROD of Oriental Mindoro. basis of which, respondent ROD issued OCT No. P-5885
Petitioner PTFI was an intervenor in Civil Case No. R- in respondent Roxas’s name on even date,15 with the
3110, as a lessee of petitioner Republic.

16
Public Land V Govnt Land – Confirmation of Imperfect or incomplete title: Natural Resources Jennica Gyrl
Delfin
following technical description of the subject 4. Roxas’s Final Proof Homestead Testimony of
property: Applicant;21
Lot No. 1, Gss-569 5. Assistant District Forester Dacanay’s letter dated
Beginning at a point marked "1" of Lot 1, Gss-569, July 12, 1965 to the District Land Officer of Calapan,
being N. 32-15 Oriental Mindoro, verifying that Lot No. 1, GSS-569,
W., 1396.63 m. from BBM No. 3, Cad-104, thence was alienable and disposable;22
S.36-38 W., 168.79m. to point 2; S.80-16 W., 46.02m. 6. Blue Print Plan of Land Group Settlement Survey as
to point 3; surveyed forthe Republic;23
S.33-22 W., 63.40m. to point 4; S.77-05 W., 17.28m. to 7. Order dated July 19, 1965 of the Director of Lands
point 5; approving Roxas’s application for patent;24
N.52.06 W., 137.92m. to point 6; N.40-51 E., 417.50m. 8. The unsigned letter dated July 19, 1965 of Gabriel
to point 7; Sansano, Chief, Records Division, Bureau of Lands, to
S.54-25 E., 115.36m. to point 8; S.24-20 W., 146.33m. the ROD of Calapan, Oriental Mindoro, transmitting
to point 1; Roxas’s Homestead Patent No. 111598 for the
point of beginning. registration and issuance of Owner’s Duplicate
Containing an area of SIXTY[-]TWO THOUSAND EIGHT Certificate of Title in accordance with Section 122,
HUNDRED AND TWENTY (62,820) SQUARE METERS. Act No. 496;25 and
All points are marked on the ground as follows: points 9. OCT No. P-5885 in Roxas’s name.26
3 & 4 by Stakes, and the rest by B.L. Cyl. Conc. Mons. Respondent Roxas maintained that OCT No. P-5885 had
Bounded on the SE., along line 1-2 by Lot 2, Gss-569; been legally and validly issued to him and that he had
on the S., along lines 2-3-4-5 by Road; on the SW., been in actual, open, and continuous possession of the
and NW., along lines 5-6-7 by Match Wood Forest subject property in the concept of an owner since 1959.
Reservation; on the NE., along line 7-8 by Lot 4, Gss- Respondent Roxas then prayed that judgment be rendered
569; and on the E., along line 8-1 by Lot 3, Gss-569. dismissing the Complaint of petitioner Republic;
Bearings true. awarding damages to him in the amount of ₱500.00 and
This lot was surveyed in accordance with law and attorney’s fees in the amount of ₱2,000.00; and
existing regulations promulgated thereunder, by R.F. declaring OCT No. P-5885 free from all claims and
Javier, Public Land Surveyor, on October 5, 1959. conflicts.
NOTE: Petitioner PTFI eventually filed a Complaint for
This lot is covered by H.A. No. 9-5122.16 Intervention on the ground that it was leasing the
On May 2, 1978, petitioner Republic, represented by entire Matchwood Forest Reserve from petitioner
the BFD, filed with the RTC a Complaint for Republic under Matchwood Plantation Lease Agreement
Cancellation of Title and/or Reversion against No. 1 for a period of 25 years that would expire on
respondents Roxas and the ROD over the subject June 30, 1990.27
property, docketed as Civil Case No. R-3110.17 The RTC granted the intervention of petitioner PTFI in
Petitioner Republic alleged that the subject property an Order dated August 10, 1979.28
was within the Matchwood Forest Reserve and could not Subsequently, during the pendency of Civil Case No. R-
be the subject of private appropriation and ownership; 3110 before the RTC, and considering the expiration of
and possession of said property, no matter how long Lease Agreement No. 1 in 1990, petitioner PTFI entered
would not convert the same into private property. The into an Industrial Tree Plantation Lease
Director of Lands could not dispose of the subject Agreement29 dated November 11, 1982 and Industrial
30
property under the provisions of Commonwealth Act No. Forest Plantation Management Agreement dated November
141, otherwise known as the Public Land Act, thus, OCT 24, 1982 with petitioner Republic, which extended the
No. P-5885 issued in respondent Roxas’s name was null lease of petitioner PTFI of the Matchwood Forest
and void ab initio. Petitioner Republic also averred Reserve until July 7, 2007.
that respondent Roxas acquired OCT No. P-5885 through To determine whether or not the subject property was
fraud and misrepresentation, not only because the within theMatchwood Forest Reserve, the RTC issued an
subject property was not capable of registration, but Order dated June 23, 1983 creating a committee to
also because respondent Roxas was disqualified to conduct a relocation survey. The committee was composed
acquire the same under the provisions of the Public of three competent government officials: (1) the
Land Act, not having exercised acts of possession in District Land Officer of Calapan, Oriental Mindoro, as
the manner and for the length of time required by law. chairman; (2) Geodetic Engineer (Engr.) Narciso Mulles
The Director of Lands was only misled into approving (Mulles) of the BFD; and (3) Geodetic Engineer Cresente
respondent Roxas’s application for homestead patent. Mendoza (Mendoza) of the Bureau of Lands, Calapan,
Petitioner Republic additionally mentioned that the Oriental Mindoro.31 However, Engr. Mulles was assigned
subject property, as part of the Matchwood Forest to Region V, Naga City, so no relocation survey was
Reserve, was included in the lease agreement of conducted. Thus, the RTC issued another Order dated
petitioner Republic with petitioner PTFI. March 15, 1984, creating a second relocation survey
In his Answer, respondent Roxas admitted applying for committee composed of District Forester Gregorio O.
and acquiring a homestead patent over the subject Nisperos (Nisperos) as team leader, with
property. Respondent Roxas, however, denied that the representatives of the District Land Office,
subject property was within the Matchwood Forest respondent Roxas, and petitioner PTFI as members.32
Reserve. To the contrary, the subject property was part The committee submitted to the RTC a Memorandum dated
and parcel of the Paspasin Group Settlement May 11, 1984, prepared by Engr. Mendoza, the
Subdivision, SA-22657, and had been the subject of representative of the Bureau of Lands, and
investigation in accordance with law, rules, and countersigned by District Forester Nisperos, the team
regulations, as established by documentary evidence, leader, presenting the results of the ocular
viz: inspection/survey work conducted by the committee from
1. LI Fernandez’s letter dated February 28, 1961 April 23 to 29, 1984 and the recommendations of the
addressed to the Director of Lands, Manila, reporting committee. Pertinent parts of the Memorandum read:
that Roxas was actually applying for Lot No. 1, not REMARKS: [W]e are submitting herewith the result of
Lot No. 4, of the Paspasin Group Settlement our ocularinspection/survey work undertaken during the
Subdivision, SA-22657 Amd., and recommending that period from April 23 to 29, 1984 in the presence of
Roxas’s application be corrected accordingly;18 Engineer Cresente M. Mendoza, Bureau of Lands (B.L.)
2. OIC Toledo’s Order dated September 27, 1961 representative, Mr. Reynaldo Labay, Bureau of Forest
approving Roxas’s application for homestead patent;19 Development (BFD) representative and Mr. Vicente
3. Roxas’s Notice of Intention to Make Final Proof, Roxas, the defendant. Findings and other related
together with his Affidavit that the said Notice was informations gathered during the survey disclosed the
accordingly posted;20 following:

17
Public Land V Govnt Land – Confirmation of Imperfect or incomplete title: Natural Resources Jennica Gyrl
Delfin
1. The titled land property claimed by Mr. Vicente plan because it was signed by Acting Regional Land
Roxas (defendant) situated at Barangay Paspasin, San Director Villapando. He further avowed that points
Teodoro, Oriental Mindoro which is subject of the BFFR-45, BFFR-46, and BFFR-47-A were still intact
complaint and inquiry covering an area of about 6.282 during the relocation survey by the committee, marked
hectares is located inside the Matchwood Forest Reserve by monuments which he believed were previously placed
No. 1 under Presidential Proclamation No. 678 dated by the people from the BFD.37
February 5, 1941 per F.R. 110 and leased to Provident Daniel de los Santos (De los Santos), a Geodetic
Tree Farms, Inc. Engineer from the Department of Environment and Natural
2. The whole land area falls inside said forest reserve Resources (DENR), Regional Office IV, also testified
reckoning from established BFFR corners (BFFR Corner for petitioners. According to Engr. De los Santos, his
Nos. 45, 46 & 47-A) as shown in the attached sketch/map supervisor showed him OCT No. P-5885 and instructed
plan. The issuance of the Original Certificate of Title him to prepare a plotting on the land classification
to herein defendant inside a proclaimed Forest Reserve map. Engr. De los Santos presented two maps before the
would not warrant nor justify the validity of RTC, both coming from the National Mapping Resources
legitimate and/or rightful ownership over said titled Administration: (1) the Land Classification, Province
land property considering the present status of the of Oriental Mindoro LC-1110 dated August 30, 1934
subject land area under question, therefore it could (marked as Exhibit "J") and (2) the Land
not complete its right under the provisions of the Classification, Province of Oriental Mindoro LC-2244
Public Land Law. dated December 15, 1958 (marked as Exhibit "K"). Engr.
ACTION RECOMMENDED: In view of the above-mentioned De los Santos demonstrated table plotting on both land
facts gathered by the team and after judicious scrutiny classification maps using the technical description of
of other informations surrounding the subject case, it the subject property as appearing on OCT No. P-5885,
is hereby recommended that the Original Certificate of which showed that the subject property fell within the
Title issued to Mr. Vicente Roxas covering a land area forest reserve.38 When cross-examined, Engr. De los
located inside the Matchwood Forest Reserve be annulled Santos reiterated that he based his plotting on the
and the retention of said area for which they have been technical description of the subject property as it
reserved. Should the Honorable Court needs some appeared on OCT No. P-5885. He did not consider Lot
clarification on the survey conducted, it is No. 1 of GSS-569 in his plotting because he was not
recommended further that Engineer Cresente M. Mendoza aware of the same.
of the Bureau of Lands, Calapan be sub-phoenaed (sic).33 Respondent Roxas himself testified for the defense.
Petitioner Roxas contested the results of the RespondentRoxas recounted that he originally joined
relocation surveyconducted by the committee, hence, in the Philippine Army in 1941, but he joined the guerilla
an Order dated August 6, 1984, the RTC directed the movement in Oriental Mindoro during the Japanese
Clerk of Court to issue a subpoena to committee members occupation, and thereafter, he re-enlisted with the
Engr. Mendoza of the Bureau of Lands and Mr. Reynaldo United States Armed Forces in the Far East (USAFFE).
Labay (Labay) of the BFD to appear before the court; Respondent Roxas was first struck with the pleasant
and a subpoena duces tecum to the District Land Officer appearance of the subject property while he was still
or his duly authorized representative to bring and in the guerilla movement, and when he retired from the
produce pertinent papers relative to cadastral survey USAFFE in 1946, he cleaned the said property, which
104 in respondent Roxas’s name.34 was still woody at that time. Respondent Roxas built a
Engr. Mendoza attested that pursuant to the RTC Order nipa hut on the subject property where he and his wife,
dated March 15, 1984, he conducted a relocation survey as well as their children, had resided, and planted
of the subject property on April 23-29, 1984. After the same with palay and bananas to sustain his family.
the said survey, he personally prepared the Plan of Sometime in 1959, a certain Luz Alegre filed a sales
Lots 1 (owned by respondent Roxas), 4 (owned by Esteban application for the subject property occupied by
Paroninog), and 5 (no registered owner, adjacent to respondent Roxas and adjoining parcels of land occupied
Lot 4), GSS-569, as relocated forVicente Roxas v. by 20 other residents. Respondent Roxas and the other
Republic of the Philippines (BFD). In the Plan, Engr. residents were spurred to petition the Bureau of Lands
Mendoza marked the boundary between the forest zone to have their respective properties surveyed. It was
and the released area by drawing a line from BFFR-45 then that respondent Roxas came to know that he had
to BFFR-46 to BFFR-47-A, which showed that Lot 1 owned developed the subject property to the extent of 6.2820
by respondent Roxas was found inside the forest zone.35 hectares. After the survey of the subject property,
On cross-examination, Engr. Mendoza acknowledged that respondent Roxas began planting thereon about 700
even before the committee conducted the relocation coconut trees, 500 calamansi trees, 200 rambutan trees,
survey, he already knew that the subject property was 50 sinturis trees, and 30 cacao trees, plus an
part of the Matchwood Forest Reserve. During the unspecified number of other trees such as abaca,
relocation survey, Engr. Mendoza did not take into banana, and mango.39
consideration the total area of the reserve since he The RTC rendered a Decision on February 10, 1994, in
had no idea as to the same. He merely relocated BFFR- respondent Roxas’s favor. The RTC declared that
45, BFFR-46, and BFFR-47-A. Per record of the BFD, the petitioner PTFI had no right whatsoever to the subject
line drawn from BFFR-45 until BFFR-47-A was the property since the latter’s lease agreement with
boundary line between the forest zone and the released petitioner Republic had already expired on June 30,
areas. Engr. Mendoza was then asked to compare the Plan 1990. It also held that the preponderance of evidence
he prepared based on the relocation survey conducted showed that the subject property was outside the forest
by the committee on April 23-29, 1984 vis-à-vis the reserve and part of the alienable and disposable lands
Plan of Land Group Settlement Survey, GSS-569, prepared of the public domain; and that there was no proof at
by Engr. Restituto Javier (Javier) and approved (for all of fraud or misrepresentation on respondent Roxas’s
the Director of Lands) by Acting Regional Land Director part in procuring OCT No. P-5885. In the end, the RTC
Narciso Villapando (Villapando), as a result of the decreed:
survey conducted on September 21-22 and October 5-19, ACCORDINGLY, judgment is hereby rendered:
1959. Engr. Mendoza conceded that Lot 1 indicated in 1. Dismissing the complaint; and
both plans in respondent Roxas’s name were the 2.Ordering the plaintiff Republic of the Philippines
same,36 but in the Plan of the Land Group Settlement (Bureau of Forest Development) and plaintiff
Survey, GSS-569, the boundary line separating the intervenor Provident Tree Farms, to pay jointly and
forest reserve from the released areas was just above severally defendant Vicente Roxas ₱25,000.00 for and
Lots 1, 4, and 5. as attorney’s fees and expenses of litigation and the
During redirect examination, Engr. Mendoza explained costs of suit.40
that he came upon the conclusion that the Plan of the
Land Group Settlement Survey, GSS-569, was the approved

18
Public Land V Govnt Land – Confirmation of Imperfect or incomplete title: Natural Resources Jennica Gyrl
Delfin
Unsatisfied with the foregoing RTC Decision, THE COURT OF APPEALS ERRED IN NOT FINDING THAT PRIVATE
petitioners jointly filed an appeal before the Court RESPONDENT PROCURED HOMESTEAD PATENT NO. 111598 AND
of Appeals, docketed as CA-G.R. CV No. 44926. ORIGINAL CERTIFICATE OF TITLE NO. P-5885 THROUGH FRAUD
In its Decision dated April 21, 2003, the Court of AND/OR MISREPRESENTATION.
Appeals sustained the appreciation of evidence by the IV
RTC, thus: THE COURT OF APEPALS ERRED IN CONCLUDING THAT
Before Roxas could be issued his corresponding PRESCRIPTION IS APPLICABLE TO THIS CASE.43
homestead patent, the Bureau of Forestry of the Meanwhile, petitioner PTFI first filed a Motion for
Department of Environment and Natural Resources Reconsideration44 with the Court of Appeals. After the
declared that: appellate court denied said Motion in a Resolution
"I have the honor to inform you that the subject area dated October 30, 2003,45 petitioner PTFI likewise
designated as Lot No. 1 Gss-569, has been verified to sought recourse from us through a Petition for Review
be within the alienable and disposable land of Project on Certiorari, docketed as G.R. No. 160640, assailing
No. 18 of San Teodoro, Oriental Mindoro per B.F. LC- the Court of Appeals judgment on the following grounds:
110 certified as such on September 30, 1934. I
The said land is no longer within the administrative THE COURT OF APPEALS’ REFUSAL TO ACCORD CREDENCE TO
jurisdiction of the Bureau of Forestry, so that, its THE TESTIMONIES OF EXPERTS IS CONTRARY TO LAW AND
disposition in accordance with the Public Land Law does JURISPRUDENCE.
not adversely affect forestry interest anymore." II
Not only does this letter prove that Lot 1-GSS-569, THE COURT OF APPEALS ACTED CONTRARY TO LAW AND
the area occupied and titled in the name of Roxas, is JURISPRUDENCE ON THE INALIENABILITY OF PUBLIC LANDS
alienable and disposable but so does the 1959 Survey WHEN IT AFFIRMED THE DECISION OF THE TRIAL COURT.
Plan, which with its dotted lines confirm that the land III
of Roxas is outside the Matchwood Forest Reserve. THE COURT OF APPEALS CONTRAVENED EXISTING LAW AND
Even the 1984 Relocation Survey conducted by Cresente JURISPRUDENCE WHEN IT CONCLUDED THAT THE INSTANT ACTION
Mendoza on the subject property showed it to be on the IS BARRED BY PRESCRIPTION AND THE PRINCIPLE OF
same location. x x x. INDEFEASIBILITY OF TITLE.46
x x x x In a Resolution47 dated December 8, 2004, we
The court a quo was correct when it did not give consolidated G.R. No. 160640 with G.R. No. 157988.
credence to the testimony of [Cresente] Mendoza that Sifting through the arguments raised by the parties,
the subject lot is within the Matchwood Forest Reserve we identify three fundamental issues for our
area because despite having performed a relocation resolution, particularly: (1) whether the subject
survey in the area, he admitted that he does not know property is forest land or alienable and disposable
the actual area of the forest reserve. x x x. agricultural land; (2) whether respondent Roxas
x x x x procured OCT No. P-5885 through fraud and
And though another witness, Geodetic Engineer Daniel misrepresentation; and (3) whether petitioner Republic
de los Santos, did a table plotting of the two Land is barred by estoppel and prescription from seeking
Classification Maps, it appears that the subject Lot the cancellation of OCT No. P-5885 and/or reversion of
1-GSS-569 was not actually included in the plotting. x the subject property.
x x.41(Citations omitted.) Review of the findings of fact of the
The Court of Appeals also ruled that respondent Roxas’s RTC and Court of Appeals is proper
compliance with substantive and procedural in this case
requirements for acquisition of public lands belied Before delving into the merits, the propriety of these
the allegation that respondent Roxas obtained grant Petitions for Review under Rule 45 of the Rules of
and title over the subject property through fraud and Court should first be addressed. We note at the outset
misrepresentation. The appellate court further that except for the third issue on estoppel and
pronounced that once a patent had been registered and prescription, the other two issues involve questions
the corresponding certificate of title had been issued, of fact that necessitate a review of the evidence on
the land covered by them ceased to be part of the record. In Decaleng v. Bishop of the Missionary
public domain and became private property; and the District of the Philippine Islands of Protestant
Torrens title issued pursuant to the patent became Episcopal Church in the Unites States of America,48 we
indefeasible upon the expiration of one year from the presented the general rule, as well as the exceptions,
date of the issuance of the patent. The Court of to the same:
Appeals, however, disagreed with the RTC in awarding Prefatorily, it is already a well-established rule that
attorney’s fees, expenses of litigation, and costs of the Court, in the exercise of its power of review under
suit to respondent Roxas, finding no basis for such Rule 45 of the Rules of Court, is not a trier of facts
awards. and does not normally embark on a re-examination of
Ultimately, the Court of Appeals disposed of CA-G.R. the evidence presented by the contending parties during
CV No. 44926 in this wise: the trial of the case, considering that the findings
WHEREFORE, except for the award of attorney’s fees, of facts of the Court of Appeals are conclusive and
expenses of litigation and costs of suit which are binding on the Court. This rule, however, admits of
hereby DELETED, the appealed Decision is otherwise exceptions as recognized by jurisprudence, to wit:
AFFIRMED.42 (1) [W]hen the findings are grounded entirely on
Petitioner Republic, through the BFD, directly filed speculation, surmises or conjectures; (2) when the
its Petition for Review on Certiorari before us, inference made is manifestly mistaken, absurd or
docketed as G.R. No. 157988. Petitioner Republic impossible; (3) when there is grave abuse of
assigned the following errors on the part of the Court discretion; (4) when the judgment is based on
of Appeals: misapprehension of facts; (5) when the findings of
I facts are conflicting; (6) when in making its findings
THE COURT OF APPEALS ERRED IN DECLARING THAT LOT NO. the Court of Appeals went beyond the issues of the
1, GSS-569 IS NOT PART OF THE MATCHWOOD FOREST RESERVE. case, or its findings are contrary to the admissions
II of both the appellant and the appellee; (7) when the
THE COURT OF APPEALS ERRED IN DISREGARDING THE findings are contrary to the trial court; (8) when the
TESTIMONY OF ENGINEER CRESENCIO MENDOZA THAT THE findings are conclusions without citation of specific
SUBJECT LOT IS WITHIN THE MATCHWOOD FOREST RESERVE AREA evidence on which they are based; (9) when the facts
ON THE SOLE BASIS OF HIS ADMISSION THAT HE DID NOT KNOW set forth in the petition as well as in the
THE ACTUAL AREA OF THE FOREST RESERVE. petitioner’s main and reply briefs are not disputed by
III the respondent; (10) when the findings of fact are
premised on the supposed absence of evidence and

19
Public Land V Govnt Land – Confirmation of Imperfect or incomplete title: Natural Resources Jennica Gyrl
Delfin
contradicted by the evidence on record; and (11) when (d) Reservations for townsites and for public and
the Court of Appeals manifestly overlooked certain quasi-public uses.
relevant facts not disputed by the parties, which, if The President, upon recommendation by the Secretary of
properly considered, would justify a different Agriculture and Natural Resources, shall from time to
conclusion. (Citations omitted.) time make the classifications provided for in this
The case at bar falls under several exceptions, i.e., section, and may, at any time and in a similar manner,
the inference made is manifestly mistaken, absurd, or transfer lands from one class to another. (Emphases
impossible; the judgment is based on misapprehension ours.)
of facts; and the findings of fact are contradicted by By virtue of Presidential Decree No. 705, otherwise
the evidence on record.As a result, we must return to known as the Revised Forestry Code,50 the President
the evidence submitted by the parties during trial and delegated to the DENR Secretary the power to determine
make our own evaluation of the same. which of the unclassified lands of the public domain
Subject property is within the are (1) needed for forest purposes and declare them as
Matchwood Forest Reserve and, permanent forest to form part of the forest reserves;
thus, inalienable and not subject to and (2) not needed for forest purposes and declare them
disposition. as alienable and disposable lands.51
Under the Regalian doctrine, which is embodied in Per the Public Land Act, alienable and disposable
Article XII, Section 2 of our Constitution, all lands public lands suitable for agricultural purposes can be
of the public domain belong to the State, which is the disposed of only as follows:
source of any asserted right to any ownership of land. 1.For homestead settlement;
All lands not appearing to be clearly within private 2.By sale;
ownership are presumed to belong to the State. 3.By lease; and
Accordingly, public lands not shown to have been 4.By confirmation of imperfect or incomplete titles:
reclassified or released as alienable agricultural (a)By judicial legalization;
land or alienated to a private person by the State (b)By administrative legalization (free patent).52
remain part of the inalienable public domain.49 Homestead over alienable and disposable public
Commonwealth Act No. 141, also known as the Public Land agricultural land is granted after compliance by an
Act, asamended by Presidential Decree No. 1073, remains applicant with the conditions and requirements laid
to this day the existing general law governing the down under Title II, Chapter IV of the Public Land Act,
classification and disposition of lands of the public the most basic of which are quoted below:
domain, other than timber and mineral lands. The SEC. 12. Any citizen of the Philippines over the age
following provisions under Title I, Chapter II of the of eighteen years, or the head of a family, who does
Public Land Act, as amended, is very specific on how not own more than twenty-four hectares of land in the
lands of the public domain become alienable or Philippines or has not had the benefit of any
disposable: gratuitous allotment of more than twenty-four hectares
SEC. 6. The President, upon the recommendation of the of land since the occupation of the Philippines by the
Secretary of Agriculture and Natural Resources, shall United States, may enter a homestead of not exceeding
from time to time classify the lands of the public twenty-four hectares of agricultural land of the public
domain into: domain.
(a)Alienable or disposable, SEC. 13. Upon the filing of an application for a
(b)Timber, and homestead, the Director of Lands, if he finds that the
(c)Mineral lands, application should be approved, shall do so and
and may at any time and in a like manner transfer such authorize the applicant to take possession of the land
lands from one class to another, for the purposes of upon the payment of five pesos, Philippine currency,
their administration and disposition. as entry fee. Within six months from and after the date
SEC. 7. For the purposes of the administration and of the approval of the application, the applicant shall
disposition of alienable or disposable public lands, begin to work the homestead, otherwise he shall lose
the Batasang Pambansa or the President, upon his prior right to the land.
recommendation by the Secretary of Natural Resources, SEC. 14. No certificate shall be given or patent issued
may from time to time declare what public lands are for the land applied for until at least one-fifth of
open to disposition or concession under this Act. the land has been improved and cultivated. The period
x x x within which the land shall be cultivated shall not be
SEC. 8. Only those lands shall be declared open to less than one nor more than five years, from and after
disposition or concession which have been officially the date of the approval of the application. The
delimited and classified and, when practicable, applicant shall, within the said period, notify the
surveyed, and which have not been reserved for public Director of Lands as soon as he is ready to acquire
or quasi-public uses, nor appropriated by the the title. If at the date of such notice, the applicant
Government, nor in any manner become private property, shall prove to the satisfaction of the Director of
nor those on which a private right authorized and Lands, that he has resided continuously for at least
recognized by this Act or any other valid law may be one year in the municipality in which the land is
claimed, or which, having been reserved or located, or in a municipality adjacent to the same and
appropriated,have ceased to be so. However, the has cultivated at least one-fifth of the land
President may, for reasons of public interest, declare continuously since the approval of the application,
lands of the public domain open to disposition before and shall make affidavit that no part of said land has
the same have had their boundaries established or been been alienated or encumbered, and that he has complied
surveyed, or may, for the same reason, suspend their with all the requirements of this Act, then, upon the
concession or disposition until they are again declared payment of five pesos, as final fee, he shall be
open to concession or disposition by proclamation duly entitled to a patent.
published or by Act of the Congress. It is clear under the law that only alienable and
SEC. 9. For the purpose of their administration and disposable agricultural lands of the public domain can
disposition, the lands of the public domain alienable be acquired by homestead.
or open to disposition shall be classified, according In the instant case, respondent Roxas applied for and
to the use or purposes to which such lands are was granted Homestead Patent No. 111598 for the subject
destined, as follows: property, pursuant to which, he acquired OCT No. P-
(a) Agricultural; 5885 in his name. The problem, however, is that the
(b) Residential, commercial, industrial, or for subject property is not alienable and disposable
similar productive purposes; agricultural land to begin with.
(c) Educational, charitable, or other similar The burden of proof in overcoming the presumption of
purposes; and State ownership of lands of the public domain is on

20
Public Land V Govnt Land – Confirmation of Imperfect or incomplete title: Natural Resources Jennica Gyrl
Delfin
the person applying for registration, or in this case, through the Director of Lands approved on March 6,
for homestead patent. The applicant must show that the 1925, the survey plan (Psu-43639) for Salming Piraso.
land subject of the application is alienable or The existence of a sketch plan of real property even
disposable.53 It must be stressed that incontrovertible if approved by the Bureau of Lands is no proof in
evidence must be presented to establish that the land itself of ownership of the land covered by the plan.
subject of the application is alienable or (Gimeno v. Court of Appeals, 80 SCRA 623). The fact
disposable.54 that a claimant or a possessor has a sketch plan or a
The Court of Appeals, in its assailed Decision, survey map prepared for a parcel of land which forms
concluded that the subject property is indeed alienable part of the country’s forest reserves does not convert
and disposable based on the (1) Letter dated July 12, such land into alienable land, much less private
1965 of Assistant District Forester Dacanay to the property. Assuming that a public officer erroneously
District Land Officer of Calapan, Oriental Mindoro approves the sketch plan, such approval is null and
informing the latter that Lot 1, GSS-569 was verified void. There must first be a formal Government
to be within the alienable and disposable land of declaration that the forest land has been re-classified
Project 18 of San Teodoro, Oriental Mindoro per B.F. into alienable and disposable agricultural land which
Map LC-1110; and (2) the Blue Print Plan of the Land may then be acquired by private persons in accordance
Group Settlement Survey, GSS-569, showing that the with the various modes of acquiring public agricultural
subject property lies beyond the Matchwood Forest lands.
Reserve. But these are hardly the kind of proof In stark contrast, more than just the presumption under
required by law. the Regalian doctrine, there is actually Presidential
As we pronounced in Republic of the Phils. v. Tri-Plus Proclamation No. 678 dated February 5, 1941, declaring
Corporation,55 to prove that the land subject of an around 928 hectares of forest land as Matchwood Forest
application for registration is alienable, an Reserve, which had been withdrawn from entry, sale, or
applicant must establish the existence of a positive settlement. Two geodetic engineers, namely, (1) Engr.
act of the Government such as a presidential Mendoza, who conducted an ocular inspection/relocation
proclamation or an executive order, an administrative survey in 1984 upon orders of the RTC; and (2) Engr.
action, investigation reports of Bureau of Lands De los Santos, who performed table plotting of the
investigators, and a legislative act or statute. The technical description of the subject property on land
applicant may also secure a certification from the classification maps, testified before the RTC that the
Government that the lands applied for are alienable subject property is within the Matchwood Forest
and disposable. Reserve.
We were even more specific in Republic of the Phils. Both the RTC and the Court of Appeals erred in brushing
v. T.A.N. Properties, Inc.56 as to what constitutes aside the testimonies of the two engineers on very
sufficient proof that a piece of land is alienable and tenuous grounds. Engr. Mendoza need not know the entire
disposable, to quote: area of the Matchwood Forest Reserve, such fact being
Further, it is not enough for the PENRO or CENRO to insignificant to the issue at hand. What Engr. Mendoza
certify that a land is alienable and disposable. The only needed to do, which he did, was to relocate on
applicant for land registration must prove that the the ground the boundary lines of the Matchwood Forest
DENR Secretary had approved the land classification Reserve which are nearest the subject property, i.e.,
and released the land of the public domain as alienable from points BFFR-45 to BFFR-46 to BFFR-47-A, and from
and disposable, and that the land subject of the there, determine whether the subject property is on
application for registration falls within the approved the side of the forest reserve or the released area.
area per verification through survey by the PENRO or It would similarly be unnecessary for Engr. De los
CENRO. In addition, the applicant for land registration Santos to conduct table plotting of Lot 1 of GSS-569
must present a copy of the original on the land classification maps. Engr. De los Santos
classificationapproved by the DENR Secretary and already plotted the subject property on the land
certified as a true copy by the legal custodian of the classification maps based on the technical description
official records. These facts must be established to of said property as it stated on OCT No. P-5885. Thus,
prove that the land is alienable and disposable. there can be no doubt that the property Engr. De los
Respondent failed to do so because the certifications Santos plotted on the land classification maps is
presented by respondent do not, by themselves, prove exactly the property awarded and registered in the name
that the land is alienable and disposable. of respondent Roxas. It bears to stress that both
Only Torres, respondent’s Operations Manager, geodetic engineers testified on matters within their
identified thecertifications submitted by respondent. competence and expertise, and other than the baseless
The government officials who issued the certifications doubts of the RTC and the Court of Appeals, there is
were not presented before the trial court to testify no evidence on record to refute said witnesses’
on their contents. The trial court should not have testimonies.
accepted the contents of the certifications as proof In sum, the subject property is within the Matchwood
of the facts stated therein. Even if the certifications Forest Reserve and, therefore, inalienable and not
are presumed duly issued and admissible in evidence, subject to disposition. Respondent Roxas could not have
they have no probative value in establishing that the validly acquired a homestead patent and certificate of
land is alienable and disposable. (Emphasis ours.) title for the same.
Assistant District Forester Dacanay’s Letter dated Although there is no evidence of
July 12, 1965 is a mere correspondence; it is not even fraud by respondent Roxas, there is
a certification. Coupled with the fact that Assistant still reason to cancel OCT No. P-
District Forester Dacanay did not personally testify 5885 and revert the subject property
before the RTC as to the truth of the contents of his to the State.
Letter dated July 12, 1965, said letter carries little We do not find evidence indicating that respondent
evidentiary weight. The Land Group Settlement Survey, Roxas committed fraud when he applied for homestead
GSS-569, prepared by Engr. Javier and approved (for patent over the subject property. It does not appear
the Director of Lands) by Acting Regional Land Director that he knowingly and intentionally misrepresented in
Villapando, also does not constitute incontrovertible his application that the subject property was alienable
evidence that the subject property is alienable and and disposable agricultural land. Nonetheless, we
disposable agricultural land of the public domain. We recognized in Republic of the Phils. v.
pointed out in Republic of the Phils. v. Court of Mangotara58 that there are instances when we granted
Appeals57 that: reversion for reasonsother than fraud:
There is no factual basis for the conclusion of the Reversion is an action where the ultimate relief sought
appellate court that the property in question was no is to revert the land back to the government under the
longer part of the public land when the Government Regalian doctrine. Considering that the land subject

21
Public Land V Govnt Land – Confirmation of Imperfect or incomplete title: Natural Resources Jennica Gyrl
Delfin
of the action originated from a grant by the of the Philippines in accordance with law. The date of
government, its cancellation is a matter between the issuance of the patent, therefore, corresponds to the
grantor and the grantee. In Estate of the Late Jesus date of the issuance of the decree in ordinary
S. Yujuico v. Republic (Yujuico case), reversion was registration cases because the decree finally awards
defined as an action which seeks to restore public land the land applied for registration to the party entitled
fraudulently awarded and disposed of to private to it, and the patent issued by the Director of Lands
individuals or corporations to the mass of public equally and finally grants, awards, and conveys the
domain. It bears to point out, though, that the Court land applied for to the applicant. This, to our mind,
also allowed the resort by the Government to actions is in consonance with the intent and spirit of the
for reversion to cancel titles that were void for homestead laws, i.e. conservation of a family home,
reasons other than fraud, i.e., violation by the and to encourage the settlement, residence and
grantee of a patent of the conditions imposed by law; cultivation and improvement of the lands of the public
and lack of jurisdiction of the Director of Lands to domain. If the title to the land grant in favor of the
grant a patent covering inalienable forest land or homesteader would be subjected to inquiry, contest and
portion of a river, even when such grant was made decision after it has been given by the Government thru
through mere oversight. In Republic v. Guerrero, the the process of proceedings in accordance with the
Court gave a more general statement that the remedy of Public Land Law, there would arise uncertainty,
reversion can be availed of "only in cases of confusion and suspicion on the government’s system of
fraudulent or unlawful inclusion of the land in patents distributing public agricultural lands pursuant to the
or certificates of title." (Emphasis ours, citations "Land for the Landless" policy of the State. (Emphases
omitted.) ours, citations omitted.)
parently, in the case at bar, a mistake or oversight Yet, we emphasize that our statement in the aforequoted
was committed on the part of respondent Roxas, as well case that a certificate of title issued pursuant to a
as the Government, resulting in the grant of a homestead patent becomes indefeasible after one year,
homestead patent over inalienable forest land. Hence, is subject to the proviso that "the land covered by
it can be said that the subject property was unlawfully said certificate is a disposable public land within
covered by Homestead Patent No. 111598 and OCT No. P- the contemplation of the Public Land Law." As we have
5885 in respondent Roxas’s name, whichentitles ruled herein, the subject property is part of the
petitioner Republic to the cancellation of said patent Matchwood Forest Reserve and is inalienable and not
and certificate of title and the reversion of the subject to disposition. Being contrary to the Public
subject property to the public domain. Land Law, Homestead Patent No. 111598 and OCT No. P-
Petitioner Republic is not barred by 5885 issued in respondent Roxas’s name are void; and
prescription and estoppel from the right of petitioner Republic to seek cancellation
seeking the cancellation of of such void patent/title and reversion of the subject
respondent Roxas’s title and property to the State is imprescriptible.
reversion of the subject property. We have addressed the same questions on indefeasibility
It is true that once a homestead patent granted in of title and prescription in Mangotara,61 thus:
accordance with the Public Land Act is registered It is evident from the foregoing jurisprudence that
pursuant to Act 496, otherwise known as The Land despite the lapse of one year from the entry of a
Registration Act, or Presidential Decree No. 1529, decree of registration/certificate of title, the
otherwise known as The Property Registration Decree, State, through the Solicitor General, may still
the certificate of title issued by virtue of said institute an action for reversion when said
patent has the force and effect of a Torrens title decree/certificate was acquired by fraud
issued under said registration laws.59 We expounded in ormisrepresentation. Indefeasibility of a title does
Ybañez v. Intermediate Appellate Court60 that: not attach to titles secured by fraud and
The certificate of title serves as evidence of an misrepresentation. Well-settled is the doctrine that
indefeasible title to the property in favor of the the registration of a patent under the Torrens system
person whose name appears therein. After the expiration does not by itself vest title; it merely confirms the
of the one (1) year period from the issuance of the registrant’s already existing one. Verily,
decree of registration upon which it is based, it registration under the Torrens system is not a mode of
becomes incontrovertible. The settled rule is that a acquiring ownership.1âwphi1
decree of registration and the certificate of title But then again, the Court had several times in the past
issued pursuant thereto may be attacked on the ground recognized the right of the State to avail itself of
of actual fraud within one (1) year from the date of the remedy of reversion in other instances when the
its entry and such an attack must be direct and not by title to the land is void for reasons other than having
a collateral proceeding. The validity of the been secured by fraud or misrepresentation. One such
certificate of title in this regard can be threshed case is SpousesMorandarte v. Court of Appeals, where
out only in an action expressly filed for the purpose. the Bureau of Lands (BOL), by mistake and oversight,
It must be emphasized that a certificate of title granted a patent to the spouses Morandarte which
issued under an administrative proceeding pursuant to included a portion of the Miputak River. The Republic
a homestead patent, as in the instant case, is as instituted an action for reversion 10 years after the
indefeasible as a certificate of title issued under a issuance of an OCT in the name of the spouses
judicial registration proceeding, provided the land Morandarte. The Court ruled:
covered by said certificate is a disposable public land Be that as it may, the mistake or error of the officials
within the contemplation of the Public Land Law. or agents of the BOL in this regard cannot be invoked
There is no specific provision in the Public Land Law against the government with regard to property of the
(C.A. No.141, as amended) or the Land Registration Act public domain. It has been said that the State cannot
(Act 496), now P.D. 1529, fixing the one (1) year be estopped by the omission, mistake or error of its
period within which the public land patent is open to officials or agents.
review on the ground of actual fraud as in Section 38 It is well-recognized that if a person obtains a title
of the Land Registration Act, now Section 32 of P.D. under the Public Land Act which includes, by oversight,
1529, and clothing a public land patent certificate of lands which cannot be registered under the Torrens
title with indefeasibility. Nevertheless, the system, or when the Director of Lands did not have
pertinent pronouncements in the aforecited cases jurisdiction over the same because it is a public
clearly reveal that Section 38 of the Land Registration domain, the grantee does not, by virtue of the said
Act, now Section 32 of P.D. 1529 was applied by certificate of title alone, become the owner of the
implication by this Court to the patent issued by the land or property illegally included. Otherwise stated,
Director of Lands duly approved by the Secretary of property of the public domain is incapable of
Natural Resources, under the signature of the President

22
Public Land V Govnt Land – Confirmation of Imperfect or incomplete title: Natural Resources Jennica Gyrl
Delfin
registration and its inclusion in a title nullifies
that title. B. SALE OF PUBLIC LAND
Another example is the case of Republic of the Phils. C. LEAASE
v. CFI of Lanao del Norte, Br. IV, in which the D. CONFIRMATION OF IMPERFECT OR INCOMPLETE TITLE
homestead patent issued by the State became null and (JUDICIAL LEGISLATION)
void because of the grantee’s violation of the
conditions for the grant. The Court ordered the
reversion even though the land subject of the patent [G.R. No. 120066. September 9, 1999]
was already covered by an OCT and the Republic availed OCTABELA ALBA Vda. De RAZ, Spouses MANUEL and SUSANA
itself of the said remedy more than 11 years after the BRAULIO, RODOLFO, LOURDES and BEATRIZ all surnamed
cause of action accrued, because: ALBA, petitioners, vs. COURT OF APPEALS and JOSE
There is merit in this appeal considering that the LACHICA, respondents.
statute of limitation does not lie against the State. D E C I S I O N
Civil Case No. 1382 of the lower court for reversion YNARES-SANTIAGO, J.:
is a suit brought by the petitioner Republic of the Before us is an appeal by certiorari from a decision
Philippines as a sovereign state and, by the express rendered by the Court of Appeals dated August 18, 1992
provision of Section 118 of Commonwealth Act No. 141, affirming in toto the decision of the Regional Trial
any transfer or alienation of a homestead grant within Court of Kalibo, Aklan, Branch I, in Land Registration
five (5) years from the issuance of the patent is null Case No. K-101, LRC Record No. K. 15104, the
and void and constitute a cause for reversion of the dispositive portion of which reads as follows:
homestead to the State. In Republic vs. Ruiz, 23 SCRA WHEREFORE, judgment is hereby rendered as follows:
348, We held that "the Court below committed no error 1. The parcel of land described in Plan Psu-161277 and
in ordering the reversion to plaintiff of the land the improvements thereon situated in the Poblacion of
grant involved herein, notwithstanding the fact that the Municipality of Banga, Province of Aklan,
the original certificate of title based on the patent Philippines, with an area of 4,845 square meters is
had been cancelled and another certificate issued in brought under the operation of the property
the names of the grantee heirs. Thus, where a grantee registration decree (PD No. 1529) and the title thereto
is found not entitled to hold and possess in fee simple is registered and confirmed in the name of applicant
the land, by reason of his having violated Section 118 Jose Lachica, married to Adela Raz of Kalibo, Aklan,
of the Public Land Law, the Court may properly order Philippines;
its reconveyance to the grantor, although the property 2. A ten (10) meter road width along the national road
has already been brought under the operation of the mentioned in the application be segregated for future
Torrens System. And, this right of the government to road widening program upon payment of just compensation
bring an appropriate action for reconveyance is not to be annotated at the back of the title;
barred by the lapse of time: the Statute of Limitations 3. For lack of merit, the opposition filed by the
does not run against the State." (Italics supplied). spouses Manuel and Susana Braulio, Octabela Alba Vda.
The above ruling was reiterated in Republic vs. Mina, De Raz, Rodolfo Alba, Lourdes Alba and Beatriz Alba
114 SCRA 945. are hereby DISMISSED.
If the Republic is able to establish after trial and SO ORDERED.[1]
hearing of Civil Case No. 6686 that the decrees and The factual antecedents of the case as summed by the
OCTs in Doña Demetria’s name are void for some reason, trial court and adopted by the Court of Appeals are as
then the trial court can still order the reversion of follows:
the parcels of land covered by the same because Applicant Jose Lachica filed this application for title
indefeasibility cannot attach to a void decree or to land on April 28, 1958 with the claim that the land
certificate of title. x x x. (Citations omitted.) applied for was purchased by him and his wife, Adela
Neither can respondent Roxas successfully invoke the Raz from, from one Eulalio Raz. The documents attached
doctrine of estoppel against petitioner Republic. to the application are: technical description,
While it is true that respondent Roxas was granted surveyors certificate, certification by the chief
Homestead Patent No. 111598 and OCT No. P-5885 only deputy assessor of Aklan and the blue print of Psu-
after undergoing appropriate administrative 161277.
proceedings, the Government is not now estopped from The initial hearing was scheduled for October 31, 1958
questioning the validity of said homestead patent and and the certificate of publication in the Official
certificate of title. It is, after all, hornbook law Gazette was issued on September 23, 1958. The
that the principle of estoppel does not operate against certification of posting of the notice of initial
the Government for the act of its agents.62 And while hearing was issued on October 13, 1958.
there may be circumstances when equitable estoppel was The land applied for is residential, situated in the
applied against public authorities, i.e., when the Poblacion of Banga, Aklan, with an area of 4,845 square
Government did not undertake any act to contest the meters, bounded on the northeast by the property of
title for an unreasonable length of time and the lot the Municipality of Banga (Sketch, Exh. F).
was already alienated to innocent buyers for value, The initial hearing was held on October 31, 1958. An
such are not present in this case.63 More importantly, order of general default was issued but those who
we cannot use the equitable principle of estoppel to presented their opposition, namely, Octabela Alba Vda.
defeat the law. Under the Public Land Act and De Raz, Manuel and Susana Braulio, Jose Rago,
Presidential Proclamation No. 678 dated February 5, representing Apolonia Rebeco, the Director of Lands
1941, the subject property is part of the Matchwood and the Municipality of Banga represented by the
Forest Reserve which is inalienable and not subject to Provincial Fiscal, were given thirty (30) days to file
disposition. their written opposition.
WHEREFORE, we GRANT the Petitions and REVERSE and SET Manuel C. Braulio and Susana P. Braulio filed their
ASIDE the Decision dated April 21, 2003 of the Court opposition on October 31, 1958. They opposed the
of Appeals in CA-G.R. CV No. 44926, which, in turn, registration of the southeastern portion of the 240
affirmed the Decision dated February 10, 1994 of the square meters of the land applied for alleging that
Regional Trial Court, Branch 39 of Oriental Mindoro, they are the owners in fee simple and possessors of
in Civil Case No. R-3110. We DECLARE Homestead Patent said portion and all the improvements thereon for not
No. 111598 and OCT No. P-5885 in the name of respondent less than 70 years together with their predecessor-in-
Vicente Roxas null and void and ORDER the cancellation interest deriving their title by purchase from the
of the said patent and certificate of title. We further original owners. They prayed for the Court to declare
ORDER the reversion of the subject property to the them the true and absolute owners of the disputed
public domain as part of the Matchwood Forest Reserve. portion of the same in their names.
SO ORDERED.

23
Public Land V Govnt Land – Confirmation of Imperfect or incomplete title: Natural Resources Jennica Gyrl
Delfin
On October 31, 1958, Octabela Vda. de Raz filed her On March 13, 1974, the Court issued an order appointing
opposition. Engr. Angeles Relor to act as Commissioner and delimit
Jose Rago filed his opposition on November 29, 1958 as the portions claimed by the three sets of oppositors
the duly constituted attorney-in-fact of Apolonia and submit an amended approved plan together with the
Rebeco although no special power of attorney was technical description for each portion.
attached. He opposed the registration of the The Commissioners report and sketch was submitted on
northeastern portion of the land applied for, with an December 4, 1974. The applicant filed his opposition
area of 43.83 square meters. He alleged that his to the Commissioners report on December 12, 1974. The
principal is the owner by right of succession and is Court in its order of December 13, 1974 required the
in the possession of said portion with all its Commissioner to submit an amended report and amended
improvements for more than 80 years together with his sketch.
predecessor-in-interest, continuously, peacefully and The Commissioners corrected report and sketch was
openly under claim of ownership. He prayed that his submitted on February 24, 1975 which the Court approved
principal be declared the true and absolute owner of on February 25, 1975 there being no objection from the
the disputed portion of 43.83 square meters. parties.
On March 22, 1966, the Court issued an Order allowing On March 15, 1977, the Court issued an order whereby
the applicant to hire another surveyor to segregate the testimony of oppositor Octabela Alba Vda. de Raz
the non-controversial portion of the land applied for was stricken off the record for her failure to appear
and to notify the oppositors and their counsels. in the scheduled hearing on March 15, 1977.
On January 12, 1970, a motion to lift the order of Again, in its order dated May 27, 1977 the testimony
general default and to admit the attached opposition of Octabela Alba Vda. de Raz was stricken off record
of Rodolfo Alba, Lourdes Alba and Beatriz Alba, as well because the latter was bedridden and can not possibly
as a motion to admit the attached amended petition of appear for cross-examination.
Octabela Vda. de Raz were filed. The Court in its order Oppositor Octabela Alba Vda. de Raz substituted by her
dated March 21, 1970 admitted said opposition and set heirs filed a formal offer of exhibits on August 24,
aside the order of default. 1988. Applicant filed his comments thereto on August
In their opposition, Rodolfo Alba, Lourdes Alba, 29, 1988. The Court admitted said exhibits and the
represented by their attorney-in-fact, Octabela Alba testimony of their witness on March 1, 1989.
Vda. de Raz, alleged that they are the co-owners of a In this applicaton for title to land filed by applicant
portion of the land applied for with an area of 2,262 Jose Lachica, four oppositions were filed by the
square meters bounded on the north by Januario Masigon, following:
Nicolas Realtor, Agustina Rebeldia and Apolonia 1. Jose Rago, in representation of Apolonia Rebeco;
Rebeco, on the south by Eulalio Raz and on the west by 2. Manuel C. Braulio and Susana Braulio;
the public market of Banga. They claimed to have 3. Rodolfo, Lourdes and Beatriz, all surnamed Alba,
inherited the above-mentioned portion from their late represented by Octabela Alba Vda. de Raz; and
father, Eufrosino M. Alba, who purchased the same from 4. Octabela Alba Vda. de Raz.
Dionisia Regado in 1918. Hence, they have been in In the hearing of October 23, 1970, counsel for
possession continuously, openly and peacefully under oppositor Jose Rago manifested that he would file a
claim of ownership of the above-mentioned portion for motion for withdrawal of opposition and Jose Rago
not less 70 years. They prayed that the disputed himself declared his conformity (Tsn, Oct. 23, 1970,
portion of 2,262 square meters be registered as p. 5). Although no formal motion to withdraw was
their pro-indiviso property. actually filed, oppositor Rago has not presented
In her amended opposition, Octabela Alba Vda. de Raz evidence on his behalf; hence, his opposition must be
opposed the registration of the southeastern portion disregarded.
of the land applied for with an area of 331.44 square As regards oppositor Manuel C. Braulio ans Susana
meters. She claimed to have been in peaceful, Braulio, a deed of sale supposedly executed by Susana
continuous and open possession together with her Braulio and Octabela Alba Vda. de Raz in 1956 was
deceased husband, Eulalio Raz, under claim of ownership identified by Felimon Raz, a witness for the oppositors
of the above-mentioned portion for not less than 70 (Tsn, Sept. 29, 1977, pp. 3 to 4). However, said deed
years, by purchase from its owners. She likewise cannot be found in the records. Even so, the Braulios
opposed the registration of the western portion of the have not presented evidence to show that by the time
land applied for, with an area of 676 square meters, this application was filed, they and their
having purchased the same from its original owners on predecessors-in-interest have been in actual, open,
(sic) her predecessor-in-interest has been open, public, peaceful and continuous possession of the land
peaceful and continuous under claim of ownership for a claimed, in concept of owner, for at least 10 years
period of not less than 70 years. She prayed that the sufficient to acquire title thereto (Arts. 1117, 1118,
portion of 331.44 square meters be registered in her 1134, Civil Code of the Philippines). As such, the
name and that of the heirs of Eulalio Raz, pro opposition of Manuel C. Braulio and Susana Braulio must
indiviso., and the other portion of 676 square meters be dismissed.[2]
be registered solely in her name. On the basis of the testimonial and documentary
On February 25, 1970, the applicant Dr. Jose Lachica evidence presented by the applicant and the oppositor
filed his consolidated opposition and reply to the Raz, the court a quo rendered judgment in favor of the
motion to lift order of default stating that there is applicant as stated at the outset. In dismissing the
no reason to do so under the Rules of Court, and that claim of the remaining oppositors Rodolfo, Lourdes and
the opposition of Rodolfo Alba, Lourdes Alba and Beatriz, all surnamed Alba, represented by Octabela
Beatriz Alba, as well as the amended opposition of Alba Vda. de Raz and Octabela Alba Vda. de Raz herself,
Octabela Alba Vda. de Raz are without merit in law and the trial court in sum noted that said oppositors have
in fact. never offered any explanation as to the non-payment of
On March 21, 1970, the motion to lift the order of realty taxes for the disputed portions of the subject
general default was granted and the opposition of property from 1941 to 1958 while the
Rodolfo Alba, Lourdes Alba and Beatriz Alba, as well respondent/applicant continuously paid taxes under Tax
as the opposition of Octabela Alba Vda. de Raz were Declaration No. 14181 covering said property from 1945-
all admitted. 1958 when the case was filed per certification issued
In the hearing of March 3, 1972, applicant offered for by the Municipal Treasurers Office of Banga.[3] In
admission exhibits A to I and the testimonies of Pedro rendering judgment in favor of respondent/applicant,
Ruiz (April 20, 1971), Jose Rago (Oct. 23, 1970) and the trial court stressed that while it is true that
Dr. Jose Lachica (July 16, 1971; Feb. 10, 1972). The tax receipts and declarations of ownership for tax
Court admitted the same. purposes are not incontrovertible evidence of
ownership, they become strong evidence of ownership

24
Public Land V Govnt Land – Confirmation of Imperfect or incomplete title: Natural Resources Jennica Gyrl
Delfin
acquired by prescription when accompanied by proof of portions of the property, he has sufficiently
actual possession. established that they were notarized documents and were
Dissatisfied, petitioners interposed an appeal to the taken by his mother-in-law sometime in 1956. He
Court of Appeals which affirmed the decision of the reported the loss to the authorities and even filed a
trial court. case of theft. He further exerted efforts and made a
Unfazed, petitioners now come to this Court arguing diligent search of those documents from the notary
that public but in vain. He presented the clerk of the
1. The Civil law provisions on prescription are Municipal Treasurers Office of Banga, who testified
inapplicable. having seen those deeds as they were presented to him
2. The applicable law is Section 48 [a] of the Public by the applicant and which were used as basis for the
Land Law or Act 141, as amended. preparation and issuance of Tax Declaration No. 14181
3. Private respondent has not acquired ownership in in the name of the tax declarant. Tax Declaration No.
fee simple, much less has he met the conditions for 14181 (Exh. H) was presented in Court, proving that
judicial confirmation of imperfect title under Section the land was declared for tax purposes in the name of
48 [a] of Act 141, as amended, except perhaps for a the applicant and his wife. The applicant has been
620 square meter portion of the land applied for paying the realty tax covering the property since 1945
because: and beyond 1958, when the application for registration
3.1. There is absolutely no proof of the alleged sales was filed in court, per certification of the Municipal
made by Raz and Alba. Treasurer of Banga (Exh. 1).
3.2. There is absolutely no reliable proof of the In resume, We find and so hold as did the trial court
alleged theft of the deeds of sale. that Dr. Jose Lachica is the abolute owner in fee
3.3. The identity of the land has not been established. simple of the land described in his application for
3.4. The Court of Appeals misapplied the basic rules its original registration in his name. The land
governing the introduction of secondary evidence. contains an area of 4,845 square meters, more or less,
3.5. The applicant/respondents Tax Declaration No. situated in Banga, Aklan, and
14181 is a doctored tax declaration. Bounded on the NE., along line1-2, by property of
3.6. Applicant/respondents tax declarations have no Apolonia Rimate; on the SE., along line 2-3, by
probative value. National road; on the SW., along line 3-4, by property
3.7. Applicant/respondent has not satisfied the of the Mpl. Government of Banga (Public Market); and
required quantum of evidence in land registration on the NW., along line 4-1, by property of the
cases. Municipal Government of Banga (Public
3.8. Petitioners-oppositors have proven their right Market). Beginning at a point marked 1 on plan, being
over the subject property. N. 45 deg. 02 E., 423.38 m. from B.L.L.M. 1, Mp. of
In rendering judgment in favor of private respondent, Banga, Aklan;
the Court of Appeals reasoned, inter alia, as follows: thence, S. 33 deg. 46 E., 87.66 m. to point 2
On the basis of the testimonial and documentary thence, S. 56 deg. 42 W., 63.81 m. to point 3
evidence presented by the applicant, the trial court thence, N. 37 deg. 22 W., 59.26 m. to point 4
did not err in confirming that the applicant is the thence, N. 33 deg. 42 E., 73.08 m. to the point of
absolute owner in fee simple of the property subject beginning, xxx All points referred to are indicated on
of the application for registration entitling him to the plan and are marked on the ground by P.L.S. Cyl.
register the same in his name under the operation of Conc. Mons. Bearings true date of the survey, January
PD 1529. 25, 1957, and that of the approval, October 3, 1957.
It is of no moment that the applicant failed to produce The applicant has been in public, open, continuous and
the originals of those other deeds/documents of adverse possession of the property since 1940-41 up to
conveyances, for he was able to present sufficient the present to the exclusion of all, and thereby also
substantial secondary evidence, in accordance with the acquired the property by acquisitive prescription, in
requirements of Section 4, Rule 130 of the Revised accordance with Sections 40 and 43 of Act 190,
Rules of Court, now Section 5, same Rule of the Revised otherwise known as the Code of Civil Procedure, having
Rules on Evidence, and the doctrines in point. been in actual and adverse possession under claim of
Thus, Government vs. Martinez, 44 Phil. 817, explained ownership for over ten (10) years, and thus in whatever
that when the original writing is not available for way his occupancy might have commenced or continued
one reason or another which is the best or primary under a claim of title exclusive of any other right
evidence, to prove its contents is the testimony of and adverse to all other claimants, resulted in the
some one who has read or known about it. Republic vs. acquisition of title to the land by acquisitive
Court of Appeals, 73 SCRA 148, laid out the foundation prescription (Vda. de Delima vs. Tio, 32 SCRA 516).
before secondary evidence is introduced, that the due Indeed, to borrow the apt words of the ponente in
execution, delivery and reason for non-production of the Delima case, such proof of ownership of, and the
the original writing must first be produced. Raylago adverse, continuous possession of the applicant since
vs Jarabe, 22 SCRA 1247, ruled that it is not necessary 1940, strongly xxx militate against any judicial
to prove the loss of the original document beyond all cognizance of a matter that could have been withheld
possibility of mistake. A reasonable probability of in its ken, hence, whatever right oppositors may have
its loss is sufficient and this may be shown by had over the property or any portion thereof was
a bonafide (sic) and diligent search, fruitlessly thereby also lost through extinctive prescription in
made, for it in places where it is likely to be favor of the applicant who had been in actual, open,
found. After proving the due execution and delivery of adverse and continuous possession of the land applied
the document, together with the fact that the same has for in the concept of owner for over 10 years when the
been lost or destroyed, its contents may be proved, application for registration was filed in court.[4]
among others, by the recollection of It is a fundamental and settled rule that findings of
witnesses. And Beall vs. Dearing, 7 ala. 126; fact by the trial court and the Court of Appeals are
and Bogardas vs. Trinity Church, 4 Sandf. Ch. (Nn.y.) final, binding or conclusive on the parties and upon
639, are of the view that that where the lost documents this Court,[5] which will not be reviewed[6] or disturbed
are more than thirty (30) years old and would thus on appeal unless these findings are not supported by
prove themselves if produced, secondary evidence of evidence[7] or unless strong and cogent reasons dictate
their contents is admissible without proof of their otherwise.[8]
execution. More explicitly, the findings of fact of the Court of
In the case at bar, petitioner acquired the property Appeals, which are as a general rule deemed conclusive,
in 1940-1941. He presented the Deed (Exh. G) executed may be reviewed by this Court in the following
by the vendor Faustino Martirez. While he failed to instances:
present the other deeds of sale covering the other

25
Public Land V Govnt Land – Confirmation of Imperfect or incomplete title: Natural Resources Jennica Gyrl
Delfin
1.] When the factual findings of the Court of Appeals Capiz. Sin ninguna mejora, de una extension
and the trial court are contradictory;[9] superficial de ochocientos cuarenta metros cuadrados
2.] When the conclusion is a finding grounded entirely (840 mts. cds.) 6 sean cuarenta metros de frente por
on speculation, surmises and conjectures;[10] otros veinte y unmetrode fondo, cuyos linderos por el
3.] When the inference made by the Court of Appeals Norte con propiedad de Eufrosino Alba y con Eulalio
from its findings of fact is manifestly mistaken, Raz; por Este con Eulalio Raz y con la carretera
absurd[11] or impossible; provincial de Kalibo a Banga; por Sur con la misma
4.] Where there is a grave abuse of discretion in the carretera provincial y con terreno del municipio para
appreciation of facts;[12] mercado; y por al Oeste con al terreno del mercado
5.] When the appellate court in making its findings municipal de Banga y con propiedad de Eufrosino Alba y
went beyond the issues of the case, and such findings al terreno tienes sus mojones de cemento en todos sus
are contrary to the submission of both appellant and cuatro cantos de linderia y sin otro limite visible de
appellee; linderia mas que dichos mojones y esta amillarado a mi
6.] When the judgment of the Court of Appeals is nombre en una sola hoja declaratoria de propiedad Tax
premised on a misapprehension of facts;[13] No. 12374 en la Oficina del Tasador Provincial de
7.] When the Court of Appeals manifestly overlooked Capiz, cuyo valor amilarado actual es veinte pesos
certain relevant facts not disputed by the parties (P20.00) xxx
which, if properly considered, would justify a leaves no room for doubt as to its identity, total area
different conclusion;[14] of 840 square meters as well as its dimensions of 40
8.] When the findings of fact are themselves meters in front and 21 meters at the base. How this
conflicting; parcel was further reduced to 620 square meters is
9.] When the findings of fact are conclusions without explained by the fact that the Municipal Government of
citation of specific evidence on which they are based; Banga appropriated 220 square meters thereof for the
and Banga Public Market Road.
10.] When the findings of fact of the Court of Appeals What, however, is seriously contested are the alleged
are premised on the absence of evidence but such purchases of the other two parcels from Eulalio Raz
findings are contradicted by the evidence on record.[15] measuring 300 square meters and from Eufrocino Alba
The primordial issue to be resolved is whether or not measuring 3,725 square meters owing to the questionable
the private respondent/applicant is entitled to the circumstances surrounding their acquisition.
confirmation of his ownership in fee simple for the 4, The records disclose that the subject land was
845 square meter parcel of land he applied for. originally owned by Dionisia Regado under Tax
In sum, both the trial court and the Court of Appeals Declaration No. 802.[19] The records further reveal that
adjudicated and confirmed private Dionisia Regado sold: [1.] 1,850 square meters of the
respondent/applicants title to the land on the basis land to the Municipality of Banga evidenced by a
of the findings that: 1.] the private Spanish document denominated as a deed of sale dated
respondent/applicant purchased the land from Faustino April 29, 1914;[20] [2.] 1,320 square meters to Eulalio
Martirez; 2.] the subject land is covered by Tax Raz evidenced by a document entitled Escritura de Venta
Declaration No. 14181; 3.] the private Absoluta dated September 6, 1918,[21] and [3.] 2,938
respondent/applicant has paid the realty taxes on the square meters to Eufrocino Alba evidenced by a deed of
land from 1945 up to the filing of his application in conveyance dated September 6, 1918 written in
1958; 4.] the private respondent/applicant has been in Spanish.[22]
actual, open and continuous possession of the subject Faustino Martirez acquired a portion of 840 square
land in the concept of owner since 1945, and 5.] the meters from Eulalio Raz on January 15, 1933.[23] Raz
private respondent/applicant has acquired the land by retained 480 square meters, however, he and his wife
prescription. Octabela Alba conveyed a 240 square meter portion
As stated earlier, a review of the findings of fact of thereof to Susana Braulio on November 5,
the Court of Appeals is not a function that this Court 1956.[24] Subsequently on May 29, 1969, the heirs of
normally undertakes[16] unless the appellate courts Eufrocino Alba sold a 676 square meter portion of the
findings are palpably unsupported by the evidence on parcel purchased by Eufrocino to Octabela Alba Vda. de
record or unless the judgment itself is based on a Raz.[25] The deed of conveyance was duly registered with
misapprehension of facts.[17] A thorough review of the the Registry of Deeds of Aklan pursuant to Act No. 334
record convinces this Court that the general rule with on June 17, 1969[26] and is covered by Tax Declaration
regard to the conclusiveness of the trial courts and No. 332 in the name of Eulalio Raz, her husband.[27]
appellate tribunals factual findings should not be Other than the foregoing transactions involving the
applied because there are material circumstances subject land which are borne out by the documentary
which, when properly considered, would have altered evidence on record, private respondent/applicant did
the result of the case. not produce the alleged deeds of conveyances evidencing
First, a circumspect scrutiny of the evidence extant the purported transfers made by Eulalio Raz and
on record reveals that with the exception of 620 square Eufrocino Alba in his favor. Instead he relied chiefly
meters, there has been no satisfactory showing of how on secondary evidence to prove the existence thereof
private respondent/applicant acquired the remainder of which was sustained by both the trial and the appellate
the subject land. courts. Such reliance on secondary evidence vis--
As can be gathered from the discussion of the appellate vis the peculiar facts prevailing in this case rests
court, as well as the arguments proffered by private on infirm legal bases much more so in the face of the
respondent, he acquired the land in question from three overwhelming documentary evidence of petitioners
(3) sources, namely: a.] A Deed of Sale dated August arrayed against it because
13, 1941 allegedly executed by Faustino Martirez . . . [a] contract of sale of realty cannot be proven
covering 840 square meters; b.] 300 square meters by means of witnesses, but must necessarily be
allegedly purchased from private respondents father- evidenced by a written instrument, duly subscribed by
in-law Eulalio Raz, and c.] 3,725 square meters private the party charged, or by his agent, or by secondary
respondent allegedly bought in 1940 from Eufrocino evidence of their contents. No other evidence,
Alba. therefore, can be received except the documentary
The sale involving the first parcel of land covering evidence referred to, in so far as regards such
840 square meters, was not questioned by petitioners contracts, and these are valueless as evidence unless
as its technical description delineated in they are drawn up in writing in the manner
the Escritura De Venta Absoluta dated August 13, aforesaid.[28]
1941,[18] to wit: An applicant for registration of land, if he relies on
Un terreno solar residencia antes palayero regado, a document evidencing his title thereto, must prove
actuado en el casco central del municipio de Banga, not only the genuineness of his title but

26
Public Land V Govnt Land – Confirmation of Imperfect or incomplete title: Natural Resources Jennica Gyrl
Delfin
the identity of the land therein referred to. The who claims that he has better right to real property
document in such a case is either a basis of his claim must prove not only his ownership of the same but also
for registration or not at all. If , as in this case, must satisfactorily prove the identity thereof.[35]
he only claims a portion of what is included in his Third, both trial and appellate courts placed undue
title, he must clearly prove that the property sought reliance on Tax Declaration No. 14181 considering that
to be registered is included in that title.[29] there is no satisfactory explanation of how the area
Second, there are glaring variances in the identities of land covered by Tax Declaration No. 14181
and technical descriptions of the land applied for by geometrically ballooned from a modest 620 square meter
private respondent/applicant and the land he lot to a huge parcel measuring 4, 845 square meters.
purportedly purchased from Eufrocino Alba. As pointed out by petitioners, Tax Declaration No.
Private respondent/applicant alleged that he purchased 14181 was preceded by 1954 Tax Declaration No. 13578
the remainder of the subject land measuring 3,725 in the name of private respondent/applicant and his
square meters from Eufrocino Alba sometime in 1940 spouse which shows that the land declared therein for
averring that this parcel is listed as Item No. 5 of taxation purposes covers an area of 620 square
his Exhibit I which is denominated as an Inventory And meters. Tax Declaration No. 13578 was preceded by 1953
Appraisal Of The Properties Of The Spouses Adela Raz Tax Declaration No. 13040 in the name of Adela Raz,
De Lachica (Deceased) and Dr. Jose Lachica. Item No. private respondents wife. The land declared for
5[30] of the said inventory described the parcel of land taxation purposes therein also has an area of 620
mentioned therein as follows: square meters. Tax Declaration No. 134040 was preceded
5. Una parcela de terreno cocal secano, amillarado en by 1947 Tax Declaration No. 6528 in the name of private
nombre de Eufrocino Alba bajo el Tax No. 12792 por respondents wife, Adela Raz.The land declared therein
valor de P390.00, situado en el municipio de Banga, for taxation purposes likewise measures 620 square
Capiz, que linda el Norte con Lorenzo Retiro, y meters.
Silverio Relis; al Este con la carretera provincial It appears that the quantum leap from 620 square meters
Banga-Libacao; al sur con Bienvenido M. Alba y al Oeste in 1947 to 4,845 square meters in 1956 came about on
con Cirilo rala y Adela Raz; con una extension account of an affidavit dated November 17, 1956 wherein
aproximada de una (1) hectarea (20) areas y (35) private respondent/applicant requested[36] the
centiareas poco mas o menos. (Note: Said property was Municipal Assessor of Banga to issue a revised tax
purchased by the spouses Jose Lachica and Adela Raz declaration covering 4,845 square meters on the bare
Lachica from Eufrocino M. Alba in the amount of P500.00 claim that the area has been decreased to only 620
as evidenced by a Escritura de Compraventa executed on square meters. The timing of the revision and its
November 25, 1940, at Himamaylan, Negros Occidental proximity to the date of filing of the application can
and notarized by Atty. Conrado Gensiano, as Reg. Not. not but engender serious doubts on the application more
122, Pag. 67, Libro VIII, Serie 1940). so considering that prior thereto realty tax payments
On the other hand, the land applied for is described covering the period 1945 to 1956 covered an area
technically per Psu 161277 as measuring 620 square meters and private
A parcel of land (as shown on Plan Psu-161277), respondent/applicant is banking on said payments to
situated in Poblacion, Municipality of Banga, Province claim possession and ownership over the same period
of Aklan. Bounded on the NE., along line 1-2, by for an infinitely larger area of 4,845 square meters.
property of Apolonia Rimate; on the SE., along line 2- A tax declaration, by itself, is not conclusive
3, by National Road; on the SW., along line 3-4, by evidence of ownership.[37] Tax declarations for a
property of the Mpl. Government of Banga (Public certain number of years, although constituting proof
Market); and on the NW., along line 4-1, by property of claim of title to land,[38] is not incontrovertible
of the Municipal Government of Banga (Public evidence of ownership unless they are supported by
Market). Beginning at a point marked 1 on plan, being other effective proof.[39] It was, thus, held in one
N. 45 deg. 02 E., 423.38 m. from B.L.L.M. 1, Mp. of case[40] that where realty taxes covering thirty-one
Banga, Aklan; (31) years were paid only a few months prior to the
thence S. 33 deg. 46 E. 87.66 m. to point 2 filing of an application, such payment does not
thence S. 56 deg. 42 W., 63.81 m. to point 3 constitute sufficient proof that the applicant had
thence N. 37 deg. 22 W., 59.26 m. to point 4 a bona fide claim of ownership prior to the filing of
thence N. 33 deg. 42 E., 73.08 m. to the point of the application. Still in another case,[41] the claim
beginning, containing an area of FOUR THOUSAND EIGHT that the applicant had been in continuous and
HUNDRED AND FORTY FIVE (4,845) SQUARE METERS. All uninterrupted possession of the disputed land was not
points referred to are indicated on the plan and are given credence because it was negated by the fact that
marked on the ground by P.L.S. Cyl. Conc. Mons. he declared the land for taxation purposes in October
Bearings true date of survey, January 25, 1957, and 1959 when he filed his application for registration
that of the approval, October 3, 1957.[31] although he could have done so in 1937 when he
It will be readily noted vis--vis the foregoing that: allegedly purchased the land. A belated declaration
a.] the land applied for is covered by Tax Declaration is, furthermore, indicative that the applicant had no
No. 14181 while the parcel allegedly purchased from real claim of ownership over the subject land prior to
Eufrocino Alba is covered by Tax Declaration No. 15792; the declaration[42] and where there are serious
b.] the land applied for is palayero whereas the land discrepancies in the tax declarations as in this case,
allegedly acquired from Eufrocino Alba is cocal registration must be denied.[43] If at all, the
secano. Palay is unhusked rice,[32] thus, the foregoing facts only serves to underscore private
term palayero refers to land devoted to the planting respondent/applicants crafty attempt to cloak with
of rice; cocal, on the other hand, means coconut tree judicial color his underhanded scheme to seize the
plantation[33] while secano denotes unwatered land or a adjoining parcels of land and to enrich himself at the
dry sand bank;[34] c.] the land applied for has an area expense of its rightful owners.
of 4,845 square meters whereas the land supposedly sold Fourth, the lower courts reliance on prescription is
by Eufrocino Alba measures 12,035 square meters; d.] not well-taken given the peculiar facts prevailing in
the land applied for is bounded on the NE by the Banga this case.
Public Market, on the SE by Apolonia Rimate, on the SW The law in force at the time an action accrues is what
by the Banga-Kalibo National Road; and on the NW by governs the proceeding consistent with the fundamental
the Banga Public Market whereas the land allegedly dictum that laws shall have no retroactive effect,
obtained from Eufrocino Alba is bounded on the N by unless the contrary is proved.[44] Basic is the rule
Ernesto Retino and Silverio Relis, on the E by the that no statute, decree, ordinance, rule, regulation
Banga-Libacao Carretera Provincial, on the S by or policy shall be given retrospective effect unless
Bienvenido Alba and on the W by Cirilo Rala and Adela explicitly stated so.[45] Along the same vein, a courts
Raz. It needs be stressed in this regard that a person jurisdiction depends on the law existing at the time

27
Public Land V Govnt Land – Confirmation of Imperfect or incomplete title: Natural Resources Jennica Gyrl
Delfin
an action is filed[46]and a law continues to be in force claimed to be owned by the applicant in fee simple
with regard to all rights which accrued prior to the which refer to:
amendment thereof.[47] 1.] Lands acquired by various types of titles from the
In this case, the controlling statute when the private government during the Spanish Regime by way of grants
respondent/applicant filed his application for by the Spanish crown namely the: a.] Titulo real or
registration on April 28, 1958 is Section 48 of royal grant; b.] Concession especial or special grant;
Commonwealth Act 141, as amended by RA Nos. 1942 and c.] Composicion con el estado title or adjustment
6236,[48] which states that: title; d.] Titulo de compra or title by purchase and;
SEC. 48. The following-described citizens of the e.] Informacion posesoria or possessory information
Philippines, occupying lands of the public domain or title, which could become a Titulo gratuito or a
claiming to own any such lands or an interest therein, gratuitous title;[56]
but whose titles have not been perfected or completed, 2.] Lands that are claimed to be owned by accession,
may apply to the Court of First Instance of the i.e. accretion, avulsion, formation of islands,
province where the land is located for confirmation of abandoned river beds, as provided for in Articles 457,
their claims and the issuance of a certificate of title 461 and 464 of the Civil Code; and
therefor, under the Land Registration Act, to wit: 3.] Lands which have been acquired in any other manner
(a) Those who prior to the transfer of sovereignty from provided by law.
Spain to the United States have applied for the Suffice it to state that the land sought to be
purchase, composition or other form of grant of lands registered by private respondent hardly falls under
of the public domain under the laws and royal decrees any of the latter classifications of land referred to
then in force and have instituted and prosecuted the by Act No. 496, as amended. Given the foregoing facts,
proceedings in connection therewith, but have with or prescription in the manner invoked by both courts can
without default upon their part, or for any other not be pleaded to bolster private
cause, not received title therefor, if such applicants respondent/applicants claim because
or grantees and their heirs have occupied and . . . [N]o public land can be acquired by private
cultivated said lands continuously since the filing of persons without any grant, express or implied from the
their applications.[49] government; it is indispensable that there be a showing
(b) Those who by themselves or through their of title from the state . . . .[57]
predecessors in interest have been in open, continuous, x x x x x x x x x
exclusive and notorious possession and occupation of Indeed, the possession of public agricultural land,
agricultural lands of the public domain under a bona however, long the period may have extended, never
fide claim of ownership, for at least thirty years confers title thereto upon the possessor.[58] The
immediately preceding the filing of the application reason, to reiterate our ruling, is because the statute
for confirmation of title except when prevented by war of limitations with regard to public agricultural land
or force majeure. These shall be conclusively presumed does not operate against the State, unless the occupant
to have performed all the conditions essential to a can prove possession and occupation of the same under
Government grant and shall be entitled to a certificate claim of ownership for the required number of years to
of title under the provisions of this chapter.[50] constitute a grant from the State.[59]
(c) Members of the national cultural minorities who by Fifth, even assuming ex gratia argumenti that
themselves or through their predecessors-in-interest prescription can be applied in the manner invoked by
have been in open, continuous, exclusive and notorious the trial court and the appellate court, it must be
possession and occupation of lands of the public domain pointed out that
suitable to agriculture, whether disposable or not, . . . [W]hile Art. 1134 of the Civil Code provides that
under a bona fide claim of ownership for at least 30 (o)wnership and other real rights over immovable
years shall be entitled to the rights granted in property are acquired by ordinary prescription through
subsection (b) hereof.[51] possession of ten years, this provision of law must be
A circumspect scrutiny of the assailed Decision readily read in conjunction with Art. 1117 of the same
shows that in affirming the ruling of the trial court, Code. This article states that xxx (o)rdinary
the Court of Appeals relied on the provisions of acquisitive prescription of things requires possession
Section 19 of Act 496[52] in relation to the Civil Codes in good faith and with just title for the time fixed
provisions on prescription on the assumption that the by law. Hence, a prescriptive title to real estate is
subject land is private land. Therein lies the flaw in not acquired by mere possession thereof under claim of
the appellate courts postulate. The application for ownership for a period of ten years unless such
registration of private respondent is for the judicial possession was acquired con justo titulo y buena
confirmation of an imperfect title considering that fe(with color of title and good faith).[60] The good
the land is presumed under the Regalian Doctrine to be faith of the possessor consists in the reasonable
part of the public domain. belief that the person from whom he received the thing
Public lands are broadly classified into 1.] Alienable was the owner thereof, and could transmit his
or disposable lands; and, 2.] Inalienable or non- ownership.[61] For purposes of prescription, there is
disposable public lands. Non-disposable public lands just title when the adverse claimant came into
or those not susceptible of private appropriation possession of the property through one of the
include a.] Timber lands; and, b.] Mineral recognized modes of acquisition of ownership or other
lands.[53] For purposes of administration and real rights but the grantor was not the owner or could
disposition, the lands of the public domain classified not transmit any right.[62]
as disposable or alienable are further sub-classified It can not be said that private respondents possession
into a.] Agricultural; b.] Residential, commercial, was con justo titulo y buena fe. On the contrary,
industrial or for similar productive purposes; c.] private respondent/applicants act of appropriating for
Educational, charitable or other similar purposes, and himself the entire area of 4,845 square meters to the
d.] Reservations for town sites and for public and exclusion of petitioners who have been occupying
quasi-public purposes.[54] portions of the disputed land constituted acts of
From the foregoing classifications, public deprivation of the latters rights which is tantamount
agricultural land may be defined as those alienable to bad faith. Indeed this Court has ruled that the
portions of the public domain which are neither timber . . . [c]oncealment and misrepresentation in the
nor mineral lands. Thus the term includes residential, application that no other persons had any claim or
commercial and industrial lands for the reason that interest in the said land, constitute specific
these lands are neither timber nor mineral lands.[55] allegations of extrinsic fraud supported by competent
On the other hand, Section 19 of Act No. 496, as proof.Failure and intentional omission of the
amended, permits the registration of private lands applicants to disclose the fact of actual physical
possession by another person constitutes an allegation

28
Public Land V Govnt Land – Confirmation of Imperfect or incomplete title: Natural Resources Jennica Gyrl
Delfin
of actual fraud.[63] Likewise, it is fraud to knowingly his claim of ownership over the entire 4, 845 square
omit or conceal a fact, upon which benefit is obtained meter area. He has not adduced evidence to show how
to the prejudice of a third person.[64] and when he was able to acquire, with the exception of
Suffice it to state in this regard that to allow 840 square meters further reduced to 620 square meters
private respondent/applicant to benefit from his own on account of 220 square meters appropriated for the
wrong would run counter to the maxim ex dolo malo non market road, the bigger area of 3,755 square meters
oritur actio - no man can be allowed to found a claim from anybody let alone the ancestral owner, Dionisia
upon his own wrongdoing.[65] Regado.
It need not be overemphasized that extraordinary His claim is anchored mainly on Revised Tax Declaration
acquisitive prescription can not similarly vest No. 14181 which he was able to procure from the
ownership over the property upon private Municipal Assessor of Banga in 1956 on the basis of a
respondent/applicant because Article 1137 of the Civil self-serving affidavit which proffered the lame excuse
Code states in no uncertain terms that that there was error in the statement of the area of
ART. 1137. Ownership and other real rights over the land which he claimed to be 4,845 square meters
immovables also prescribe through uninterrupted instead of 620 square meters which was the area
adverse possession thereof for thirty years, without reflected in earlier tax declarations namely, 1954 Tax
need of good faith. Declaration No. 13578; 1953 Tax Declaration No. 13043;
Needless to state, private respondent/applicants and 1947 Tax Declaration No. 6528.
possession of thirteen (13) years falls way below the Be that as it may, the Court has reservations on the
thirty-year requirement mandated by Article 1137. propriety of adjudicating to petitioners the contested
Sixth, petitioners/oppositors have, in stark contrast portions of the subject land, in view of their failure
to the secondary proof of private respondent, adduced to present the technical descriptions of these
overwhelming evidence to prove their ownership of the areas. Furthermore, there is no sufficient evidence
portions they claim in the subject land.The evidence showing that petitioners have been in open, adverse,
on record clearly points to the fact that private exclusive, peaceful and continuous possession thereof
respondent/applicants right, if at all, is confined to in the concept of owner, considering that the testimony
only 620 square meters or what has been left of the of Octabela Alba vda. De Raz was stricken off the
840 square meters he purchased from Faustino Martirez record.
after 220 square meters thereof were appropriated by WHEREFORE, based on foregoing premises, the Decision
the Municipality of Banga for the Public Market of the Regional Trial Court of Kalibo, Aklan, Branch 1
Road.[66] dated August 18, 1992 in Land Registration Case No. K-
The records further bear out that the original owner 101, LRC Record No. K-15104 is hereby MODIFIED as
of the whole area was one Dionisia Regado who executed follows:
three (3) deeds of sale covering certain portions of 1.] The 620 square meter portion on which private
the disputed lands, namely: 1.] the Deed of Sale respondent Jose N. Lachicas house is situated, clearly
dated April 29, 1914 covering 1,850 square meters delineating its metes and bounds, is hereby ORDERED
executed in favor of the Municipality of Banga;[67] 2.] segregated from the parcel of land described in Psu-
the Deed of Sale dated July 10, 1915 covering 1,320 161277 situated in the Poblacion of the Municipality
square meters executed in favor of Eulalio Raz;[68] and, of Banga, Province of Aklan, Philippines with an area
3.] the Deed of Sale dated September 6, 1918 covering of 4,484 square meters, to be registered and confirmed
the balance with an area of 2,938 square meters in in the name of private respondent;
favor of Eufrocino Alba.[69] 2.] A ten (10) meter road width along the National road
Faustino Martirez acquired only an 840 square meter mentioned in the application be segregated for future
portion of the land by purchase from Eulalio Raz on road widening programs upon the payment of just
January 15, 1933 as confirmed in paragraph 2 of compensation to be annotated at the back of the title.
the Escritura De Venta Absoluta executed by him on 3.] Insofar as the ownership of the remainder of the
August 13, 1941.[70] After selling 840 square meters to subject land is concerned, the case is hereby REMANDED
Faustino Martirez, Eulalio Raz retained 480 square to the court of origin for the reception of further
meters but on November 5, 1956 Eulalio Raz and his wife evidence for the petitioners to establish the other
Octabela Alba conveyed 240 square meters to Susana requisites for the confirmation of title and
Braulio[71] leaving a balance of 240 square meters which registration in their names of the areas they
remained undisposed. respectively claim.
On May 29, 1969, Virginia Alba, Inocentes Alba and SO ORDERED.
Estrella Alba, children of the deceased Eufrocino Alba,
sold a 676 square meter portion of the 2,938 square [G.R. No. 95608. January 21, 1997]
meter lot purchased by their father from Dionisia SPOUSES IGNACIO PALOMO and TRINIDAD PASCUAL, and CARMEN
Regado to petitioner/oppositor Octabela alba Vda. De PALOMO VDA. DE BUENAVENTURA, petitioners, vs. THE
Raz.[72] This Deed was duly registered with the Registry HONORABLE COURT OF APPEALS, THE REPUBLIC OF THE
of Deeds of Aklan in accordance with Act No. 3344 on PHILIPPINES, FAUSTINO J. PERFECTO, RAFFY SANTILLAN,
June 17, 1969.[73] The land is covered by Tax BOY ARIADO, LORENZO BROCALES, SALVADOR DOE, and other
Declaration No. 332 in the name of Octabela Alba Vda. DOES, respondents.
De Razs husband.[74] D E C I S I O N
Petitioner/oppositor Octabela Alba Vda. De Razs ROMERO, J.:
ownership of the remaining 240 square meter portion The issue in the case at bar pertains to ownership of
which she and her husband Eulalio Raz bought from 15 parcels of land in Tiwi, Albay which form part of
Dionisia Regado[75] and the 676 square meter portion the "Tiwi Hot Spring National Park." The facts of the
which they bought from the heirs of Eufrocino case are as follows.
Alba[76] is fully substantiated by documentary On June 13, 1913, then Governor General of the
proof.[77] Rodolfo Alba, Lourdes Alba and Beatriz Albas Philippine Islands, William Cameron Forbes issued
ownership of a portion measuring 1,335 square Executive Order No. 40 which reserved for provincial
meters[78] and another portion measuring 2,262 square park purposes some 440,530 square meters of land
meters[79] is likewise backed by documentary evidence. situated in Barrio Naga, Municipality of Tiwi, Province
Susana Braulios ownership of a 240 square meter of Albay pursuant to the provisions of Act 648 of the
portion[80] which she acquired from Octabela Alba Vda. Philippine Commission.[1]
De Raz on November 11, 1956[81] is also documented, her Subsequently, the then Court of First Instance of
predecessor-in-interest having acquired the same from Albay, 15th Judicial District, United States of
Dionisia Regado on September 6, 1918.[82] America, ordered the registration of 15 parcels of land
The foregoing only serves to underscore the paucity of covered by Executive Order No. 40 in the name of Diego
the proof of private respondent/applicant to support Palomo on December 9, 1916; [2] December 28, 1916;[3] and

29
Public Land V Govnt Land – Confirmation of Imperfect or incomplete title: Natural Resources Jennica Gyrl
Delfin
January 17, 1917.[4] Diego Palomo donated these parcels (3) Declaring Lot Nos. 1, 2, 3, 4, 5, 6, 7, 8, 9, 10,
of land consisting of 74,872 square meters which were 11 and 12, Plan II-9299 and Lots 1, 21,[11] 3 and 4 of
allegedly covered by Original Certificates of Title Plan II-9205 as part of the Tiwi Hot Spring National
Nos. 513, 169, 176 and 173[5] to his heirs, herein Park;
petitioners, Ignacio and Carmen Palomo two months (4) and Finally, the Register of Deeds of Albay is
before his death in April 1937.[6] hereby ordered to cancel the alleged Original
Claiming that the aforesaid original certificates of Certificates of Titles Nos. 513, 169, 173 and 176,
title were lost during the Japanese occupation, Ignacio Transfer Certificates of Title Nos. T-3911, T-3912, T-
Palomo filed a petition for reconstitution with the 3913 and T-3914.
Court of First Instance of Albay on May 30, 1950.[7] The Costs against the defendants.
Register of Deeds of Albay issued Transfer Certificates So Ordered."[12]
of Title Nos. 3911, 3912, 3913 and 3914 sometime in The court a quo in ruling for the Republic found no
October 1953.[8] sufficient proof that the Palomos have established
On July 10, 1954 President Ramon Magsaysay issued property rights over the parcels of land in question
Proclamation No. 47 converting the area embraced by before the Treaty of Paris which ended the Spanish-
Executive Order No. 40 into the "Tiwi Hot Spring American War at the end of the century. The court
National Park," under the control, management, further stated that assuming that the decrees of the
protection and administration of the defunct Court of First Instance of Albay were really issued,
Commission of Parks and Wildlife, now a division of the Palomos obtained no right at all over the
the Bureau of Forest Development. The area was never Properties because these were issued only when
released as alienable and disposable portion of the Executive Order No. 40 was already in force. At this
public domain and, therefore, is neither susceptible point, we take note that although the Geodetic Engineer
to disposition under the provisions of the Public Land of the Bureau of Lands appointed as one of the
Law (CA 141) nor registrable under the Land Commissioners in the relocation survey of the
Registration Act (Act No. 496). properties stated in his reamended report that of the
The Palomos, however, continued in possession of the 3,384 square meters covered by Lot 2, Plan II-9205,
property, paid real estate taxes thereon[9] and only 1,976 square meters fall within the reservation
introduced improvements by planting rice, area,[13] the RTC ordered TCT 3913 covering the entire
bananas, pandan and coconuts. On April 8, 1971, Lot 21 (sic) Plan II-9205 cancelled.
petitioner Carmen vda. de Buenaventura and spouses The petitioners appealed to the Court of Appeals which
Ignacio Palomo and Trinidad Pascual mortgaged the affirmed in toto the findings of the lower Court;
parcels of land covered by TCT 3911, 3912, 3913 and hence this petition raising the following issues:
3914 to guarantee a loan of P200,000 from the Bank of 1. The respondent Court of Appeals committed grave
the Philippine Islands. abuse of discretion in affirming in toto the decision
In May 7, 1974 petitioner Carmen vda. de Buenaventura of the lower court.
and spouses Ignacio Palomo and Trinidad Pascual filed 2. The declaration of nullity of the original
Civil Case No. T-143 before the then Court of First certificates of title and subsequent transfer
Instance of Albay for Injunction with damages against certificates of titles of the petitioners over the
private respondents Faustino J. Perfecto, Raffy properties in question is contrary to law and
Santillan, Boy Ariado, Lorenzo Brocales, Salvador Doe jurisprudence on the matter.
and other Does who are all employees of the Bureau of 3. The forfeiture of all improvements introduced by
Forest Development who entered the land covered by TCT the petitioners in the premises in favor of the
No. 3913 and/or TCT 3914 and cut down bamboos thereat, government is against our existing law and
totally leveling no less than 4 groves worth not less jurisprudence.
than P2,000.00. The issues raised essentially boil down to whether or
On October 11, 1974, the Republic of the Philippines not the alleged original certificate of titles issued
filed Civil Case No. T-176 for annulment and pursuant to the order of the Court of First Instance
cancellation of Certificates of Title involving the 15 in 1916-1917 and the subsequent TCTs issued in 1953
parcels of land registered in the name of the pursuant to the petition for reconstitution are valid.
petitioners and subject of Civil Case T-143. Impleaded Petitioners contend that the Treaty of Paris which
with the petitioners as defendants were the Bank of ended the Spanish-American War at the end of the 19th
the Philippine Islands, Legazpi Branch and the Register century recognized the property rights of Spanish and
of Deeds of Albay. Filipino citizens and the American government had no
The case against the Bank of Philippine Islands was inherent power to confiscate properties of private
dismissed because the loan of P200,000 with the Bank citizens and declare them part of any kind of
was already paid and the mortgage in its favor government reservation. They allege that their
cancelled. predecessors in interest have been in open, adverse
A joint trial of Civil Case T-143 and T-176 was and continuous possession of the subject lands for 20-
conducted upon agreement of the parties and on July 50 years prior to their registration in 1916-1917.
31, 1986, the trial court rendered the following Hence, the reservation of the lands for provincial
decision: purposes in 1913 by then Governor-general Forbes was
"WHEREFORE, premises considered, judgment is hereby tantamount to deprivation of private property without
rendered: due process of law.
IN CIVIL CASE No. T-143, in favor of the defendants In support of their claim, the petitioners presented
and against the plaintiffs, dismissing the complaint copies of a number of decisions of the Court of First
for injunction and damages, as it is hereby DISMISSED. Instance of Albay, 15th Judicial District of the United
Costs against the plaintiffs. States of America which state that the predecessors in
In CIVIL CASE No. T-176, in favor of the plaintiffs interest of the petitioners' father Diego Palomo, were
and against the defendants: in continuous, open and adverse possession of the lands
(1) Declaring null and void and no force and effect from 20 to 50 years at the time of their registration
the Order dated September 14, 1953, as well as the in 1916.
Original Certificate of Titles Nos. 153,[10] 169, 173 We are not convinced.
and 176 and Transfer Certificates of Titles Nos. 3911, The Philippines passed to the Spanish Crown by
T-3912, T-3913, and T-3914, all of the Register of discovery and conquest in the 16th century. Before the
Deeds of Albay and all transactions based on said Treaty of Paris in April 11, 1899, our lands, whether
titles. agricultural, mineral or forest were under the
(2) Forfeiting in favor of the plaintiff Government exclusive patrimony and dominion of the Spanish Crown.
any and all improvements on the lands in question that Hence, private ownership of land could only be acquired
are found therein and introduced by the defendants; through royal concessions which were documented in

30
Public Land V Govnt Land – Confirmation of Imperfect or incomplete title: Natural Resources Jennica Gyrl
Delfin
various forms, such as (1) Titulo Real or Royal Grant," Having disposed of the issue of ownership, we now come
(2) Concession Especial or Special Grant, to the matter regarding the forfeiture of improvements
(3) Titulo de Compra or Title by Purchase and introduced on the subject lands. It bears emphasis that
(4) Informacion Posesoria or Possessory Information Executive Order No. 40 was already in force at the time
title obtained under the Spanish Mortgage Law or under the lands in question were surveyed for Diego Palomo.
the Royal Decree of January 26, 1889. Petitioners also apparently knew that the subject lands
Unfortunately, no proof was presented that the were covered under the reservation when they filed a
petitioners' predecessors in interest derived title petition for reconstitution of the lost original
from an old Spanish grant. Petitioners placed much certificates of title inasmuch as the blueprint of
reliance upon the declarations in Expediente No. 5, Survey Work Order Number 21781 of Plan II-9299 approved
G.L.R.O. Record Decision No. 9820, dated January 17, by the Chief of the Land Registration Office Enrique
1917; Expediente No. 6, G.L.R.O. Record No. 9821, dated Altavas in 1953 as a true and correct copy of the
December 28, 1916; Expediente No. 7, G.L.R.O. Record Original Plan No. II-9299 filed in the Bureau of Lands
No. 9822, dated December 9, 1916; Expediente No. 8, dated September 11, 1948[19] contains the following
G.L.R.O. Record No. 9823, dated December 28, 1916 and note, "in conflict with provincial reservation."[20] In
Expediente No. 10, G.L.R.O. Record No. 9868, dated any case, petitioners are presumed to know the law and
December 9, 1916 of the Court of First Instance of the failure of the government to oppose the
Albay, 15th Judicial District of the United States of registration of the lands in question is no
America presided by Judge Isidro Paredes that their justification for the petitioners to plead good faith
predecessors in interest were in open, adverse and in introducing improvements on the lots.
continuous possession of the subject lands for 20-50 Finally, since 1,976 square meters of the 3,384 square
years.[14] The aforesaid "decisions" of the Court of meters covered by TCT 3913 fall within the reservation,
First Instance, however, were not signed by the judge TCT 3913 should be annulled only with respect to the
but were merely certified copies of notification to aforesaid area. Inasmuch as the bamboo groves leveled
Diego Palomo bearing the signature of the clerk of in TCT 3913 and subject of Civil Case T-143,[21] were
court. within the perimeter of the national park,[22] no
Moreover, despite claims by the petitioners that their pronouncement as to damages is in order.
predecessors in interest were in open , adverse and WHEREFORE, the decision of the Court of Appeals is
continuous possession of the lands for 20 to 50 years hereby AFFIRMED with the modification that TCT 3913 be
prior to their registration in 1916-1917, the lands annulled with respect to the 1,976 square meter area
were surveyed only in December 1913, the very same year falling within the reservation zone.
they were acquired by Diego Palomo. Curiously, in SO ORDERED.
February 1913 or 10 months before the lands were
surveyed for Diego Palomo, the government had already [G.R. No. 107427. January 25, 2000]
surveyed the area in preparation for its reservation JAMES R. BRACEWELL, petitioner, vs. HONORABLE COURT OF
for provincial park purposes. If the petitioners' APPEALS and REPUBLIC OF THE
predecessors in interest were indeed in possession of PHILIPPINES, respondents. ULANDU
the lands for a number of years prior to their D E C I S I O N
registration in 1916-1917, they would have undoubtedly YNARES-SANTIAGO, J.:
known about the inclusion of these properties in the Before us is a petition to affirm the Order of the
reservation in 1913. It certainly is a trifle late at Regional Trial Court of Makati, Branch 58, in LRC Case
this point to argue that the government had no right No. M-77,[1] which was reversed by respondent Court of
to include these properties in the reservation when Appeals in its Decision dated June 29, 1992 in CA-G.R.
the question should have been raised 83 years ago. CV No. 26122.[2] Petitioners Motion for Reconsideration
As regards the petitioners' contention that inasmuch was denied by respondent court on September 30, 1992.[3]
as they obtained the titles without government The controversy involves a total of nine thousand six
opposition, the government is now estopped from hundred fifty-seven (9,657) square meters of land
questioning the validity of the certificates of title located in Las Pias, Metro Manila. The facts show that
which were granted. As correctly pointed out by the sometime in 1908, Maria Cailles, married to James
respondent Court of Appeals, the principle of estoppel Bracewell, Sr., acquired the said parcels of land from
does not operate against the Government for the act of the Dalandan and Jimenez families of Las Pias; after
its agents. [15] which corresponding Tax Declarations were issued in
Assuming that the decrees of the Court of First the name of Maria Cailles. On January 16, 1961, Maria
Instance were really issued, the lands are still not Cailles sold the said parcels of land to her son, the
capable of appropriation. The adverse possession which petitioner, by virtue of a Deed of Sale which was duly
may be the basis of a grant of title in confirmation annotated and registered with the Registry of Deeds of
of imperfect title cases applies only to alienable Pasig, Rizal. Tax Declarations were thereafter issued
lands of the public domain. in the name of petitioner, cancelling the previous Tax
There is no question that the lands in the case at bar Declarations issued to Maria Cailles.
were not alienable lands of the public domain. As On September 19, 1963, petitioner filed before the then
testified by the District Forester, records in the Court of First Instance of Pasig, Rizal an action for
Bureau of Forestry show that the subject lands were confirmation of imperfect title under Section 48 of
never declared as alienable and disposable and subject Commonwealth Act No. 141.[4] The case was docketed as
to private alienation prior to 1913 up to the L.R.C. Case No.4328. On February 21, 1964, the Director
present.[16] Moreover, as part of the reservation for of Lands, represented by the Solicitor General, opposed
provincial park purposes, they form part of the forest petitioners application on the grounds that neither he
zone. nor his predecessors-in-interest possessed sufficient
It is elementary in the law governing natural resources title to the subject land nor have they been in open,
that forest land cannot be owned by private persons. continuous, exclusive and notorious possession and
It is not registrable and possession thereof, no matter occupation of the same for at least thirty (30) years
how lengthy, cannot convert it into private prior to the application, and that the subject land is
property,[17] unless such lands are reclassified and part of the public domain.[5]
considered disposable and alienable. The registration proceedings were meanwhile suspended
Neither do the tax receipts which were presented in on account of an action filed by Crescencio Leonardo
evidence prove ownership of the parcels of land against Maria Cailles before the then Court of First
inasmuch as the weight of authority is that tax Instance of Pasig, Rizal. The case was finally disposed
declarations are not conclusive proof of ownership in of by this Court in G.R. No. 5263 where the rights of
land registration cases.[18] Maria Cailles were upheld over those of the oppositor
Leonardo.[6]

31
Public Land V Govnt Land – Confirmation of Imperfect or incomplete title: Natural Resources Jennica Gyrl
Delfin
On March 26, 1985, the entire records of the be since time immemorial or for the period prescribed
registration case were forwarded to the Makati Regional in the Public Land Act. When the conditions set by law
Trial Court[7] where it was docketed as Land are complied with, the possessor of the land, by
Registration Case No. M-77. The Solicitor General operation of law, acquires a right to a grant, a
resubmitted his opposition to the application on July government grant, without the necessity of a
22, 1985,[8] this time alleging the following certificate of title being issued."
additional grounds: (1) the failure of petitioner to Clear from the above is the requirement that the
prosecute his action for an unreasonable length of applicant must prove that the land is alienable public
time; and (2) that the tax declarations attached to land. On this score, we agree with respondents that
the complaint do not constitute acquisition of the petitioner failed to show that the parcels of land
lands applied for. Manikx subject of his application are alienable or disposable.
On May 3, 1989, the lower court issued an Order On the contrary, it was conclusively shown by the
granting the application of petitioner.[9] The government that the same were only classified as
Solicitor General promptly appealed to respondent alienable or disposable on March 27, 1972. Thus, even
Court which, on June 29, 1992, reversed and set aside granting that petitioner and his predecessors-in-
the lower courts Order.[10] It also denied petitioners interest had occupied the same since 1908, he still
Motion for Reconsideration in its Resolution of cannot claim title thereto by virtue of such possession
September 30, 1992.[11] since the subject parcels of land were not yet
Hence, the instant Petition anchored upon the following alienable land at that time nor capable of private
grounds - appropriation. The adverse possession which may be the
"I. The Honorable Court of Appeals ERRED in finding basis of a grant of title or confirmation of an
that the commencement of thirty 30) year period imperfect title refers only to alienable or disposable
mandated under Sec. 48 (b ) shall commence only on portions of the public domain.[15]
March 27, 1972 in accordance with the classification A similar situation existed in the case of Reyes v.
made by the Bureau of Forestry in First (1st) Court of Appeals,[16] where a homestead patent issued
Indorsement dated August 20, 1986. to the petitioners predecessor-in-interest was
II. The Honorable Court of Appeals committed an ERROR cancelled on the ground t at at the time it was issued,
in DRAWING conclusion and inference that prior to the the subject land was still part of the public domain.
declaration by the Bureau of Forestry in Marc 27; 1972, In the said case, this Court ruled as follows --
the parcels of land sought to be registered by "Under the Regalian doctrine, all lands of the public
Applicant was part of the forest land or forest domain belong to the State, and that the State is the
reserves. source of any asserted right to ownership in land and
III. The Honorable Court of Appeals ERRED and failed charged with the conservation of such patrimony. This
to consider VESTED RIGHTS of the applicant-appellant same doctrine also states that all lands not otherwise
and his predecessors-in-interest land occupied from appearing to be clearly within private ownership are
1908."[12] presumed to belong to the State (Director of Lands vs.
The controversy is simple. On one hand, petitioner Intermediate Appellate Court, 219 SCRA 340). Manikan
asserts his right of title to the subject land under Hence, the burden of proof in overcoming the
Section 48 (b) of Commonwealth Act No. 141, having by presumption of State ownership of lands of the public
himself and through his predecessors-in-interest been domain is on the person applying for registration. The
in open, continuous, exclusive and notorious applicant must show that the land subject of the
possession and occupation of the subject parcels of application is alienable or disposable. This
land, under a bona fide claim of acquisition or petitioners failed to do.
ownership, since 1908. On the other hand it is the We have stated earlier that at the time the homestead
respondents position that since the subject parcels of patent was issued to petitioners predecessor-in-
land were only classified as alienable or disposable interest, the subject land belong to the inalienable
on March 27, 1972,[13] petitioner did not have any title and undisposable portion of the public domain. Thus,
to confirm when he filed his application in 1963. any title issued in their name by mistake or oversight
Neither was the requisite thirty years possession met. is void ab initio because at the time the homestead
We agree with respondents. parent was issued to petitioners, as successors-in-
In Republic vs. Doldol,[14] the requisites to acquire interest of the original patent applicant, the Director
title to public land were laid down, as follows -- of Lands was not then authorized to dispose of the same
"x x x. The original Section 48(b) of C.A. No. 141 because the area was not yet classified as disposable
provided for possession and occupation of lands of the public land. Consequently, the title issued to herein
public domain since July 26, 1894. This was superseded petitioners by the Bur au of Lands is void ab initio."
b R.A. No. 1942 which provided for a simple thirty- Prior to March 27, 1972, when the subject parcels of
year prescriptive period of occupation by an applicant land were classified as inalienable or indisposable,
for judicial confirmation of imperfect title. The same, therefore, the same could not be the subject of
however, has already been amended by Presidential confirmation of imperfect title. There can be no
Decree No. 1073, approved on January 25, 1977. As imperfect title to be confirmed over lands not yet
amended, Section 48(b) now reads: Maniks classified as disposable or alienable.[17] In the
(b) Those who by themselves or through their absence of such classification, the land remains
predecessors-in-interest have been in open, unclassified public land until released therefrom and
continuous, exclusive and notorious possession and open to disposition.[18] Indeed, it has been held that
occupation of agricultural lands of the public domain, the rules on the confirmation of imperfect title do
under a bona fide claim of acquisition or not apply unless and until the land classified as
ownership, since June 12, 1945, or earlier, forest land is released in an official proclamation to
immediately preceding the filing of the application that effect so that it may form part of the disposable
for confirmation of title, except when prevented by agricultural lands of the public domain.[19]
wars or force majeure. Those shall be conclusively Neither has petitioner shown proof that the subject
presumed to have performed all the conditions essential Forestry Administrative Order recognizes private or
to a Government grant and shall be entitled to a vested rights under which his case may fall. We only
certificate of title under the provisions of this find on record the Indorsement of the Bureau of Forest
chapter." (italicized in the original) Development[20] from which no indication of such
Thus, in the aforecited Republic vs. CA case, we stated exemption may be gleaned.
that the Public Land Act requires that the applicant Having found petitioner to have no cause of action for
must prove (a) that the land is alienable public land his application for confirmation of imperfect title,
and (b) that his open, continuous, exclusive and we see no need to discuss the other errors raised in
notorious possession and occupation of the same must this petition.

32
Public Land V Govnt Land – Confirmation of Imperfect or incomplete title: Natural Resources Jennica Gyrl
Delfin
WHEREFORE, premises considered, the instant Petition applied for was declared alienable and disposable only
is hereby DENIED for lack of merit. No pronouncement on October 15, 1980, per the certification from
as to costs. Regional Executive Director Raoul T. Geollegue of the
SO ORDERED. Department of Environment and Natural Resources,
[G.R. No. 144057. January 17, 2005] Region VI.[7] However, the court denied the motion for
REPUBLIC OF THE PHILIPPINES, petitioner, vs. THE reconsideration in an order dated February 18, 1998.[8]
HONORABLE COURT OF APPEALS and CORAZON Thereafter, the Republic appealed the decision and the
NAGUIT, respondents. order of the MCTC to the RTC, Kalibo, Aklan, Branch 8.
D E C I S I O N On February 26, 1999, the RTC rendered its decision,
TINGA, J.: dismissing the appeal.[9]
This is a Petition for Review on Certiorari under Rule Undaunted, the Republic elevated the case to the Court
45 of the 1997 Rules of Civil Procedure, seeking to of Appeals via Rule 42 of the 1997 Rules of Civil
review the Decision[1] of the Sixth Division of the Procedure. On July 12, 2000, the appellate court
Court of Appeals dated July 12, 2000 in CA-G.R. SP No. rendered a decision dismissing the petition filed by
51921. The appellate court affirmed the decisions of the Republic and affirmed in toto the assailed
both the Regional Trial Court (RTC),[2] Branch 8, of decision of the RTC.
Kalibo, Aklan dated February 26, 1999, and the Hence, the present petition for review raising a pure
7thMunicipal Circuit Trial Court (MCTC)[3] of Ibajay- question of law was filed by the Republic on September
Nabas, Aklan dated February 18, 1998, which granted 4, 2000.[10]
the application for registration of a parcel of land The OSG assails the decision of the Court of Appeals
of Corazon Naguit (Naguit), the respondent herein. contending that the appellate court gravely erred in
The facts are as follows: holding that there is no need for the governments prior
On January 5, 1993, Naguit, a Filipino citizen, of release of the subject lot from the public domain
legal age and married to Manolito S. Naguit, filed with before it can be considered alienable or disposable
the MCTC of Ibajay-Nabas, Aklan, a petition for within the meaning of P.D. No. 1529, and that Naguit
registration of title of a parcel of land situated in had been in possession of Lot No. 10049 in the concept
Brgy. Union, Nabas, Aklan. The parcel of land is of owner for the required period.[11]
designated as Lot No. 10049, Cad. 758-D, Nabas Hence, the central question for resolution is whether
Cadastre, AP 060414-014779, and contains an area of is necessary under Section 14(1) of the Property
31,374 square meters. The application seeks judicial Registration Decree that the subject land be first
confirmation of respondents imperfect title over the classified as alienable and disposable before the
aforesaid land. applicants possession under a bona fide claim of
On February 20, 1995, the court held initial hearing ownership could even start.
on the application. The public prosecutor, appearing The OSG invokes our holding in Director of Lands v.
for the government, and Jose Angeles, representing the Intermediate Appellate Court[12] in arguing that the
heirs of Rustico Angeles, opposed the petition. On a property which is in open, continuous and exclusive
later date, however, the heirs of Rustico Angeles filed possession must first be alienable. Since the subject
a formal opposition to the petition. Also on February land was declared alienable only on October 15, 1980,
20, 1995, the court issued an order of general default Naguit could not have maintained a bona fide claim of
against the whole world except as to the heirs of ownership since June 12, 1945, as required by Section
Rustico Angeles and the government. 14 of the Property Registration Decree, since prior to
The evidence on record reveals that the subject parcel 1980, the land was not alienable or disposable, the
of land was originally declared for taxation purposes OSG argues.
in the name of Ramon Urbano (Urbano) in 1945 under Tax Section 14 of the Property Registration Decree,
Declaration No. 3888 until 1991.[4] On July 9, 1992, governing original registration proceedings, bears
Urbano executed a Deed of Quitclaim in favor of the close examination. It expressly provides:
heirs of Honorato Maming (Maming), wherein he renounced SECTION 14. Who may apply. The following persons may
all his rights to the subject property and confirmed file in the proper Court of First Instance an
the sale made by his father to Maming sometime in 1955 application for registration of title to land, whether
or 1956.[5] Subsequently, the heirs of Maming executed personally or through their duly authorized
a deed of absolute sale in favor of respondent Naguit representatives:
who thereupon started occupying the same. She (1) those who by themselves or through their
constituted Manuel Blanco, Jr. as her attorney-in-fact predecessors-in-interest have been in open,
and administrator. The administrator introduced continuous, exclusive and notorious possession and
improvements, planted trees, such as mahogany, coconut occupation of alienable and disposable lands of the
and gemelina trees in addition to existing coconut public domain under a bona fide claim of ownership
trees which were then 50 to 60 years old, and paid the since June 12, 1945, or earlier.
corresponding taxes due on the subject land. At (2) Those who have acquired ownership over private
present, there are parcels of land surrounding the lands by prescription under the provisions of existing
subject land which have been issued titles by virtue laws.
of judicial decrees. Naguit and her predecessors-in- . . . .
interest have occupied the land openly and in the There are three obvious requisites for the filing of
concept of owner without any objection from any private an application for registration of title under Section
person or even the government until she filed her 14(1) that the property in question is alienable and
application for registration. disposable land of the public domain; that the
After the presentation of evidence for Naguit, the applicants by themselves or through their
public prosecutor manifested that the government did predecessors-in-interest have been in open,
not intend to present any evidence while oppositor Jose continuous, exclusive and notorious possession and
Angeles, as representative of the heirs of Rustico occupation, and; that such possession is under a bona
Angeles, failed to appear during the trial despite fide claim of ownership since June 12, 1945 or earlier.
notice. On September 27, 1997, the MCTC rendered a Petitioner suggests an interpretation that the
decision ordering that the subject parcel be brought alienable and disposable character of the land should
under the operation of the Property Registration Decree have already been established since June 12, 1945 or
or Presidential Decree (P.D.) No. 1529 and that the earlier. This is not borne out by the plain meaning of
title thereto registered and confirmed in the name of Section 14(1). Since June 12, 1945, as used in the
Naguit.[6] provision, qualifies its antecedent phrase under a
The Republic of the Philippines (Republic), thru the bonafide claim of ownership. Generally speaking,
Office of the Solicitor General (OSG), filed a motion qualifying words restrict or modify only the words or
for reconsideration. The OSG stressed that the land phrases to which they are immediately associated, and

33
Public Land V Govnt Land – Confirmation of Imperfect or incomplete title: Natural Resources Jennica Gyrl
Delfin
not those distantly or remotely located.[13] Ad proximum inapplicable, as correctly held by the Court of
antecedents fiat relation nisi impediatur sentencia. Appeals.[23]
Besides, we are mindful of the absurdity that would It must be noted that the present case was decided by
result if we adopt petitioners position. Absent a the lower courts on the basis of Section 14(1) of the
legislative amendment, the rule would be, adopting the Property Registration Decree, which pertains to
OSGs view, that all lands of the public domain which original registration through ordinary registration
were not declared alienable or disposable before June proceedings. The right to file the application for
12, 1945 would not be susceptible to original registration derives from a bona fide claim of
registration, no matter the length of unchallenged ownership going back to June 12, 1945 or earlier, by
possession by the occupant. Such interpretation reason of the claimants open, continuous, exclusive
renders paragraph (1) of Section 14 virtually and notorious possession of alienable and disposable
inoperative and even precludes the government from lands of the public domain.
giving it effect even as it decides to reclassify A similar right is given under Section 48(b) of the
public agricultural lands as alienable and disposable. Public Land Act, which reads:
The unreasonableness of the situation would even be Sec. 48. The following described citizens of the
aggravated considering that before June 12, 1945, the Philippines, occupying lands of the public domain or
Philippines was not yet even considered an independent claiming to own any such land or an interest therein,
state. but those titles have not been perfected or completed,
Instead, the more reasonable interpretation of Section may apply to the Court of First Instance of the
14(1) is that it merely requires the property sought province where the land is located for confirmation of
to be registered as already alienable and disposable their claims and the issuance of a certificate of title
at the time the application for registration of title therefor, under the Land Registration Act, to wit:
is filed. If the State, at the time the application is xxx xxx xxx
made, has not yet deemed it proper to release the (b) Those who by themselves or through their
property for alienation or disposition, the predecessors in interest have been in open, continuous,
presumption is that the government is still reserving exclusive, and notorious possession and occupation of
the right to utilize the property; hence, the need to agricultural lands of the public domain, under a bona
preserve its ownership in the State irrespective of fide claim of acquisition of ownership, for at least
the length of adverse possession even if in good faith. thirty years immediately preceding the filing of the
However, if the property has already been classified application for confirmation of title except when
as alienable and disposable, as it is in this case, prevented by war or force majeure. These shall be
then there is already an intention on the part of the conclusively presumed to have performed all the
State to abdicate its exclusive prerogative over the conditions essential to a Government grant and shall
property. be entitled to a certificate of title under the
This reading aligns conformably with our holding provisions of this chapter.
in Republic v. Court of Appeals.[14] Therein, the Court When the Public Land Act was first promulgated in 1936,
noted that to prove that the land subject of an the period of possession deemed necessary to vest the
application for registration is alienable, an right to register their title to agricultural lands of
applicant must establish the existence of a positive the public domain commenced from July 26, 1894.
act of the government such as a presidential However, this period was amended by R.A. No. 1942,
proclamation or an executive order; an administrative which provided that the bona fide claim of ownership
action; investigation reports of Bureau of Lands must have been for at least thirty (30) years. Then in
investigators; and a legislative act or a 1977, Section 48(b) of the Public Land Act was again
statute.[15] In that case, the subject land had been amended, this time by P.D. No. 1073, which pegged the
certified by the DENR as alienable and disposable in reckoning date at June 12, 1945. This new starting
1980, thus the Court concluded that the alienable point is concordant with Section 14(1) of the Property
status of the land, compounded by the established fact Registration Decree.
that therein respondents had occupied the land even Indeed, there are no material differences between
before 1927, sufficed to allow the application for Section 14(1) of the Property Registration Decree and
registration of the said property. In the case at bar, Section 48(b) of the Public Land Act, as amended. True,
even the petitioner admits that the subject property the Public Land Act does refer to agricultural lands
was released and certified as within alienable and of the public domain, while the Property Registration
disposable zone in 1980 by the DENR.[16] Decree uses the term alienable and disposable lands of
This case is distinguishable from Bracewell v. Court the public domain. It must be noted though that the
of Appeals,[17] wherein the Court noted that while the Constitution declares that alienable lands of the
claimant had been in possession since 1908, it was only public domain shall be limited to agricultural
in 1972 that the lands in question were classified as lands.[24] Clearly, the subject lands under Section
alienable and disposable. Thus, the bid at registration 48(b) of the Public Land Act and Section 14(1) of the
therein did not succeed. In Bracewell, the claimant Property Registration Decree are of the same type.
had filed his application in 1963, or nine (9) years Did the enactment of the Property Registration Decree
before the property was declared alienable and and the amendatory P.D. No. 1073 preclude the
disposable. Thus, in this case, where the application application for registration of alienable lands of the
was made years after the property had been certified public domain, possession over which commenced only
as alienable and disposable, the Bracewell ruling does after June 12, 1945? It did not, considering Section
not apply. 14(2) of the Property Registration Decree, which
A different rule obtains for forest lands,[18] such as governs and authorizes the application of those who
those which form part of a reservation for provincial have acquired ownership of private lands by
park purposes[19] the possession of which cannot ripen prescription under the provisions of existing laws.
into ownership.[20] It is elementary in the law Prescription is one of the modes of acquiring ownership
governing natural resources that forest land cannot be under the Civil Code.[25] There is a consistent
owned by private persons. As held in Palomo v. Court jurisprudential rule that properties classified as
of Appeals,[21] forest land is not registrable and alienable public land may be converted into private
possession thereof, no matter how lengthy, cannot property by reason of open, continuous and exclusive
convert it into private property, unless such lands possession of at least thirty (30) years.[26] With such
are reclassified and considered disposable and conversion, such property may now fall within the
alienable.[22] In the case at bar, the property in contemplation of private lands under Section 14(2),
question was undisputedly classified as disposable and and thus susceptible to registration by those who have
alienable; hence, the ruling in Palomo is acquired ownership through prescription. Thus, even if
possession of the alienable public land commenced on a

34
Public Land V Govnt Land – Confirmation of Imperfect or incomplete title: Natural Resources Jennica Gyrl
Delfin
date later than June 12, 1945, and such possession This is a petition for review on certiorari seeking a
being been open, continuous and exclusive, then the reversal of the decision of Respondent Court of Appeals
possessor may have the right to register the land by 1 dated February 5, 1975 in CA-G.R. No. 50076-R,
virtue of Section 14(2) of the Property Registration entitled "EMILIO BERNABE, SR., Et. Al. v. REPUBLIC OF
Decree. THE PHILIPPINES," affirming the order of the Court of
The land in question was found to be cocal in nature, First Instance of Bataan dated August 14, 1971 in
it having been planted with coconut trees now over Cadastral Case No. 19, LRC Cadastral Record No. 1097,
fifty years old.[27] The inherent nature of the land but which dismissed petitioner Republic’s petition for
confirms its certification in 1980 as alienable, hence review of the decrees of registration issued pursuant
agricultural. There is no impediment to the application to the decision rendered on December 17, 1968
of Section 14(1) of the Property Registration Decree, adjudicating in favor of the private Respondents
as correctly accomplished by the lower courts. herein, the lots applied for by them, and the
The OSG posits that the Court of Appeals erred in Resolution of Respondent Court dated March 19, 1975
holding that Naguit had been in possession in the denying herein Petitioner’s motion for
concept of owner for the required period. The argument reconsideration.
begs the question. It is again hinged on the
assertionshown earlier to be unfoundedthat there could The undisputed facts are as follows:chanrob1es virtual
have been no bona fide claim of ownership prior to 1aw library
1980, when the subject land was declared alienable or
disposable. Lot No. 622 of the Mariveles Cadastre was declared
We find no reason to disturb the conclusion of both public land in a decision rendered before the last war
the RTC and the Court of Appeals that Naguit had the in Cadastral Case No. 19, LRC Cadastral Record No.
right to apply for registration owing to the continuous 1097.
possession by her and her predecessors-in-interest of
the land since 1945. The basis of such conclusion is On July 6, 1965, Lot 622 was segregated from the forest
primarily factual, and the Court generally respects zone and released and certified by the Bureau of
the factual findings made by lower courts. Notably, Forestry as an agricultural land for disposition under
possession since 1945 was established through proof of the Public Land Act (Record on Appeal, p. 7).
the existence of 50 to 60-year old trees at the time
Naguit purchased the property as well as tax On April 26, 1967, Respondents filed in the Court of
declarations executed by Urbano in 1945. Although tax First Instance of Bataan a petition to reopen Cadastral
declarations and realty tax payment of property are Case No. 19, LRC Cadastral Record No. 1097, under
not conclusive evidence of ownership, nevertheless, Republic Act 931, as amended by Republic Act 2061,
they are good indicia of the possession in the concept concerning a portion of Lot No. 622 — Lot Nos. 792,
of owner for no one in his right mind would be paying 793, 794, 795, 796, 797, 798 — and a portion of Lot
taxes for a property that is not in his actual or at No. 324 — Lot Nos. 791 and 799 — more particularly
least constructive possession. They constitute at identified and delineated in the segregation plans of
least proof that the holder has a claim of title over Sgs-3343, Sgs-3440, Sgs-3340, Sgs-3341, Sgs-3342 and
the property. The voluntary declaration of a piece of Sgs-3339, approved by the Director of Lands, to perfect
property for taxation purposes manifests not only ones their rights and register their titles to said lots,
sincere and honest desire to obtain title to the having allegedly acquired ownership and possession of
property and announces his adverse claim against the said parcels of land by purchase from the original
State and all other interested parties, but also the owners thereof, whose possession of the same including
intention to contribute needed revenues to the that of the herein Respondents, has always been
Government. Such an act strengthens ones bona continuous, open, active, exclusive, public, adverse,
fide claim of acquisition of ownership.[28] and in the concept of owners thereof for more than 30
Considering that the possession of the subject parcel years (Record on Appeal, pp. 3-5 and 11).
of land by the respondent can be traced back to that
of her predecessors-in-interest which commenced since On May 17, 1967, the lower court issued an Order
1945 or for almost fifty (50) years, it is indeed setting the petition for hearing and directing that
beyond any cloud of doubt that she has acquired title the Republic of the Philippines be notified thereof by
thereto which may be properly brought under the furnishing the Solicitor-General, the Director of
operation of the Torrens system. That she has been in Lands and the Director of Forestry, a copy of said
possession of the land in the concept of an owner, Order together with Respondents’ petition by
open, continuous, peaceful and without any opposition registered mail (Record on Appeal, p. 6).
from any private person and the government itself makes
her right thereto undoubtedly settled and deserving of On August 24, 1967, the Director of Forestry filed an
protection under the law. opposition to the petition praying for the denial of
WHEREFORE, foregoing premises considered, the the petition once the area involved is found to be
assailed Decision of the Court of Appeals dated July within the timberland and therefore inalienable under
12, 2000 is hereby AFFIRMED. No costs. the Constitution (Record on Appeal, p. 7). Upon
SO ORDERED. verification, however, the Director of Forestry found
the area to be the portion of the timberland already
SECOND DIVISION released by the government from the mass of public
forests and promptly withdrew his Opposition (Record
[G.R. No. L-40402. March 16, 1987.] on Appeal, p. 8).

REPUBLIC OF THE PHILIPPINES, Petitioner, v. THE HON. On September 1, 1967, the Acting Provincial Fiscal of
COURT OF APPEALS, and EMILIO BERNABE, SR., EMILIO Bataan, for and in behalf of the Director of Lands,
BERNABE, JR., LUZ BERNABE, AMPARO BERNABE, and ELISA filed his opposition to the petition alleging that the
BERNABE, Respondents. land is still, in truth and in fact, public land and
as such cannot be the subject of a land registration
proceeding under Act 496.
D E C I S I O N
The lower court found that the petitioners have
complied with all the terms and conditions which would
PARAS, J.: entitle them to a grant. Thus, the dispositive portion
of its decision dated December 17, 1968 (Record on
Appeal, p. 19), reads:jgc:chanrobles.com.ph

35
Public Land V Govnt Land – Confirmation of Imperfect or incomplete title: Natural Resources Jennica Gyrl
Delfin
Solicitor-General even filed the petition for review
"WHEREFORE, the segregation plans, Sgs-3340, Sgs-3339, (Record on Appeal, pp. 27-40).
Sgs-3341, Sgs-3342, Sgs-3343 and Sgs-3340 and their
technical descriptions are hereby APPROVED, and Their motion to dismiss having been held in abeyance
pursuant to Sec. 11 of Act 2259, the court hereby until the hearing of the merits of the case which was
adjudicates in favor of petitioners Emilio Bernabe, set for August 16, 1970, respondents filed their answer
Sr., married; Emilio Bernabe, Jr., married; Luz to the Petition for Review on August 4, 1970. In their
Bernabe, single; Amparo Bernabe, single and Elisa answer, respondents reiterated their grounds in their
Bernabe, single, all Filipinos and residents of motion to dismiss (Record on Appeal, pp. 40-44).
Balanga, Bataan, the lots herein applied for as
follows:chanrob1es virtual 1aw library On November 12, 1970, Petitioner filed an amended
Petition for Review, with the additional allegation
Luz Bernabe Sgs-791 82,771 sq. m. that after having fraudulently secured title over the
parcels of land involved, the petitioners executed
3339 simulated deeds of sale purporting to convey various
lots composing portions of the parcels involved to
Elisa Bernabe Sgs-793 71.596 sq. m. third parties for fictitious considerations in an
obvious attempt to remove the parcels of land involved
3341 from the coverage of Section 38 of Act 496, but in
truth, the aforementioned third parties are not
Amparo Bernabe Sgs-794 43,399 sq. m. innocent purchasers for value, being mere dummies of
the petitioners, holding the parcels of land involved
3342 795 100,439 sq. m. only in trust for the petitioners. On November 23,
1970, respondents filed their answer to the Amended
Josefina Bernabe Sgs-796 69,355 sq. m. Petition for Review (Record on Appeal, p. 56).

3343 797 75,100 sq. m. On August 14, 1971, the lower court issued its Order
denying petitioner’s Amended Petition for Review
Emilio Bernabe, Jr. Sgs-798 100,183 sq. m. (Record on Appeal, p. 56).

Sgs-3440 Sgs-799 64,052. sq. m. On appeal to the Court of Appeals on September 20,
1971, the questioned Order of the Court of First
and upon this decision having become final, the Instance of Bataan, Branch I was affirmed (Rollo, p.
Commissioner of Land Registration is hereby directed 33).
to issue the corresponding decrees of registration
therefor."cralaw virtua1aw library On February 25, 1975, Petitioner filed a Motion for
Reconsideration which was denied by the Court of
Pursuant to the aforecited decision, the Commissioner Appeals for lack of merit, in the Resolution of a
of Land Registration issued Decrees Nos. N-124813- special Division of Five, promulgated on March 19,
124818, all dated May 7, 1969 (Record on Appeal, pp. 1975.
20-25).
Hence this petition.
On May 7, 1979, petitioner Republic of the Philippines,
acting in its behalf and in behalf of the Director of Without giving due course to the Petition, the Court,
Lands and the Director of Forestry, through the through its First Division, resolved on May 5, 1975 to
Solicitor-General, filed a petition for review of the require the respondents to comment thereon. On May 30,
decrees of registration under Section 38, of Act No. 1975, respondents filed their comment, alleging that
496, as amended, and the corresponding decision of the the decision of respondent Court and the questioned
lower court, on the grounds that the entire proceeding resolution were not rendered without or in excess of
was vitiated by lack of notice to the Solicitor General its jurisdiction. Neither was the discretion exercised
of the subsequent hearings of the petition for re- by respondent Court arbitrary or despotic.
opening of the cadastral proceedings; that the parcels
of land subject matter of the petition to re-open In its Resolution dated June 4, 1975, the Court
cadastral proceedings are portions of the public resolved to give due course to the Petition and denied
domain, admittedly within the unclassified public the urgent motion of respondents for leave to file a
forest of Mariveles, Bataan, opened for disposition supplemental and/or amended comment. Petitioners filed
only on or about July 6, 1965; that subsequently, its Brief on November 29, 1975; respondents, on March
respondents do not have a registerable title to the 2, 1976. Petitioner filed its Reply Brief on March 25,
land subject matter of the proceedings; and the lower 1976 and on May 5, 1976, the case was deemed submitted
court, without jurisdiction to decree the confirmation for decision.
of registerable title to respondents over portions of
the public domain, as respondents do not qualify under Petitioner assigns the following errors:chanrob1es
the provisions of Section 48(b) of CA 141, as amended, virtual 1aw library
and that under the circumstances, respondents employed
actual fraud in procuring title over the parcels of I. THE RESPONDENT COURT OF APPEALS COMMITTED A
land (Record on Appeal, p. 25). REVERSIBLE ERROR IN TOTALLY DISREGARDING THE
UNDISPUTED FACT THAT THE LOTS CLAIMED BY HEREIN PRIVATE
On May 29, 1979, respondents moved to dismiss the RESPONDENTS BECAME AGRICULTURAL ONLY ON JULY 6, 1965
Petition for Review on the grounds that: (1) The trial WHEN THE SAME WERE RELEASED FROM THE FOREST ZONE AND
court has no jurisdiction over the nature of the action THAT CONSEQUENTLY THEY LACK THE REQUISITE THIRTY (30)
or suit as there is no fraud to justify the setting YEARS POSSESSION TO ENTITLE THEM TO A GRANT.
aside on review of a decree of registration. If the
Solicitor General was not notified of the subsequent II. THE RESPONDENT COURT OF APPEALS COMMITTED A
hearings, it was because he delegated his appearance REVERSIBLE ERROR IN NOT HOLDING THAT THE ENTIRE
to the Provincial Fiscal of Bataan. Besides the setting PROCEEDING FOR REOPENING OF THE CADASTRAL CASE OVER
aside or review was filed out of time. (2) The petition THE LOTS IN QUESTION WAS VITIATED BY LACK OF NOTICE TO
states no cause of action, the parcels of land involved THE SOLICITOR-GENERAL.
in the actions having been already transferred to
innocent purchasers for value long before the III. THE RESPONDENT COURT ERRED IN NOT HOLDING THAT

36
Public Land V Govnt Land – Confirmation of Imperfect or incomplete title: Natural Resources Jennica Gyrl
Delfin
THE ALLEGED TRANSFER OF THE LOTS IN QUESTION BY PRIVATE Thus, even if the reopening of the cadastral
RESPONDENTS TO THIRD PARTIES WHEN THEIR TITLES WERE proceedings was at all possible, private respondents
STILL SUBJECT TO THE ONE-YEAR PERIOD OF REVIEW have not qualified for a grant under Sec. 48(b) of
CONSTITUTES FRAUD SCHEMED BY THE TRANSFERORS AS A MEANS Commonwealth Act 141, the facts being that private
OF FRUSTRATING ANY ACTION AIMED AT NULLIFYING THEIR respondents could only be credited with 1 year, 9
TITLES THERETO. months and 20 days possession and occupation of the
lots involved, counted from July 6, 1965, the date when
The government’s cause is meritorious. the land area in sitio San Jose, barrio Cabcaban,
I Mariveles, Bataan, known as Bataan PMD No. 267, which
includes the lots claimed by respondents, had been
segregated from the forest zone and released by the
It is evident from the facts of the case at bar that Bureau of Forestry as an agricultural land for
private respondents did file a claim for Lot No. 622 disposition under the Public Land Act. (Record on
of the Mariveles Cadastre and in fact a decision was Appeal, p. 19). Consequently, under the above mentioned
rendered before the last war in Cadastral Case No. 19 jurisprudence, neither private respondents nor their
LRC Cadastral Record No. 1097, declaring the lot in predecessors-in-interest could have possessed the lots
question as public land. It must be stressed that said for the requisite period of thirty (30) years as
lot was declared public land by virtue of a court disposable agricultural land.
decision which has become final and as held by the II
Supreme Court aforesaid decision is res judicata.
(Republic v. Estenzo, 120 SCRA 222 [1983]). It is
therefore beyond question that the trial court has no Petitioner argues that the government, being a
jurisdiction to reopen the cadastral proceeding under necessary party in the cadastral case, as reopened,
R.A. 931 as amended by R.A. 2061 and the decision its counsel, the Solicitor-General, should have been
therein rendered is null and void ab initio. furnished copies of all court orders, notices and
decisions, as in ordinary cases, in order to bind the
Furthermore, it is undisputed that aforesaid Lot No. government. Failure to give such notice deprives the
622 was released as an agricultural land for State of its day in Court, and renders the decision
disposition under Public Land Act only on July 6, 1965. void. (Brief for Petitioner, pp. 16-17).
The lower court ordered the issuance of the
corresponding decrees of registration for the lots, The records show that the Solicitor-General was duly
pursuant to Sec. 48(b), C.A. 141, otherwise known as notified of the initial hearing on the petition to
the Public Land Act, as amended by Republic Act No. reopen Cadastral Case No. 19 but thereafter, notice of
1942, providing for the confirmation of imperfect or subsequent hearings as well as a copy of the decision
incomplete titles, which reads:jgc:chanrobles.com.ph itself promulgated by the lower court on December 19,
1968 was sent instead to the Provincial Fiscal of
"(b) Those who by themselves or through their Bataan, admittedly the duly authorized representative
predecessors in interest have been in open, continuous, of the Solicitor-General in the cadastral proceeding
exclusive, and notorious possession and occupation of as shown in a telegram dated January 19, 1968. (Record
agricultural lands of the public domain, under a bona on Appeal, p. 47).
fide claim of acquisition of ownership, for at least
thirty years immediately preceding the filing of the In the case of Republic v. Director of Lands (71 SCRA
application for confirmation of title except when 426 [1976], the Supreme Court, applying the time-
prevented by war or force majeure. These shall be honored principle of agency ruled that the service of
conclusively presumed to have performed all the the questioned decision on the Provincial Fiscal must
conditions essential to a Government grant and shall necessarily be service on the Solicitor-General, and
be entitled to a certificate of title under the added that technical transgressions relative to the
provisions of this chapter."cralaw virtua1aw library filing and service may be brushed aside when the
adverse party (this time the Director of Lands and
As pointed out by petitioner, the question is whether Forestry and their counsel, the Solicitor-General) is
or not the lots claimed by respondents could legally aware of the matter which his adversary would want the
be the subject of a judicial confirmation of title court to act upon. Once it appears that the party is
under the aforequoted provisions of the Public Land already informed by one means or another of what he is
Act, as amended. to be notified, the required service becomes an empty
gesture and strict observance thereof is considered
The answer is in the negative. waived. (Citing Estrada v. Sto. Domingo, 28 SCRA 890
[1969]).
Section 48(b) of C.A. No. 141, as amended, applies
exclusively to public agricultural land. Forest lands In the case at bar, it does not appear that the
or areas covered with forests are excluded. They are Solicitor General was so apprised of the decision of
incapable of registration and their inclusion in a the lower court in question as there is no proof that
title, whether such title be one issued during the the Provincial Fiscal of Bataan ever sent the
Spanish sovereignty or under the present Torrens system Solicitor-General a copy thereof. Furthermore, after
of registration, nullifies the title (Li Seng Giap v. the 3rd Assistant Provincial Fiscal filed a notice of
Director of Lands, 55 Phil. 693 [1931]; Director of appeal from the decision of the trial court, the
Lands v. Reyes, 68 SCRA 177 [1975]). Thus, possession Provincial Fiscal on March 21, 1969 manifested that he
of forest lands, however long, cannot ripen into was withdrawing the appeal upon the intervention of
private ownership (Vano v. Government, 41 Phil. 161 the District Forester. (Respondent’s Brief, p. 44).
[1920]; Adorable v. Director of Forestry, 107 Phil.
401 [1960]; Director of Forestry v. Muñoz, 23 SCRA 1183 It will be observed however that later decisions of
[1968]; Director of Lands v. Abanzado, 65 SCRA 5 the Supreme Court tend to be more strict in the matter
[1975]). A parcel of forest land is within the of giving notice to the Solicitor General. In a more
exclusive jurisdiction of the Bureau of Forestry and recent case, Republic v. Court of Appeals, 135 SCRA
beyond the power and jurisdiction of the cadastral 161 [1985], it was established that the Solicitor-
court to register under the Torrens System (Republic General is the only legal counsel of the government in
v. Court of Appeals, 89 SCRA 648 [1979]; Republic v. land registration cases and as such, he alone may
Vera (120 SCRA 210 [1983]; Director of Lands v. Court withdraw the Government’s appeal with binding effect
of Appeals, 129 SCRA 689 [1984]. on the latter. He is entitled to be furnished copies
of all court orders, notices and decisions and as held

37
Public Land V Govnt Land – Confirmation of Imperfect or incomplete title: Natural Resources Jennica Gyrl
Delfin
the reglementary thirty-day period for appeal should Et Al., 37 Phil. 651[1918]. Without the needed
be reckoned from the time the Solicitor-General’s verification, he cannot claim to be an innocent
Office is apprised of the 1970 order of denial and not purchaser for value in contemplation of law.
from the time the special counsel or the fiscal was
served with that order. Thus, representatives of the Moreover, it is well-settled that a certificate of
Solicitor General in the case at bar, had no power to title is void, when it covers property of public domain
decide whether or not an appeal should be made. They classified as forest or timber and mineral lands. Any
should have referred the matter to the Solicitor- title issued on non-disposable lots even in the hands
General and without copies of court orders, notices of an alleged innocent purchaser for value, shall be
and decisions, having been provided by either the trial cancelled. (Lepanto Consolidated Mining Company v.
court or the Provincial Fiscal of Bataan to the Dumyung, 89 SCRA 540 [1979] underscoring supplied). In
Solicitor-General, the assailed decision has no the case at bar, it will be noted that in granting
binding effect on the government. titles to the land in dispute, the lower court counted
III the period of possession of private respondents before
the same were released as forest lands for disposition,
which release is tantamount to qualifying the latter
The petition for review of Decrees Nos. N-124813 to N- to a grant on said lands while they were still non-
124818 under Sec. 38 of Act No. 496 as amended was disposable. Thus, under the foregoing rulings, even
filed by the Solicitor General on May 7, 1970 in assuming that the transferees are innocent purchasers
representation of the Republic of the Philippines, in for value, their titles to said lands derived from the
the same Cadastral Case No. 19, LRC Cadastral Record titles of private respondents which were not validly
No. 1097, exactly a year after the issuance of issued as they cover lands still a part of the public
aforesaid decrees of registration, on the ground of domain, may be cancelled.
actual fraud. (Record on Appeal, pp. 43-44).
PREMISES CONSIDERED, the assailed decision of the Court
The basic elements for the allowance of the reopening of Appeals and the decision of the Court of First
or review of a decree, are: (1) that the petitioner Instance are hereby SET ASIDE and REVERSED, because
has real or dominical right; (2) that he has been the lots in question still form part of the public
deprived thereof through fraud; (3) that the petition domain. The certificates of title issued over them are
is filed within one year from the issuance of the hereby ordered CANCELLED.
decree and (4) that the property has not as yet been
transferred to an innocent purchaser. (Libudan v. Gil, SO ORDERED.
45 SCRA 27 [1972]; Rubico, Et. Al. v. Orellana, 30 SCRA
513 [1969]). It has been held however that the action FIRST DIVISION
to annul a judgment, upon the ground of fraud would be G.R. No. 166890, June 28, 2016
unavailing unless the fraud be extrinsic or collateral REPUBLIC OF THE PHILIPPINES, Petitioner, v. APOLONIO
and the facts upon which it is based have not been BAUTISTA, JR., Respondent.
controverted or resolved in the case where the judgment D E C I S I O N
sought to be annulled was rendered. (Libudan v. Gil, BERSAMIN, J.:
supra). Review of the decree demands a showing of The applicant for judicial confirmation of imperfect
actual (not constructive) fraud, i.e. actual malice. title must trace his possession of the subject land to
(Rublico v. Orellana, supra). June 12, 1945, or earlier. Any length of possession
that does not comply with the requirement cannot
In the case at bar, it cannot be said that private support the application, which must be then dismissed
respondents employed actual fraud in procuring titles for failure to comply with Commonwealth Act No. 141
over parcels of land of the public domain as it is a (Public Land Act) and Presidential Decree No. 1529
matter of record that the land in question was opened (Property Registration Decree).
for disposition and alienation only on July 6, 1965. The Case
The matter was threshed out in the lower court and the
decision of the latter was affirmed by the Court of The Government appeals the adverse judgment
Appeals. Actual malice is therefore absent. promulgated on September 30, 2004,1 whereby the Court
of Appeals (CA) affirmed the decision of the Municipal
However, it has been held that, if a decree issued in Trial Court (MTC) of Subic, Zambales rendered on
pursuance of a valid decision, obtained by fraud, may November 17, 1998 in LRC Case No. N-12-10-96
be annulled within one (1) year from entry of said entitled In Re: Application for Land Registration of
decree, there is more reason to hold that the same is Lot 17078 of Cad. 547-D, Subic Cadastre2 granting the
true if entered in compliance with a decision suffering application of respondent Apolonio Bautista, Jr. for
from a fatal infirmity, such as want of due process, the judicial confirmation of title of Lot 17078 of Cad.
(Vda. de Cuaycong v. Vda. de Sangbengoo, 110 Phil. 118 547-D, Subic Cadastre.
[1960] or lack of jurisdiction of the court that Antecedents
decided the cadastral case. (Republic v. De Kalintas,
25 SCRA 720 [1969]). Thus, on both counts, the case at After acquiring Lot 17078 of Cad. 547-D, Subic
bar can properly be the subject of review, it having Cadastre, located in Capisanan, Subic, Zambales from
been shown that the Solicitor-General was not properly Mario Jardin on February 15, 1971 and Cornelia
furnished the requisite notices and copy of the Villanueva on May 25, 1973, Apolonio, Sr. had the
assailed decision but more importantly, the lower court property declared for taxation purposes. He had been
as previously stated had no jurisdiction to re-open the sole and exclusive possessor and occupant from the
the cadastral proceeding under Republic Act 931 as time of acquisition until his death, with no party
amended by R.A. No. 2061. questioning his possession and ownership, or staking
IV any adverse claim against him thereon.3 He died in
1987, and was succeeded by his children, namely:
respondent Apolonio, Jr. and his siblings. Apolonio,
As to whether or not the transferees of the lot in Sr.'s children executed an extra-judicial settlement
question are innocent purchasers for value, it is a of their father's estate, whereby Apolonio, Jr.'s
well settled rule that a purchaser cannot close his brothers and sisters waived their rights in his favor.
eyes to facts which should put a reasonable man upon Thus, the property was declared for taxation purposes
his guard, and then claim that he acted in good faith in Apolonio, Jr.'s name under Tax Declaration No. 014-
under the belief that there was no defect in the title 0432A of the Municipality of Subic, Zambales. There
of the vendor. (Leung Yee v. F.L. Strong Machiner Co., were no arrears in real estate taxes.4 The declared

38
Public Land V Govnt Land – Confirmation of Imperfect or incomplete title: Natural Resources Jennica Gyrl
Delfin
value was P73,040.00.5chanrobleslaw The Government has correctly insisted that the
requisite period of possession of the property should
On October 21, 1996, Apolonio Jr. commenced LRC Case conform to that provided for in Section 48(b) of
No. N-12-10-96 in the MTC. He later on testified that the Public Land Act, as amended by Presidential Decree
his father had been in actual possession since 1969, No. 1073, which has limited the right to apply for
and had eventually acquired the land from Jardin and judicial confirmation to citizens of the Philippines
Villanueva through the notarized Deeds of Absolute Sale "who by themselves or through their predecessors in
dated February 15, 1971, and May 25, 1973; and that interest have been in open, continuous, exclusive, and
his father had paid taxes on the land. notorious possession and occupation of alienable and
disposable lands of the public domain, under a bona
The Government did not interpose any timely objection fide claim of acquisition of ownership, since June 12,
to the testimony of Apolonio, Jr. It did not also 1945, or earlier, immediately preceding the filing of
object to the documentary evidence (i.e., the deeds of the application for confirmation of title except when
absolute sale and tax declarations) offered by him. prevented by war or force majeure. x x x" The provision
Hence, the MTC admitted all the evidence presented by is reprised by Section 14(1) of Presidential Decree
Apolonio, Jr. No. 1529 (Property Registration Decree), adopting the
length of possession and occupation of alienable and
In due course, the MTC granted Apolonio, Jr.'s disposable lands of the public domain under a bona
application, and declared him as the owner in fee fide claim of ownership since June 12, 1945, or
simple of the land,6 and confirmed his ownership earlier.
thereof.7chanrobleslaw
We note that in its amendment of the Public Land
The Government appealed the decision to the Court of Act that took effect on January 25, 1977, Presidential
Appeals (CA), which, on September 30, 2004, promulgated Decree No. 1073 changed the length of the requisite
its assailed decision affirming the ruling of the possession from "thirty (30) years immediately
MTC.8 The CA pointed out that the Government did not preceding the filing of the application" to possession
present evidence against the claim of Apolonio Jr.; "since June 12, 1945, or earlier." Republic v.
and that the Government did not timely object to his Naguit15 has explained this change
testimony on the ground of its being thusly:ChanRoblesVirtualawlibrary
hearsay.9chanrobleslaw When the Public Land Act was first promulgated in 1936,
Issue the period of possession deemed necessary to vest the
right to register their title to agricultural lands of
In this appeal, the Government reiterates that the the public domain commenced from July 26, 1894.
testimony of Apolonio, Jr. on possession, being However, this period was amended by R.A. No. 1942,
hearsay, had no probative value; that the alienation which provided that the bona fide claim of ownership
of public land should always undergo careful scrutiny; must have been for at least thirty (30) years. Then in
and that the Court should carefully re-examine the 1977, Section 48(b) of the Public Land Act was again
factual issues that could alter the result of the amended, this time by P.D. No. 1073, which pegged the
case.10chanrobleslaw reckoning dale at June 12, 1945. x x x
Based on the records before us, Apolonio, Jr. presented
The Government points out that Apolonio, Jr. had given only himself to establish the possession and ownership
only general statements pertaining to the open, of his father, Apolonio, Sr., who was his immediate
continuous, exclusive and notorious possession of his predecessor-in-interest. Me did not present as
father since 1971; that such statements were mere witnesses during the trial either of the transferors
conclusions of law, and did not prove the alleged of Apolonio, Sr. - that is, Mario Jardin or Cornelia
possession; that because the application for judicial Villanueva - to establish the requisite length of the
confirmation of imperfect title was filed on October possession of the predecessors-in-interest of the
21, 1996, the applicable law was Section 48(b) of applicant that would be tacked to his own. His personal
Commonwealth Act No. 141 (Public Land Act), as amended incompetence to attest to the possession of the
by Presidential Decree No. 1073; that, accordingly, property within the time required by law underscored
the required period of possession must be "since June the weakness of the evidence on possession,
12, 1945 or earlier," as stated in Republic v. particularly as it has not been denied that the
Doldol,11 a more stringent requirement the non- applicant had arrived in the Philippines only on
compliance with which was fatal to his November 28, 1987. Considering that the possession and
cause.12chanrobleslaw occupation of the property in question by Apolonio,
Jr. and his predecessors-in-interest were not shown in
Lastly, the Government points out that tax declarations the records to have been "since June 12, 1945, or
or tax receipts did not suffice to prove ownership of earlier," the application must be rejected.
land in fee simple; that although it was the State's
policy to encourage and promote distribution of We should stress that only the title of those who had
alienable public lands as an ideal of social justice, possessed and occupied alienable and disposable lands
stringent safeguards must be adopted and applied to of the public domain within the requisite period could
prevent the lands from going to the wrong hands; and be judicially confirmed. Indeed, alienable public land
that Apolonio, Jr.'s reliance on hearsay evidence held by a possessor, either personally or through his
showed his unfitness to own the land.13chanrobleslaw predecessors-in-interest, openly, continuously and
exclusively during the prescribed statutory period is
In response, Apolonio Jr. insists that he had duly converted to private property by the mere lapse or
established his lawful occupation of the land as owner completion of the period.16chanrobleslaw
in fee simple; that the Government did not timely
object to his testimony, and did not also controvert That the Government did not timely object to the
his evidence; that the property had been properly admission of the testimony of Apolonio, Jr., or of the
identified; and that the lower courts had observed the other evidence presented by him was of no consequence
legal safeguards and guidelines in granting his to the success of the application. If he had no
application for judicial confirmation of his ownership personal knowledge of the facts establishing the
in fee simple.14chanrobleslaw possession of property for the requisite period, no
Ruling of the Court court can give any value to his assertion, particularly
as it was conceded by him no less that he had no
We reverse. personal or direct competence to know the truth of his
assertion. It was one thing for the trial court to

39
Public Land V Govnt Land – Confirmation of Imperfect or incomplete title: Natural Resources Jennica Gyrl
Delfin
admit the evidence, but quite another to give it any This decision inevitably affects all untitled lands
worth for purposes of judicial adjudication. currently in possession of persons and entities other
than the Philippine government. The petition, while
WHEREFORE, the Court GRANTS the petition for review unremarkable as to the facts, was accepted by the
on certiorari; REVERSES and SETS ASIDE the decision Court en banc in order to provide definitive clarity
promulgated on September 30, 2004; DISMISSES the to the applicability and scope of original registration
application of respondent Apolonio Bautista, Jr. for proceedings under Sections 14(1) and 14(2) of the
the judicial confirmation of his imperfect title in Property Registration Decree. In doing so, the Court
LRC Case No. N-12-10-96; and ORDERSApolonio Bautista, confronts not only the relevant provisions of the
Jr. to pay the costs of suit. Public Land Act and the Civil Code, but also the
reality on the ground. The countrywide phenomenon of
SO ORDERED.chanRoblesvirtualLawlibrary untitled lands, as well as the problem of informal
settlement it has spawned, has unfortunately been
EN BANC treated with benign neglect. Yet our current laws are
hemmed in by their own circumscriptions in addressing
HEIRS OF MARIO MALABANAN, G.R. No. 179987 the phenomenon. Still, the duty on our part is
Petitioner, primarily to decide cases before us in accord with the
Present: Constitution and the legal principles that have
PUNO, C.J., developed our public land law, though our social
QUISUMBING, obligations dissuade us from casting a blind eye on
YNARES-SANTIAGO, the endemic problems.
CARPIO,
- versus - AUSTRIA-MARTINEZ, I.
CORONA,
CARPIO MORALES, On 20 February 1998, Mario Malabanan filed an
TINGA, application for land registration covering a parcel of
CHICO-NAZARIO, land identified as Lot 9864-A, Cad-452-D, Silang
VELASCO, JR., Cadastre,[2] situated in Barangay Tibig, Silang Cavite,
NACHURA, and consisting of 71,324 square meters. Malabanan
LEONARDO DE CASTRO, claimed that he had purchased the property from Eduardo
BRION, Velazco,[3] and that he and his predecessors-in-
REPUBLIC OF THE PHILIPPINES, PERALTA, and interest had been in open, notorious, and continuous
Respondent. BERSAMIN, JJ. adverse and peaceful possession of the land for more
than thirty (30) years.
Promulgated:
The application was raffled to the Regional Trial Court
April 29, 2009 of (RTC) Cavite-Tagaytay City, Branch 18. The Office
of the Solicitor General (OSG) duly designated the
x---------------------------------------------------- Assistant Provincial Prosecutor of Cavite, Jose
----------------------- x Velazco, Jr., to appear on behalf of the State.[4] Apart
from presenting documentary evidence, Malabanan
himself and his witness, Aristedes Velazco, testified
D E C I S I O N at the hearing. Velazco testified that the property
was originally belonged to a twenty-two hectare
TINGA, J.: property owned by his great-grandfather, Lino Velazco.
Lino had four sons Benedicto, Gregorio, Eduardo and
One main reason why the informal sector has not become Estebanthe fourth being Aristedess grandfather. Upon
formal is that from Indonesia to Brazil, 90 percent of Linos death, his four sons inherited the property and
the informal lands are not titled and registered. This divided it among themselves. But by 1966, Estebans
is a generalized phenomenon in the so-called Third wife, Magdalena, had become the administrator of all
World. And it has many consequences. the properties inherited by the Velazco sons from their
father, Lino. After the death of Esteban and Magdalena,
their son Virgilio succeeded them in administering the
xxx properties, including Lot 9864-A, which originally
belonged to his uncle, Eduardo Velazco. It was this
The question is: How is it that so many governments, property that was sold by Eduardo Velazco to
from Suharto's in Indonesia to Fujimori's in Peru, Malabanan.[5]
have wanted to title these people and have not been
able to do so effectively? One reason is that none of Assistant Provincial Prosecutor Jose Velazco, Jr. did
the state systems in Asia or Latin America can gather not cross-examine Aristedes Velazco. He further
proof of informal titles. In Peru, the informals have manifested that he also [knew] the property and I
means of proving property ownership to each other which affirm the truth of the testimony given by Mr.
are not the same means developed by the Spanish legal Velazco.[6] The Republic of the Philippines likewise
system. The informals have their own papers, their own did not present any evidence to controvert the
forms of agreements, and their own systems of application.
registration, all of which are very clearly stated in
the maps which they use for their own informal business Among the evidence presented by Malabanan during trial
transactions. was a Certification dated 11 June 2001, issued by the
If you take a walk through the countryside, Community Environment & Natural Resources Office,
from Indonesia to Peru, and you walk by field after Department of Environment and Natural Resources
field--in each field a different dog is going to bark (CENRO-DENR), which stated that the subject property
at you. Even dogs know what private property is all was verified to be within the Alienable or Disposable
about. The only one who does not know it is the land per Land Classification Map No. 3013 established
government. The issue is that there exists a "common under Project No. 20-A and approved as such under FAO
law" and an "informal law" which the Latin American 4-1656 on March 15, 1982.[7]
formal legal system does not know how to recognize.
- Hernando De Soto[1] On 3 December 2002, the RTC rendered judgment in favor
of Malabanan, the dispositive portion of which reads:

40
Public Land V Govnt Land – Confirmation of Imperfect or incomplete title: Natural Resources Jennica Gyrl
Delfin
WHEREFORE, this Court hereby approves this application classified as alienable and disposable as of June 12,
for registration and thus places under the operation 1945 or is it sufficient that such classification occur
of Act 141, Act 496 and/or P.D. 1529, otherwise known at any time prior to the filing of the applicant for
as Property Registration Law, the lands described in registration provided that it is established that the
Plan Csd-04-0173123-D, Lot 9864-A and containing an applicant has been in open, continuous, exclusive and
area of Seventy One Thousand Three Hundred Twenty Four notorious possession of the land under a bona
(71,324) Square Meters, as supported by its technical fide claim of ownership since June 12, 1945 or earlier?
description now forming part of the record of this
case, in addition to other proofs adduced in the name 2. For purposes of Section 14(2) of the Property
of MARIO MALABANAN, who is of legal age, Filipino, Registration Decree may a parcel of land classified as
widower, and with residence at Munting Ilog, Silang, alienable and disposable be deemed private land and
Cavite. therefore susceptible to acquisition by prescription
in accordance with the Civil Code?
Once this Decision becomes final and executory, the
corresponding decree of registration shall forthwith 3. May a parcel of land established as agricultural in
issue. character either because of its use or because its
slope is below that of forest lands be registrable
SO ORDERED. under Section 14(2) of the Property Registration Decree
in relation to the provisions of the Civil Code on
acquisitive prescription?
The Republic interposed an appeal to the Court of
Appeals, arguing that Malabanan had failed to prove 4. Are petitioners entitled to the registration of the
that the property belonged to the alienable and subject land in their names under Section 14(1) or
disposable land of the public domain, and that the RTC Section 14(2) of the Property Registration Decree or
had erred in finding that he had been in possession of both?[13]
the property in the manner and for the length of time
required by law for confirmation of imperfect title. Based on these issues, the parties formulated their
respective positions.
On 23 February 2007, the Court of Appeals rendered a
Decision[8] reversing the RTC and dismissing the With respect to Section 14(1), petitioners reiterate
application of Malabanan. The appellate court held that that the analysis of the Court in Naguit is the correct
under Section 14(1) of the Property Registration Decree interpretation of the provision. The seemingly
any period of possession prior to the classification contradictory pronouncement in Herbieto, it is
of the lots as alienable and disposable was submitted, should be considered obiter dictum, since
inconsequential and should be excluded from the the land registration proceedings therein was void ab
computation of the period of possession. Thus, the initio due to lack of publication of the notice of
appellate court noted that since the CENRO-DENR initial hearing. Petitioners further point out that
certification had verified that the property was in Republic v. Bibonia,[14] promulgated in June of 2007,
declared the Court applied Naguit and adopted the same
alienable and disposable only on 15 March 1982, the observation that the preferred interpretation by the
Velazcos possession prior to that date could not be OSG of Section 14(1) was patently absurd. For its part,
factored in the computation of the period of the OSG remains insistent that for Section 14(1) to
possession. This interpretation of the Court of Appeals apply, the land should have been classified as
of Section 14(1) of the Property Registration Decree alienable and disposable as of 12 June 1945. Apart
was based on the Courts ruling in Republic v. from Herbieto, the OSG also cites the subsequent
Herbieto.[9] rulings in Buenaventura v. Republic,[15] Fieldman
Agricultural Trading v. Republic[16] and Republic v.
Malabanan died while the case was pending with the Imperial Credit Corporation,[17] as well as the earlier
Court of Appeals;[10] hence, it was his heirs who case of Director of Lands v. Court of Appeals.[18]
appealed the decision of the appellate court.
Petitioners, before this Court, rely on our ruling With respect to Section 14(2), petitioners submit that
in Republic v. Naguit,[11] which was handed down just open, continuous, exclusive and notorious possession
four months prior to Herbieto. Petitioners suggest of an alienable land of the public domain for more than
that the discussion in Herbieto cited by the Court of 30 years ipso jure converts the land into private
Appeals is actually obiter dictum since the property, thus placing it under the coverage of Section
Metropolitan Trial Court therein which had directed 14(2). According to them, it would not matter whether
the registration of the property had no jurisdiction the land sought to be registered was previously
in the first place since the requisite notice of classified as agricultural land of the public domain
hearing was published only after the hearing had so long as, at the time of the application, the
already begun. Naguit, petitioners argue, remains the property had already been converted into private
controlling doctrine, especially when the property in property through prescription. To bolster their
question is agricultural land. Therefore, with respect argument, petitioners cite extensively from our 2008
to agricultural lands, any possession prior to the ruling in Republic v. T.A.N. Properties.[19]
declaration of the alienable property as disposable
may be counted in reckoning the period of possession
to perfect title under the Public Land Act and the The arguments submitted by the OSG with respect to
Property Registration Decree. Section 14(2) are more extensive. The OSG notes that
under Article 1113 of the Civil Code, the acquisitive
prescription of properties of the State refers to
patrimonial property, while Section 14(2) speaks of
The petition was referred to the Court en banc,[12] and private lands. It observes that the Court has yet to
on 11 November 2008, the case was heard on oral decide a case that presented Section 14(2) as a ground
arguments. The Court formulated the principal issues for application for registration, and that the 30-year
for the oral arguments, to wit: possession period refers to the period of possession
under Section 48(b) of the Public Land Act, and not
1. In order that an alienable and disposable land of the concept of prescription under the Civil Code. The
the public domain may be registered under Section 14(1) OSG further submits that, assuming that the 30-year
of Presidential Decree No. 1529, otherwise known as prescriptive period can run against public lands, said
the Property Registration Decree, should the land be

41
Public Land V Govnt Land – Confirmation of Imperfect or incomplete title: Natural Resources Jennica Gyrl
Delfin
period should be reckoned from the time the public land
was declared alienable and disposable. Second, the length of the requisite possession was
changed from possession for thirty (30) years
Both sides likewise offer special arguments with immediately preceding the filing of the application to
respect to the particular factual circumstances possession since June 12, 1945 or earlier. The Court
surrounding the subject property and the ownership in Naguit explained:
thereof.
When the Public Land Act was first promulgated in 1936,
II. the period of possession deemed necessary to vest the
right to register their title to agricultural lands of
First, we discuss Section 14(1) of the Property the public domain commenced from July 26, 1894.
Registration Decree. For a full understanding of the However, this period was amended by R.A. No. 1942,
provision, reference has to be made to the Public Land which provided that the bona fide claim of ownership
Act. must have been for at least thirty (30) years. Then in
1977, Section 48(b) of the Public Land Act was again
A. amended, this time by P.D. No. 1073, which pegged the
reckoning date at June 12, 1945. xxx
Commonwealth Act No. 141, also known as the Public Land
Act, has, since its enactment, governed the
classification and disposition of lands of the public It bears further observation that Section 48(b) of Com.
domain. The President is authorized, from time to time, Act No, 141 is virtually the same as Section 14(1) of
to classify the lands of the public domain into the Property Registration Decree. Said Decree codified
alienable and disposable, timber, or mineral the various laws relative to the registration of
lands.[20] Alienable and disposable lands of the public property, including lands of the public domain. It is
domain are further classified according to their uses Section 14(1) that operationalizes the registration of
into (a) agricultural; (b) residential, commercial, such lands of the public domain. The provision reads:
industrial, or for similar productive purposes; (c)
educational, charitable, or other similar purposes; or
(d) reservations for town sites and for public and SECTION 14. Who may apply. The following persons may
quasi-public uses.[21] file in the proper Court of First Instance an
application for registration of title to land, whether
May a private person validly seek the registration in personally or through their duly authorized
his/her name of alienable and disposable lands of the representatives:
public domain? Section 11 of the Public Land Act
acknowledges that public lands suitable for (1) those who by themselves or through their
agricultural purposes may be disposed of by predecessors-in-interest have been in open,
confirmation of imperfect or incomplete titles through continuous, exclusive and notorious possession and
[22]
judicial legalization. Section 48(b) of the Public occupation of alienable and disposable lands of the
Land Act, as amended by P.D. No. 1073, supplies the public domain under a bona fide claim of ownership
details and unmistakably grants that right, subject to since June 12, 1945, or earlier.
the requisites stated therein:

Sec. 48. The following described citizens of the Notwithstanding the passage of the Property
Philippines, occupying lands of the public domain or Registration Decree and the inclusion of Section 14(1)
claiming to own any such land or an interest therein, therein, the Public Land Act has remained in effect.
but whose titles have not been perfected or completed, Both laws commonly refer to persons or their
may apply to the Court of First Instance of the predecessors-in-interest who have been in open,
province where the land is located for confirmation of continuous, exclusive and notorious possession and
their claims and the issuance of a certificate of title occupation of alienable and disposable lands of the
therefor, under the Land Registration Act, to wit: public domain under a bona fide claim of ownership
since June 12, 1945, or earlier. That circumstance may
xxx have led to the impression that one or the other is a
redundancy, or that Section 48(b) of the Public Land
(b) Those who by themselves or through their Act has somehow been repealed or mooted. That is not
predecessors in interest have been in open, continuous, the case.
exclusive, and notorious possession and occupation of
alienable and disposable lands of the public domain, The opening clauses of Section 48 of the Public Land
under a bona fide claim of acquisition of ownership, Act and Section 14 of the Property Registration Decree
since June 12, 1945, or earlier, immediately preceding warrant comparison:
the filing of the application for confirmation of title
except when prevented by war or force majeure. These Sec. 48 [of the Public Land Act]. The following
shall be conclusively presumed to have performed all described citizens of the Philippines, occupying lands
the conditions essential to a Government grant and of the public domain or claiming to own any such land
shall be entitled to a certificate of title under the or an interest therein, but whose titles have not been
provisions of this chapter. perfected or completed, may apply to the Court of First
Instance of the province where the land is located for
confirmation of their claims and the issuance of a
Section 48(b) of Com. Act No. 141 received its present certificate of title therefor, under the Land
wording in 1977 when the law was amended by P.D. No. Registration Act, to wit:
1073. Two significant amendments were introduced by
P.D. No. 1073. First, the term agricultural lands was xxx
changed to alienable and disposable lands of the public
domain. The OSG submits that this amendment restricted Sec. 14 [of the Property Registration Decree]. Who may
the scope of the lands that may be registered.[23] This apply. The following persons may file in the proper
is not actually the case. Under Section 9 of the Public Court of First Instance an application for registration
Land Act, agricultural lands are a mere subset of lands of title to land, whether personally or through their
of the public domain alienable or open to disposition. duly authorized representatives:
Evidently, alienable and disposable lands of the public
domain are a larger class than only agricultural lands. xxx

42
Public Land V Govnt Land – Confirmation of Imperfect or incomplete title: Natural Resources Jennica Gyrl
Delfin

It is clear that Section 48 of the Public Land Act is Petitioner suggests an interpretation that the
more descriptive of the nature of the right enjoyed by alienable and disposable character of the land should
the possessor than Section 14 of the Property have already been established since June 12, 1945 or
Registration Decree, which seems to presume the pre- earlier. This is not borne out by the plain meaning of
existence of the right, rather than establishing the Section 14(1). Since June 12, 1945, as used in the
right itself for the first time. It is proper to assert provision, qualifies its antecedent phrase under a
that it is the Public Land Act, as amended by P.D. No. bonafide claim of ownership. Generally speaking,
1073 effective 25 January 1977, that has primarily qualifying words restrict or modify only the words or
established the right of a Filipino citizen who has phrases to which they are immediately associated, and
been in open, continuous, exclusive, and notorious not those distantly or remotely located.[25] Ad proximum
possession and occupation of alienable and disposable antecedents fiat relation nisi impediatur sentencia.
lands of the public domain, under a bona fide claim of
acquisition of ownership, since June 12, 1945 to Besides, we are mindful of the absurdity that would
perfect or complete his title by applying with the result if we adopt petitioners position. Absent a
proper court for the confirmation of his ownership legislative amendment, the rule would be, adopting the
claim and the issuance of the corresponding certificate OSGs view, that all lands of the public domain which
of title. were not declared alienable or disposable before June
12, 1945 would not be susceptible to original
registration, no matter the length of unchallenged
Section 48 can be viewed in conjunction with the afore- possession by the occupant. Such interpretation
quoted Section 11 of the Public Land Act, which renders paragraph (1) of Section 14 virtually
provides that public lands suitable for agricultural inoperative and even precludes the government from
purposes may be disposed of by confirmation of giving it effect even as it decides to reclassify
imperfect or incomplete titles, and given the notion public agricultural lands as alienable and
that both provisions declare that it is indeed the disposable. The unreasonableness of the situation
Public Land Act that primarily establishes the would even be aggravated considering that before June
substantive ownership of the possessor who has been in 12, 1945, the Philippines was not yet even considered
possession of the property since 12 June 1945. In turn, an independent state.
Section 14(a) of the Property Registration Decree
recognizes the substantive right granted under Section Accordingly, the Court in Naguit explained:
48(b) of the Public Land Act, as well provides the
corresponding original registration procedure for the [T]he more reasonable interpretation of Section 14(1)
judicial confirmation of an imperfect or incomplete is that it merely requires the property sought to be
title. registered as already alienable and disposable at the
time the application for registration of title is
There is another limitation to the right granted under filed. If the State, at the time the application is
Section 48(b). Section 47 of the Public Land Act limits made, has not yet deemed it proper to release the
the period within which one may exercise the right to property for alienation or disposition, the
seek registration under Section 48. The provision has presumption is that the government is still reserving
been amended several times, most recently by Rep. Act the right to utilize the property; hence, the need to
No. 9176 in 2002. It currently reads thus: preserve its ownership in the State irrespective of
the length of adverse possession even if in good faith.
Section 47. The persons specified in the next following However, if the property has already been classified
section are hereby granted time, not to extend beyond as alienable and disposable, as it is in this case,
December 31, 2020 within which to avail of the benefits then there is already an intention on the part of the
of this Chapter: Provided, That this period shall apply State to abdicate its exclusive prerogative over the
only where the area applied for does not exceed twelve property.
(12) hectares: Provided, further, That the several
periods of time designated by the President in
accordance with Section Forty-Five of this Act shall The Court declares that the correct interpretation of
apply also to the lands comprised in the provisions of Section 14(1) is that which was adopted in Naguit. The
this Chapter, but this Section shall not be construed contrary pronouncement in Herbieto, as pointed out
as prohibiting any said persons from acting under this in Naguit, absurdly limits the application of the
Chapter at any time prior to the period fixed by the provision to the point of virtual inutility since it
President.[24] would only cover lands actually declared alienable and
disposable prior to 12 June 1945, even if the current
Accordingly under the current state of the law, the possessor is able to establish open, continuous,
substantive right granted under Section 48(b) may be exclusive and notorious possession under a bona
availed of only until 31 December 2020. fide claim of ownership long before that date.

B. Moreover, the Naguit interpretation allows more


possessors under a bona fide claim of ownership to
Despite the clear text of Section 48(b) of the Public avail of judicial confirmation of their imperfect
Land Act, as amended and Section 14(a) of the Property titles than what would be feasible under Herbieto. This
Registration Decree, the OSG has adopted the position balancing fact is significant, especially considering
that for one to acquire the right to seek registration our forthcoming discussion on the scope and reach of
of an alienable and disposable land of the public Section 14(2) of the Property Registration Decree.
domain, it is not enough that the applicant and his/her
predecessors-in-interest be in possession under a bona Petitioners make the salient observation that the
fide claim of ownership since 12 June 1945; the contradictory passages from Herbieto are obiter
alienable and disposable character of the property must dicta since the land registration proceedings therein
have been declared also as of 12 June 1945. Following is void ab initio in the first place due to lack of
the OSGs approach, all lands certified as alienable the requisite publication of the notice of initial
and disposable after 12 June 1945 cannot be registered hearing. There is no need to explicitly
either under Section 14(1) of the Property Registration overturn Herbieto, as it suffices that the Courts
Decree or Section 48(b) of the Public Land Act as acknowledgment that the particular line of argument
amended. The absurdity of such an implication was used therein concerning Section 14(1) is
discussed in Naguit. indeed obiter.

43
Public Land V Govnt Land – Confirmation of Imperfect or incomplete title: Natural Resources Jennica Gyrl
Delfin
It may be noted that in the subsequent case Thus, while the Court of Appeals erred in ruling that
of Buenaventura,[26] the Court, citing Herbieto, again mere possession of public land for the period required
stated that [a]ny period of possession prior to the by law would entitle its occupant to a confirmation of
date when the [s]ubject [property was] classified as imperfect title, it did not err in ruling in favor of
alienable and disposable is inconsequential and should private respondents as far as the first requirement in
be excluded from the computation of the period of Section 48(b) of the Public Land Act is concerned, for
possession That statement, in the context of Section they were able to overcome the burden of proving the
14(1), is certainly erroneous. Nonetheless, the alienability of the land subject of their application.
passage as cited in Buenaventura should again be
considered as obiter. The application therein was As correctly found by the Court of Appeals, private
ultimately granted, citing Section 14(2). The evidence respondents were able to prove their open, continuous,
submitted by petitioners therein did not establish any exclusive and notorious possession of the subject land
mode of possession on their part prior to 1948, thereby even before the year 1927. As a rule, we are bound by
precluding the application of Section 14(1). It is not the factual findings of the Court of Appeals. Although
even apparent from the decision whether petitioners there are exceptions, petitioner did not show that this
therein had claimed entitlement to original is one of them.[29]
registration following Section 14(1), their position
being that they had been in exclusive possession under Why did the Court in Ceniza, through the same eminent
a bona fide claim of ownership for over fifty (50) member who authored Bracewell, sanction the
years, but not before 12 June 1945. registration under Section 48(b) of public domain lands
Thus, neither Herbieto nor its principal discipular declared alienable or disposable thirty-five (35)
ruling Buenaventura has any precedental value with years and 180 days after 12 June 1945? The telling
respect to Section 14(1). On the other hand, the ratio difference is that in Ceniza, the application for
of Naguit is embedded in Section 14(1), since it registration was filed nearly six (6) years after the
precisely involved situation wherein the applicant had land had been declared alienable or disposable, while
been in exclusive possession under a bona fide claim in Bracewell, the application was filed nine (9)
of ownership prior to 12 June 1945. The Courts years before the land was declared alienable or
interpretation of Section 14(1) therein was decisive disposable. That crucial difference was also stressed
to the resolution of the case. Any doubt as to which in Naguit to contradistinguish it from Bracewell, a
between Naguit or Herbieto provides the final word of difference which the dissent seeks to belittle.
the Court on Section 14(1) is now settled in favor
of Naguit. III.

We noted in Naguit that it should be distinguished We next ascertain the correct framework of analysis
from Bracewell v. Court of Appeals[27] since in the with respect to Section 14(2). The provision reads:
latter, the application for registration had been
filed before the land was declared alienable or
disposable. The dissent though pronounces Bracewell as
the better rule between the two. Yet two years SECTION 14. Who may apply. The following persons may
after Bracewell, its ponente, the esteemed Justice file in the proper Court of First Instance an
Consuelo Ynares-Santiago, penned the ruling application for registration of title to land, whether
in Republic v. Ceniza,[28] which involved a claim of personally or through their duly authorized
possession that extended back to 1927 over a public representatives:
domain land that was declared alienable and disposable
only in 1980. Ceniza cited Bracewell, quoted xxx
extensively from it, and following the mindset of the
dissent, the attempt at registration in Cenizashould (2) Those who have acquired ownership over
have failed. Not so. private lands by prescription under the provisions of
existing laws.
To prove that the land subject of an application for
registration is alienable, an applicant must establish
the existence of a positive act of the government such The Court in Naguit offered the following discussion
as a presidential proclamation or an executive order; concerning Section 14(2), which we did even then
an administrative action; investigation reports of recognize, and still do, to be an obiter dictum, but
Bureau of Lands investigators; and a legislative act we nonetheless refer to it as material for further
or a statute. discussion, thus:

In this case, private respondents presented a Did the enactment of the Property Registration Decree
certification dated November 25, 1994, issued by and the amendatory P.D. No. 1073 preclude the
Eduardo M. Inting, the Community Environment and application for registration of alienable lands of the
Natural Resources Officer in the Department of public domain, possession over which commenced only
Environment and Natural Resources Office in Cebu City, after June 12, 1945? It did not, considering Section
stating that the lots involved were "found to be within 14(2) of the Property Registration Decree, which
the alienable and disposable (sic) Block-I, Land governs and authorizes the application of those who
Classification Project No. 32-A, per map 2962 4-I555 have acquired ownership of private lands by
dated December 9, 1980." This is sufficient evidence prescription under the provisions of existing laws.
to show the real character of the land subject of
private respondents application. Further, the Prescription is one of the modes of acquiring ownership
certification enjoys a presumption of regularity in under the Civil Code.[[30]] There is a consistent
the absence of contradictory evidence, which is true jurisprudential rule that properties classified as
in this case. Worth noting also was the observation of alienable public land may be converted into private
the Court of Appeals stating that: property by reason of open, continuous and exclusive
possession of at least thirty (30) years.[[31]] With
[n]o opposition was filed by the Bureaus of Lands and such conversion, such property may now fall within the
Forestry to contest the application of appellees on contemplation of private lands under Section 14(2),
the ground that the property still forms part of the and thus susceptible to registration by those who have
public domain. Nor is there any showing that the lots acquired ownership through prescription. Thus, even if
in question are forestal land.... possession of the alienable public land commenced on a
date later than June 12, 1945, and such possession

44
Public Land V Govnt Land – Confirmation of Imperfect or incomplete title: Natural Resources Jennica Gyrl
Delfin
being been open, continuous and exclusive, then the own any such lands or an interest therein, but whose
possessor may have the right to register the land by titles have not been perfected or completed, may apply
virtue of Section 14(2) of the Property Registration to the Court of First Instance of the province where
Decree. the land is located for confirmation of their claims
and the issuance of a certificate of title therefor,
Naguit did not involve the application of Section under the Land Registration Act, to wit:
14(2), unlike in this case where petitioners have based
their registration bid primarily on that provision, x x x x x x x x x
and where the evidence definitively establishes their
claim of possession only as far back as 1948. It is in (b) Those who by themselves or through their
this case that we can properly appreciate the nuances predecessors in interest have been in open, continuous,
of the provision. exclusive and notorious possession and occupation of
agricultural lands of the public domain, under a bona
A. fide claim of acquisition of ownership, for at least
thirty years immediately preceding the filing of the
The obiter in Naguit cited the Civil Code provisions application for confirmation of title, except when
on prescription as the possible basis for application prevented by war or force majeure. These shall be
for original registration under Section 14(2). conclusively presumed to have performed all the
Specifically, it is Article 1113 which provides legal conditions essential to a Government grant and shall
foundation for the application. It reads: be entitled to a certificate of title under the
provisions of this Chapter. (emphasis supplied)[37]
All things which are within the commerce of men are
susceptible of prescription, unless otherwise
provided. Property of the State or any of its This provision was repealed in 1977 with the enactment
subdivisions not patrimonial in character shall not be of P.D. 1073, which made the date 12 June 1945 the
the object of prescription. reckoning point for the first time. Nonetheless,
applications for registration filed prior to 1977 could
have invoked the 30-year rule introduced by Rep. Act
It is clear under the Civil Code that where lands of No. 1942.
the public domain are patrimonial in character, they
are susceptible to acquisitive prescription. On the The second source is Section 14(2) of P.D. 1529 itself,
other hand, among the public domain lands that are not at least by implication, as it applies the rules on
susceptible to acquisitive prescription are timber prescription under the Civil Code, particularly
lands and mineral lands. The Constitution itself Article 1113 in relation to Article 1137. Note that
proscribes private ownership of timber or mineral there are two kinds of prescription under the Civil
lands. Codeordinary acquisitive prescription and
extraordinary acquisitive prescription, which, under
There are in fact several provisions in the Civil Code Article 1137, is completed through uninterrupted
concerning the acquisition of real property through adverse possession for thirty years, without need of
prescription. Ownership of real property may be title or of good faith.
acquired by ordinary prescription of ten (10)
years,[32] or through extraordinary prescription of Obviously, the first source of the thirty (30)-year
thirty (30) years.[33] Ordinary acquisitive period rule, Rep. Act No. 1942, became unavailable
prescription requires possession in good faith,[34]as after 1977. At present, the only legal basis for the
well as just title.[35] thirty (30)-year period is the law on prescription
under the Civil Code, as mandated under Section 14(2).
When Section 14(2) of the Property Registration Decree However, there is a material difference between how
explicitly provides that persons who have acquired the thirty (30)-year rule operated under Rep. Act No.
ownership over private lands by prescription under the 1942 and how it did under the Civil Code.
provisions of existing laws, it unmistakably refers to
the Civil Code as a valid basis for the registration Section 48(b) of the Public Land Act, as amended by
of lands. The Civil Code is the only existing law that Rep. Act No. 1942, did not refer to or call into
specifically allows the acquisition by prescription of application the Civil Code provisions on prescription.
private lands, including patrimonial property It merely set forth a requisite thirty-year possession
belonging to the State. Thus, the critical question period immediately preceding the application for
that needs affirmation is whether Section 14(2) does confirmation of title, without any qualification as to
encompass original registration proceedings over whether the property should be declared alienable at
patrimonial property of the State, which a private the beginning of, and continue as such, throughout the
person has acquired through prescription. entire thirty-(30) years. There is neither statutory
nor jurisprudential basis to assert Rep. Act No. 1942
The Naguit obiter had adverted to a frequently had mandated such a requirement,[38] similar to our
reiterated jurisprudence holding that properties earlier finding with respect to the present language
classified as alienable public land may be converted of Section 48(b), which now sets 12 June 1945 as the
into private property by reason of open, continuous point of reference.
and exclusive possession of at least thirty (30)
years.[36] Yet if we ascertain the source of the thirty- Then, with the repeal of Rep. Act No. 1942, the thirty-
year period, additional complexities relating to year possession period as basis for original
Section 14(2) and to how exactly it operates would registration became Section 14(2) of the Property
emerge. For there are in fact two distinct origins of Registration Decree, which entitled those who have
the thirty (30)-year rule. acquired ownership over private lands by prescription
under the provisions of existing laws to apply for
The first source is Rep. Act No. 1942, enacted in 1957, original registration. Again, the thirty-year period
which amended Section 48(b) of the Public Land Act by is derived from the rule on extraordinary prescription
granting the right to seek original registration of under Article 1137 of the Civil Code. At the same time,
alienable public lands through possession in the Section 14(2) puts into operation the entire regime of
concept of an owner for at least thirty years. prescription under the Civil Code, a fact which does
not hold true with respect to Section 14(1).
The following-described citizens of the Philippines,
occupying lands of the public domain or claiming to B.

45
Public Land V Govnt Land – Confirmation of Imperfect or incomplete title: Natural Resources Jennica Gyrl
Delfin
incapable of acquisition by prescription. It is only
Unlike Section 14(1), Section 14(2) explicitly refers when such alienable and disposable lands are expressly
to the principles on prescription under existing laws. declared by the State to be no longer intended for
Accordingly, we are impelled to apply the civil law public service or for the development of the national
concept of prescription, as set forth in the Civil wealth that the period of acquisitive prescription can
Code, in our interpretation of Section 14(2). There is begin to run. Such declaration shall be in the form of
no similar demand on our part in the case of Section a law duly enacted by Congress or a Presidential
14(1). Proclamation in cases where the President is duly
authorized by law.
The critical qualification under Article 1113 of the
Civil Code is thus: [p]roperty of the State or any of It is comprehensible with ease that this reading of
its subdivisions not patrimonial in character shall Section 14(2) of the Property Registration Decree
not be the object of prescription. The identification limits its scope and reach and thus affects the
what consists of patrimonial property is provided by registrability even of lands already declared
Articles 420 and 421, which we quote in full: alienable and disposable to the detriment of the bona
fide possessors or occupants claiming title to the
Art. 420. The following things are property of public lands. Yet this interpretation is in accord with the
dominion: Regalian doctrine and its concomitant assumption that
all lands owned by the State, although declared
(1) Those intended for public use, such as roads, alienable or disposable, remain as such and ought to
canals, rivers, torrents, ports and bridges be used only by the Government.
constructed by the State, banks, shores, roadsteads,
and others of similar character; Recourse does not lie with this Court in the matter.
The duty of the Court is to apply the Constitution and
(2) Those which belong to the State, without being for the laws in accordance with their language and intent.
public use, and are intended for some public service The remedy is to change the law, which is the province
or for the development of the national wealth. of the legislative branch. Congress can very well be
entreated to amend Section 14(2) of the Property
Art. 421. All other property of the State, which is Registration Decree and pertinent provisions of the
not of the character stated in the preceding article, Civil Code to liberalize the requirements for judicial
is patrimonial property confirmation of imperfect or incomplete titles.

It is clear that property of public dominion, which The operation of the foregoing interpretation can be
generally includes property belonging to the State, illustrated by an actual example. Republic Act No.
cannot be the object of prescription or, indeed, be 7227, entitled An Act Accelerating The Conversion Of
subject of the commerce of man.[39] Lands of the public Military Reservations Into Other Productive Uses,
domain, whether declared alienable and disposable or etc., is more commonly known as the BCDA law. Section
not, are property of public dominion and thus 2 of the law authorizes the sale of certain military
insusceptible to acquisition by prescription. reservations and portions of military camps in Metro
Manila, including Fort Bonifacio and Villamor Air
Let us now explore the effects under the Civil Code of Base. For purposes of effecting the sale of the
a declaration by the President or any duly authorized military camps, the law mandates the President to
government officer of alienability and disposability transfer such military lands to the Bases Conversion
of lands of the public domain. Would such lands so Development Authority (BCDA)[40] which in turn is
declared alienable and disposable be converted, under authorized to own, hold and/or administer them.[41] The
the Civil Code, from property of the public dominion President is authorized to sell portions of the
into patrimonial property? After all, by connotative military camps, in whole or in part.[42] Accordingly,
definition, alienable and disposable lands may be the the BCDA law itself declares that the military lands
object of the commerce of man; Article 1113 provides subject thereof are alienable and disposable pursuant
that all things within the commerce of man are to the provisions of existing laws and regulations
susceptible to prescription; and the same provision governing sales of government properties.[43]
further provides that patrimonial property of the State
may be acquired by prescription. From the moment the BCDA law was enacted the subject
military lands have become alienable and disposable.
Nonetheless, Article 422 of the Civil Code states that However, said lands did not become patrimonial, as the
[p]roperty of public dominion, when no longer intended BCDA law itself expressly makes the reservation that
for public use or for public service, shall form part these lands are to be sold in order to raise funds for
of the patrimonial property of the State. It is this the conversion of the former American bases
provision that controls how public dominion property at Clark and Subic.[44]Such purpose can be tied to
may be converted into patrimonial property susceptible either public service or the development of national
to acquisition by prescription. After all, Article 420 wealth under Article 420(2). Thus, at that time, the
(2) makes clear that those property which belong to lands remained property of the public dominion under
the State, without being for public use, and are Article 420(2), notwithstanding their status as
intended for some public service or for the development alienable and disposable. It is upon their sale as
of the national wealth are public dominion property. authorized under the BCDA law to a private person or
For as long as the property belongs to the State, entity that such lands become private property and
although already classified as alienable or cease to be property of the public dominion.
disposable, it remains property of the public dominion
if when it is intended for some public service or for C.
the development of the national wealth.
Should public domain lands become patrimonial because
Accordingly, there must be an express declaration by they are declared as such in a duly enacted law or duly
the State that the public dominion property is no promulgated proclamation that they are no longer
longer intended for public service or the development intended for public service or for the development of
of the national wealth or that the property has been the national wealth, would the period of possession
converted into patrimonial. Without such express prior to the conversion of such public dominion into
declaration, the property, even if classified as patrimonial be reckoned in counting the prescriptive
alienable or disposable, remains property of the public period in favor of the possessors? We rule in the
dominion, pursuant to Article 420(2), and thus negative.

46
Public Land V Govnt Land – Confirmation of Imperfect or incomplete title: Natural Resources Jennica Gyrl
Delfin
acquiring patrimonial property of the State through
The limitation imposed by Article 1113 dissuades us ordinary acquisitive prescription, nor is there any
from ruling that the period of possession before the apparent reason to impose such a rule. At the same
public domain land becomes patrimonial may be counted time, there are indispensable requisitesgood faith and
for the purpose of completing the prescriptive period. just title. The ascertainment of good faith involves
Possession of public dominion property before it the application of Articles 526, 527, and 528, as well
becomes patrimonial cannot be the object of as Article 1127 of the Civil Code,[45] provisions that
prescription according to the Civil Code. As the more or less speak for themselves.
application for registration under Section 14(2) falls
wholly within the framework of prescription under the On the other hand, the concept of just title requires
Civil Code, there is no way that possession during the some clarification. Under Article 1129, there is just
time that the land was still classified as public title for the purposes of prescription when the adverse
dominion property can be counted to meet the requisites claimant came into possession of the property through
of acquisitive prescription and justify registration. one of the modes recognized by law for the acquisition
of ownership or other real rights, but the grantor was
Are we being inconsistent in applying divergent rules not the owner or could not transmit any right. Dr.
for Section 14(1) and Section 14(2)? There is no Tolentino explains:
inconsistency. Section 14(1) mandates registration on
the basis of possession, while Section 14(2) entitles Just title is an act which has for its purpose the
registration on the basis transmission of ownership, and which would have
of prescription. Registration under Section 14(1) is actually transferred ownership if the grantor had been
extended under the aegis of the Property Registration the owner. This vice or defect is the one cured by
Decree and the Public Land Act while registration under prescription. Examples: sale with delivery, exchange,
Section 14(2) is made available both by the Property donation, succession, and dacion in payment.[46]
Registration Decree and the Civil Code. The OSG submits that the requirement of just title
necessarily precludes the applicability of ordinary
In the same manner, we can distinguish between the acquisitive prescription to patrimonial property. The
thirty-year period under Section 48(b) of the Public major premise for the argument is that the State, as
Land Act, as amended by Rep. Act No. 1472, and the the owner and grantor, could not transmit ownership to
thirty-year period available through Section 14(2) of the possessor before the completion of the required
the Property Registration Decree in relation to Article period of possession.[47] It is evident that the OSG
1137 of the Civil Code. The period under the former erred when it assumed that the grantor referred to in
speaks of a thirty-year period of possession, while Article 1129 is the State. The grantor is the one from
the period under the latter concerns a thirty-year whom the person invoking ordinary acquisitive
period of extraordinary prescription. Registration prescription derived the title, whether by sale,
under Section 48(b) of the Public Land Act as amended exchange, donation, succession or any other mode of
by Rep. Act No. 1472 is based on thirty years of the acquisition of ownership or other real rights.
possession alone without regard to the Civil Code,
while the registration under Section 14(2) of the Earlier, we made it clear that, whether under ordinary
Property Registration Decree is founded on prescription or extraordinary prescription, the period
extraordinary prescription under the Civil Code. of possession preceding the classification of public
dominion lands as patrimonial cannot be counted for
It may be asked why the principles of prescription the purpose of computing prescription. But after the
under the Civil Code should not apply as well to property has been become patrimonial, the period of
Section 14(1). Notwithstanding the vaunted status of prescription begins to run in favor of the possessor.
the Civil Code, it ultimately is just one of numerous Once the requisite period has been completed, two legal
statutes, neither superior nor inferior to other events ensue: (1) the patrimonial property is ipso
statutes such as the Property Registration Decree. The jure converted into private land; and (2) the person
legislative branch is not bound to adhere to the in possession for the periods prescribed under the
framework set forth by the Civil Code when it enacts Civil Code acquires ownership of the property by
subsequent legislation. Section 14(2) manifests a operation of the Civil Code.
clear intent to interrelate the registration allowed
under that provision with the Civil Code, but no such It is evident that once the possessor automatically
intent exists with respect to Section 14(1). becomes the owner of the converted patrimonial
property, the ideal next step is the registration of
IV. the property under the Torrens system. It should be
remembered that registration of property is not a mode
One of the keys to understanding the framework we set of acquisition of ownership, but merely a mode of
forth today is seeing how our land registration confirmation of ownership.[48]
procedures correlate with our law on prescription,
which, under the Civil Code, is one of the modes for Looking back at the registration regime prior to the
acquiring ownership over property. adoption of the Property Registration Decree in 1977,
it is apparent that the registration system then did
The Civil Code makes it clear that patrimonial property not fully accommodate the acquisition of ownership of
of the State may be acquired by private persons through patrimonial property under the Civil Code. What the
prescription. This is brought about by Article 1113, system accommodated was the confirmation of imperfect
which states that [a]ll things which are within the title brought about by the completion of a period of
commerce of man are susceptible to prescription, and possession ordained under the Public Land Act (either
that [p]roperty of the State or any of its subdivisions 30 years following Rep. Act No. 1942, or since 12 June
not patrimonial in character shall not be the object 1945 following P.D. No. 1073).
of prescription.
The Land Registration Act[49] was noticeably silent on
There are two modes of prescription through which the requisites for alienable public lands acquired
immovables may be acquired under the Civil Code. The through ordinary prescription under the Civil Code,
first is ordinary acquisitive prescription, which, though it arguably did not preclude such
under Article 1117, requires possession in good faith registration.[50] Still, the gap was lamentable,
and with just title; and, under Article 1134, is considering that the Civil Code, by itself, establishes
completed through possession of ten (10) years. There ownership over the patrimonial property of persons who
is nothing in the Civil Code that bars a person from have completed the prescriptive periods ordained

47
Public Land V Govnt Land – Confirmation of Imperfect or incomplete title: Natural Resources Jennica Gyrl
Delfin
therein. The gap was finally closed with the adoption of the Public Land Act. There is no substantive
of the Property Registration Decree in 1977, with evidence to establish that Malabanan or petitioners as
Section 14(2) thereof expressly authorizing original his predecessors-in-interest have been in possession
registration in favor of persons who have acquired of the property since 12 June 1945 or earlier. The
ownership over private lands by prescription under the earliest that petitioners can date back their
provisions of existing laws, that is, the Civil Code possession, according to their own evidencethe Tax
as of now. Declarations they presented in particularis to the year
1948. Thus, they cannot avail themselves of
V. registration under Section 14(1) of the Property
Registration Decree.
We synthesize the doctrines laid down in this case, as
follows:
Neither can petitioners properly invoke Section 14(2)
(1) In connection with Section 14(1) of the Property as basis for registration. While the subject property
Registration Decree, Section 48(b) of the Public Land was declared as alienable or disposable in 1982, there
Act recognizes and confirms that those who by is no competent evidence that is no longer intended
themselves or through their predecessors in interest for public use service or for the development of the
have been in open, continuous, exclusive, and notorious national evidence, conformably with Article 422 of the
possession and occupation of alienable and disposable Civil Code. The classification of the subject property
lands of the public domain, under a bona fide claim of as alienable and disposable land of the public domain
acquisition of ownership, since June 12, 1945 have does not change its status as property of the public
acquired ownership of, and registrable title to, such dominion under Article 420(2) of the Civil Code.Thus,
lands based on the length and quality of their it is insusceptible to acquisition by prescription.
possession.
VI.
(a) Since Section 48(b) merely requires possession
since 12 June 1945 and does not require that the lands A final word. The Court is comfortable with the
should have been alienable and disposable during the correctness of the legal doctrines established in this
entire period of possession, the possessor is entitled decision. Nonetheless, discomfiture over the
to secure judicial confirmation of his title thereto implications of todays ruling cannot be discounted.
as soon as it is declared alienable and disposable, For, every untitled property that is occupied in the
subject to the timeframe imposed by Section 47 of the country will be affected by this ruling. The social
Public Land Act.[51] implications cannot be dismissed lightly, and the Court
would be abdicating its social responsibility to the
(b) The right to register granted under Section 48(b) Filipino people if we simply levied the law without
of the Public Land Act is further confirmed by Section comment.
14(1) of the Property Registration Decree.
The informal settlement of public lands, whether
(2) In complying with Section 14(2) of the Property declared alienable or not, is a phenomenon tied to
Registration Decree, consider that under the Civil long-standing habit and cultural acquiescence, and is
Code, prescription is recognized as a mode of acquiring common among the so-called Third World countries. This
ownership of patrimonial property. However, public paradigm powerfully evokes the disconnect between a
domain lands become only patrimonial property not only legal system and the reality on the ground. The law so
with a declaration that these are alienable or far has been unable to bridge that gap. Alternative
disposable. There must also be an express government means of acquisition of these
manifestation that the property is already patrimonial public domain lands, such as through homestead or
or no longer retained for public service or the free patent, have
development of national wealth, under Article 422 of
the Civil Code. And only when the property has become
patrimonial can the prescriptive period for the
acquisition of property of the public dominion begin proven unattractive due to limitations imposed on the
to run. grantee in the encumbrance or alienation of said
properties.[52] Judicial confirmation of imperfect
(a) Patrimonial property is private property of the title has emerged as the most viable, if not the most
government. The person acquires ownership of attractive means to regularize the informal settlement
patrimonial property by prescription under the Civil of alienable or disposable lands of the public domain,
Code is entitled to secure registration thereof under yet even that system, as revealed in this decision,
Section 14(2) of the Property Registration Decree. has considerable limits.

(b) There are two kinds of prescription by which There are millions upon millions of Filipinos who have
patrimonial property may be acquired, one ordinary and individually or exclusively held residential lands on
other extraordinary. Under ordinary acquisitive which they have lived and raised their families. Many
prescription, a person acquires ownership of a more have tilled and made productive idle lands of the
patrimonial property through possession for at least State with their hands. They have been regarded for
ten (10) years, in good faith and with just title. generation by their families and their communities as
Under extraordinary acquisitive prescription, a common law owners. There is much to be said about the
persons uninterrupted adverse possession of virtues of according them legitimate states. Yet such
patrimonial property for at least thirty (30) years, virtues are not for the Court to translate into
regardless of good faith or just title, ripens into positive law, as the law itself considered such lands
ownership. as property of the public dominion. It could only be
up to Congress to set forth a new phase of land reform
B. to sensibly regularize and formalize the settlement of
such lands which in legal theory are lands of the
We now apply the above-stated doctrines to the case at public domain before the problem becomes insoluble.
bar. This could be accomplished, to cite two examples, by
liberalizing the standards for judicial confirmation
It is clear that the evidence of petitioners is of imperfect title, or amending the Civil Code itself
insufficient to establish that Malabanan has acquired to ease the requisites for the conversion of public
ownership over the subject property under Section 48(b) dominion property into patrimonial.

48
Public Land V Govnt Land – Confirmation of Imperfect or incomplete title: Natural Resources Jennica Gyrl
Delfin
widower, and with residence at Munting Ilog, Silang,
Ones sense of security over land rights infuses into Cavite.
every aspect of well-being not only of that individual, Once this Decision becomes final and executory, the
but also to the persons family. Once that sense of corresponding decree of registration shall forthwith
security is deprived, life and livelihood are put on issue.
stasis. It is for the political branches to bring SO ORDERED.3
welcome closure to the long pestering problem. The Office of the Solicitor General (OSG) appealed the
judgment to the CA, arguing that Malabanan had failed
WHEREFORE, the Petition is DENIED. The Decision of the to prove that the property belonged to the alienable
Court of Appeals dated 23 February 2007 and Resolution and disposable land of the public domain, and that the
dated 2 October 2007 are AFFIRMED. No pronouncement as RTC erred in finding that he had been in possession of
to costs. the property in the manner and for the length of time
required by law for confirmation of imperfect title.
SO ORDERED. On February 23, 2007, the CA promulgated its decision
reversing the RTC and dismissing the application for
registration of Malabanan. Citing the ruling in
G.R. No. 179987 September 3, 2013 Republic v. Herbieto (Herbieto),4 the CA declared that
HEIRS OF MARIO MALABANAN, (Represented by Sally A. under Section 14(1) of the Property Registration
Malabanan), Petitioners, Decree, any period of possession prior to the
vs. classification of the land as alienable and disposable
REPUBLIC OF THE PHILIPPINES, Respondent. was inconsequential and should be excluded from the
R E S O L U T I O N computation of the period of possession. Noting that
BERSAMIN, J.: the CENRO-DENR certification stated that the property
For our consideration and resolution are the motions had been declared alienable and disposable only on
for reconsideration of the parties who both assail the March 15, 1982, Velazco’s possession prior to March
decision promulgated on April 29, 2009, whereby we 15, 1982 could not be tacked for purposes of computing
upheld the ruling of the Court of Appeals (CA) denying Malabanan’s period of possession.
the application of the petitioners for the registration Due to Malabanan’s intervening demise during the appeal
of a parcel of land situated in Barangay Tibig, Silang, in the CA, his heirs elevated the CA’s decision of
Cavite on the ground that they had not established by February 23, 2007 to this Court through a petition for
sufficient evidence their right to the registration in review on certiorari.
accordance with either Section 14(1) or Section 14(2) The petitioners assert that the ruling in Republic v.
of Presidential Decree No. 1529 (Property Registration Court of Appeals and Corazon Naguit5 (Naguit) remains
Decree). the controlling doctrine especially if the property
Antecedents involved is agricultural land. In this regard, Naguit
The property subject of the application for ruled that any possession of agricultural land prior
registration is a parcel of land situated in Barangay to its declaration as alienable and disposable could
Tibig, Silang Cavite, more particularly identified as be counted in the reckoning of the period of possession
Lot 9864-A, Cad-452-D, with an area of 71,324-square to perfect title under the Public Land Act
meters. On February 20, 1998, applicant Mario (Commonwealth Act No. 141) and the Property
Malabanan, who had purchased the property from Eduardo Registration Decree. They point out that the ruling in
Velazco, filed an application for land registration Herbieto, to the effect that the declaration of the
covering the property in the Regional Trial Court (RTC) land subject of the application for registration as
in Tagaytay City, Cavite, claiming that the property alienable and disposable should also date back to June
formed part of the alienable and disposable land of 12, 1945 or earlier, was a mere obiter dictum
the public domain, and that he and his predecessors- considering that the land registration proceedings
in-interest had been in open, continuous, therein were in fact found and declared void ab initio
uninterrupted, public and adverse possession and for lack of publication of the notice of initial
occupation of the land for more than 30 years, thereby hearing.
entitling him to the judicial confirmation of his The petitioners also rely on the ruling in Republic v.
title.1 T.A.N. Properties, Inc.6 to support their argument that
To prove that the property was an alienable and the property had been ipso jure converted into private
disposable land of the public domain, Malabanan property by reason of the open, continuous, exclusive
presented during trial a certification dated June 11, and notorious possession by their predecessors-in-
2001 issued by the Community Environment and Natural interest of an alienable land of the public domain for
Resources Office (CENRO) of the Department of more than 30 years. According to them, what was
Environment and Natural Resources (DENR), which reads: essential was that the property had been "converted"
This is to certify that the parcel of land designated into private property through prescription at the time
as Lot No. 9864 Cad 452-D, Silang Cadastre as surveyed of the application without regard to whether the
for Mr. Virgilio Velasco located at Barangay Tibig, property sought to be registered was previously
Silang, Cavite containing an area of 249,734 sq. meters classified as agricultural land of the public domain.
as shown and described on the Plan Ap-04-00952 is As earlier stated, we denied the petition for review
verified to be within the Alienable or Disposable land on certiorari because Malabanan failed to establish by
per Land Classification Map No. 3013 established under sufficient evidence possession and occupation of the
Project No. 20-A and approved as such under FAO 4-1656 property on his part and on the part of his
on March 15, 1982.2 predecessors-in interest since June 12, 1945, or
After trial, on December 3, 2002, the RTC rendered earlier.
judgment granting Malabanan’s application for land Petitioners’ Motion for Reconsideration
registration, disposing thusly: In their motion for reconsideration, the petitioners
WHEREFORE, this Court hereby approves this application submit that the mere classification of the land as
for registration and thus places under the operation alienable or disposable should be deemed sufficient to
of Act 141, Act 496 and/or P.D. 1529, otherwise known convert it into patrimonial property of the State.
as Property Registration Law, the lands described in Relying on the rulings in Spouses De Ocampo v.
Plan Csd-04-0173123-D, Lot 9864-A and containing an Arlos,7 Menguito v. Republic8 and Republic v. T.A.N.
area of Seventy One Thousand Three Hundred Twenty Four Properties, Inc.,9 they argue that the
(71,324) Square Meters, as supported by its technical reclassification of the land as alienable or disposable
description now forming part of the record of this opened it to acquisitive prescription under the Civil
case, in addition to other proofs adduced in the name Code; that Malabanan had purchased the property from
of MARIO MALABANAN, who is of legal age, Filipino, Eduardo Velazco believing in good faith that Velazco

49
Public Land V Govnt Land – Confirmation of Imperfect or incomplete title: Natural Resources Jennica Gyrl
Delfin
and his predecessors-in-interest had been the real Section 2, Article XII of the 1987 Constitution, only
owners of the land with the right to validly transmit agricultural lands of the public domain may be
title and ownership thereof; that consequently, the alienated; all other natural resources may not be.
ten-year period prescribed by Article 1134 of the Civil Alienable and disposable lands of the State fall into
Code, in relation to Section 14(2) of the Property two categories, to wit: (a) patrimonial lands of the
Registration Decree, applied in their favor; and that State, or those classified as lands of private
when Malabanan filed the application for registration ownership under Article 425 of the Civil
on February 20, 1998, he had already been in possession Code,23 without limitation; and (b) lands of the public
of the land for almost 16 years reckoned from 1982, domain, or the public lands as provided by the
the time when the land was declared alienable and Constitution, but with the limitation that the lands
disposable by the State. must only be agricultural. Consequently, lands
The Republic’s Motion for Partial Reconsideration classified as forest or timber, mineral, or national
The Republic seeks the partial reconsideration in order parks are not susceptible of alienation or disposition
to obtain a clarification with reference to the unless they are reclassified as agricultural.24 A
application of the rulings in Naguit and Herbieto. positive act of the Government is necessary to enable
Chiefly citing the dissents, the Republic contends that such reclassification,25 and the exclusive prerogative
the decision has enlarged, by implication, the to classify public lands under existing laws is vested
interpretation of Section 14(1) of the Property in the Executive Department, not in the courts.26 If,
Registration Decree through judicial legislation. It however, public land will be classified as neither
reiterates its view that an applicant is entitled to agricultural, forest or timber, mineral or national
registration only when the land subject of the park, or when public land is no longer intended for
application had been declared alienable and disposable public service or for the development of the national
since June 12, 1945 or earlier. wealth, thereby effectively removing the land from the
Ruling ambit of public dominion, a declaration of such
We deny the motions for reconsideration. conversion must be made in the form of a law duly
In reviewing the assailed decision, we consider to be enacted by Congress or by a Presidential proclamation
imperative to discuss the different classifications of in cases where the President is duly authorized by law
land in relation to the existing applicable land to that effect.27 Thus, until the Executive Department
registration laws of the Philippines. exercises its prerogative to classify or reclassify
Classifications of land according to ownership lands, or until Congress or the President declares that
Land, which is an immovable property,10 may be the State no longer intends the land to be used for
classified as either of public dominion or of private public service or for the development of national
ownership.11Land is considered of public dominion if it wealth, the Regalian Doctrine is applicable.
either: (a) is intended for public use; or (b) belongs Disposition of alienable public lands
to the State, without being for public use, and is Section 11 of the Public Land Act (CA No. 141) provides
intended for some public service or for the development the manner by which alienable and disposable lands of
of the national wealth.12 Land belonging to the State the public domain, i.e., agricultural lands, can be
that is not of such character, or although of such disposed of, to wit:
character but no longer intended for public use or for Section 11. Public lands suitable for agricultural
public service forms part of the patrimonial property purposes can be disposed of only as follows, and not
of the State.13 Land that is other than part of the otherwise:
patrimonial property of the State, provinces, cities (1) For homestead settlement;
and municipalities is of private ownership if it (2) By sale;
belongs to a private individual. (3) By lease; and
Pursuant to the Regalian Doctrine (Jura Regalia), a (4) By confirmation of imperfect or incomplete titles;
legal concept first introduced into the country from (a) By judicial legalization; or
the West by Spain through the Laws of the Indies and (b) By administrative legalization (free patent).
the Royal Cedulas,14 all lands of the public domain The core of the controversy herein lies in the proper
belong to the State.15This means that the State is the interpretation of Section 11(4), in relation to Section
source of any asserted right to ownership of land, and 48(b) of the Public Land Act, which expressly requires
is charged with the conservation of such patrimony.16 possession by a Filipino citizen of the land since June
All lands not appearing to be clearly under private 12, 1945, or earlier, viz:
ownership are presumed to belong to the State. Also, Section 48. The following-described citizens of the
public lands remain part of the inalienable land of Philippines, occupying lands of the public domain or
the public domain unless the State is shown to have claiming to own any such lands or an interest therein,
reclassified or alienated them to private persons.17 but whose titles have not been perfected or completed,
Classifications of public lands may apply to the Court of First Instance of the
according to alienability province where the land is located for confirmation of
Whether or not land of the public domain is alienable their claims and the issuance of a certificate of title
and disposable primarily rests on the classification thereafter, under the Land Registration Act, to wit:
of public lands made under the Constitution. Under the x x x x
1935 Constitution,18 lands of the public domain were (b) Those who by themselves or through their
classified into three, namely, agricultural, timber predecessors-in-interest have been in open,
and mineral.19 Section 10, Article XIV of the 1973 continuous, exclusive, and notorious possession and
Constitution classified lands of the public domain into occupation of alienable and disposable lands of the
seven, specifically, agricultural, industrial or public domain, under a bona fide claim of acquisition
commercial, residential, resettlement, mineral, timber of ownership, since June 12, 1945, or earlier,
or forest, and grazing land, with the reservation that immediately preceding the filing of the applications
the law might provide other classifications. The 1987 for confirmation of title, except when prevented by
Constitution adopted the classification under the 1935 war or force majeure. These shall be conclusively
Constitution into agricultural, forest or timber, and presumed to have performed all the conditions essential
mineral, but added national parks.20 Agricultural lands to a Government grant and shall be entitled to a
may be further classified by law according to the uses certificate of title under the provisions of this
to which they may be devoted.21 The identification of chapter. (Bold emphasis supplied)
lands according to their legal classification is done Note that Section 48(b) of the Public Land Act used
exclusively by and through a positive act of the the words "lands of the public domain" or "alienable
Executive Department.22 and disposable lands of the public domain" to clearly
Based on the foregoing, the Constitution places a limit signify that lands otherwise classified, i.e.,
on the type of public land that may be alienated. Under mineral, forest or timber, or national parks, and lands

50
Public Land V Govnt Land – Confirmation of Imperfect or incomplete title: Natural Resources Jennica Gyrl
Delfin
of patrimonial or private ownership, are outside the for as long as the lands were already converted to
coverage of the Public Land Act. What the law does not private ownership, by operation of law, as a result of
include, it excludes. The use of the descriptive phrase satisfying the requisite period of possession
"alienable and disposable" further limits the coverage prescribed by the Public Land Act.30 It is for this
of Section 48(b) to only the agricultural lands of the reason that the property subject of the application of
public domain as set forth in Article XII, Section 2 Malabanan need not be classified as alienable and
of the 1987 Constitution. Bearing in mind such disposable agricultural land of the public domain for
limitations under the Public Land Act, the applicant the entire duration of the requisite period of
must satisfy the following requirements in order for possession.
his application to come under Section 14(1) of the To be clear, then, the requirement that the land should
Property Registration Decree,28 to wit: have been classified as alienable and disposable
1. The applicant, by himself or through his agricultural land at the time of the application for
predecessor-in-interest, has been in possession and registration is necessary only to dispute the
occupation of the property subject of the application; presumption that the land is inalienable.
2. The possession and occupation must be open, The declaration that land is alienable and disposable
continuous, exclusive, and notorious; also serves to determine the point at which
3. The possession and occupation must be under a bona prescription may run against the State. The imperfect
fide claim of acquisition of ownership; or incomplete title being confirmed under Section 48(b)
4. The possession and occupation must have taken place of the Public Land Act is title that is acquired by
since June 12, 1945, or earlier; and reason of the applicant’s possession and occupation of
5. The property subject of the application must be an the alienable and disposable agricultural land of the
agricultural land of the public domain. public domain. Where all the necessary requirements
Taking into consideration that the Executive for a grant by the Government are complied with through
Department is vested with the authority to classify actual physical, open, continuous, exclusive and
lands of the public domain, Section 48(b) of the Public public possession of an alienable and disposable land
Land Act, in relation to Section 14(1) of the Property of the public domain, the possessor is deemed to have
Registration Decree, presupposes that the land subject acquired by operation of law not only a right to a
of the application for registration must have been grant, but a grant by the Government, because it is
already classified as agricultural land of the public not necessary that a certificate of title be issued in
domain in order for the provision to apply. Thus, order that such a grant be sanctioned by the courts.31
absent proof that the land is already classified as If one follows the dissent, the clear objective of the
agricultural land of the public domain, the Regalian Public Land Act to adjudicate and quiet titles to
Doctrine applies, and overcomes the presumption that unregistered lands in favor of qualified Filipino
the land is alienable and disposable as laid down in citizens by reason of their occupation and cultivation
Section 48(b) of the Public Land Act. However, emphasis thereof for the number of years prescribed by law32 will
is placed on the requirement that the classification be defeated. Indeed, we should always bear in mind that
required by Section 48(b) of the Public Land Act is such objective still prevails, as a fairly recent
classification or reclassification of a public land as legislative development bears out, when Congress
agricultural. enacted legislation (Republic Act No. 10023)33in order
The dissent stresses that the classification or to liberalize stringent requirements and procedures in
reclassification of the land as alienable and the adjudication of alienable public land to qualified
disposable agricultural land should likewise have been applicants, particularly residential lands, subject to
made on June 12, 1945 or earlier, because any area limitations.34
possession of the land prior to such classification or On the other hand, if a public land is classified as
reclassification produced no legal effects. It no longer intended for public use or for the
observes that the fixed date of June 12, 1945 could development of national wealth by declaration of
not be minimized or glossed over by mere judicial Congress or the President, thereby converting such land
interpretation or by judicial social policy concerns, into patrimonial or private land of the State, the
and insisted that the full legislative intent be applicable provision concerning disposition and
respected. registration is no longer Section 48(b) of the Public
We find, however, that the choice of June 12, 1945 as Land Act but the Civil Code, in conjunction with
the reckoning point of the requisite possession and Section 14(2) of the Property Registration Decree.35 As
occupation was the sole prerogative of Congress, the such, prescription can now run against the State.
determination of which should best be left to the To sum up, we now observe the following rules relative
wisdom of the lawmakers. Except that said date to the disposition of public land or lands of the
qualified the period of possession and occupation, no public domain, namely:
other legislative intent appears to be associated with (1) As a general rule and pursuant to the Regalian
the fixing of the date of June 12, 1945. Accordingly, Doctrine, all lands of the public domain belong to the
the Court should interpret only the plain and literal State and are inalienable. Lands that are not clearly
meaning of the law as written by the legislators. under private ownership are also presumed to belong to
Moreover, an examination of Section 48(b) of the Public the State and, therefore, may not be alienated or
Land Act indicates that Congress prescribed no disposed;
requirement that the land subject of the registration (2) The following are excepted from the general rule,
should have been classified as agricultural since June to wit:
12, 1945, or earlier. As such, the applicant’s (a) Agricultural lands of the public domain are
imperfect or incomplete title is derived only from rendered alienable and disposable through any of the
possession and occupation since June 12, 1945, or exclusive modes enumerated under Section 11 of the
earlier. This means that the character of the property Public Land Act. If the mode is judicial confirmation
subject of the application as alienable and disposable of imperfect title under Section 48(b) of the Public
agricultural land of the public domain determines its Land Act, the agricultural land subject of the
eligibility for land registration, not the ownership application needs only to be classified as alienable
or title over it. and disposable as of the time of the application,
Alienable public land held by a possessor, either provided the applicant’s possession and occupation of
personally or through his predecessors-in-interest, the land dated back to June 12, 1945, or earlier.
openly, continuously and exclusively during the Thereby, a conclusive presumption that the applicant
prescribed statutory period is converted to private has performed all the conditions essential to a
property by the mere lapse or completion of the government grant arises,36 and the applicant becomes
period.29 In fact, by virtue of this doctrine, the owner of the land by virtue of an imperfect or
corporations may now acquire lands of the public domain incomplete title. By legal fiction, the land has

51
Public Land V Govnt Land – Confirmation of Imperfect or incomplete title: Natural Resources Jennica Gyrl
Delfin
already ceased to be part of the public domain and has purposes in the name of Maxima L. Sin (deceased) under
become private property.37 Tax Declaration No. 10701 (1985) with an assessed value
(b) Lands of the public domain subsequently classified of Php1,320.00.2
or declared as no longer intended for public use or On August 26, 1991, respondent heirs instituted in the
for the development of national wealth are removed from RTC of Kalibo, Aklan a complaint against Lucio
the sphere of public dominion and are considered Arquisola, in his capacity as Superintendent of ANCF
converted into patrimonial lands or lands of private (hereinafter ANCF Superintendent), for recovery of
ownership that may be alienated or disposed through possession, quieting of title, and declaration of
any of the modes of acquiring ownership under the Civil ownership with damages. Respondent heirs claim that a
Code. If the mode of acquisition is prescription, 41,231-square meter-portion of the property they
whether ordinary or extraordinary, proof that the land inherited had been usurped by ANCF, creating a cloud
has been already converted to private ownership prior of doubt with respect to their ownership over the
to the requisite acquisitive prescriptive period is a parcel of land they wish to remove from the ANCF
condition sine qua non in observance of the law reservation.
(Article 1113, Civil Code) that property of the State The ANCF Superintendent countered that the parcel of
not patrimonial in character shall not be the object land being claimed by respondents was the subject of
of prescription. Proclamation No. 2074 of then President Ferdinand E.
To reiterate, then, the petitioners failed to present Marcos allocating 24.0551 hectares of land within the
sufficient evidence to establish that they and their area, which included said portion of private
predecessors-in-interest had been in possession of the respondents’ alleged property, as civil reservation
land since June 12, 1945. Without satisfying the for educational purposes of ANCF. The ANCF
requisite character and period of possession - Superintendent furthermore averred that the subject
possession and occupation that is open, continuous, parcel of land is timberland and therefore not
exclusive, and notorious since June 12, 1945, or susceptible of private ownership.
earlier - the land cannot be considered ipso jure Subsequently, the complaint was amended to include ANCF
converted to private property even upon the subsequent as a party defendant and Lucio Arquisola, who retired
declaration of it as alienable and disposable. from the service during the pendency of the case, was
Prescription never began to run against the State, such substituted by Ricardo Andres, then the designated
that the land has remained ineligible for registration Officer-in-Charge of ANCF.
under Section 14(1) of the Property Registration The RTC remanded the case to the MCTC of New Washington
Decree. Likewise, the land continues to be ineligible and Batan, Aklan, in view of the enactment of Republic
for land registration under Section 14(2) of the Act No. 7659 which expanded the jurisdiction of first-
Property Registration Decree unless Congress enacts a level courts. The case was docketed as Civil Case No.
law or the President issues a proclamation declaring 1181 (4390).
the land as no longer intended for public service or Before the MCTC, respondent heirs presented evidence
for the development of the national wealth.1âwphi1 that they inherited a bigger parcel of land from their
WHEREFORE, the Court DENIES the petitioners' Motion mother, Maxima Sin, who died in the year 1945 in New
for Reconsideration and the respondent's Partial Washington, Capiz (now Aklan). Maxima Sin acquired said
Motion for Reconsideration for their lack of merit. bigger parcel of land by virtue of a Deed of Sale
SO ORDERED. (Exhibit "B"), and then developed the same by planting
coconut trees, banana plants, mango trees and nipa
March 26, 2014 palms and usufructing the produce of said land until
G.R. No. 157485 her death in 1945.
REPUBLIC OF THE PHILIPPINES represented by AKLAN In the year 1988, a portion of said land respondents
NATIONAL COLLEGE OF FISHERIES (ANCF) and DR. ELENITA inherited from Maxima Sin was occupied by ANCF and
R. ANDRADE, in her capacity as ANCF converted into a fishpond for educational purpose.
Superintendent, Petitioner, Respondent heirs of Maxima Sin asserted that they were
vs. previously in possession of the disputed land in the
HEIRS OF MAXIMA LACHICA SIN, namely: SALVACION L. SIN, concept of an owner. The disputed area was a swampy
ROSARIO S. ENRIQUEZ, FRANCISCO L. SIN, MARIA S. land until it was converted into a fishpond by the
YUCHINTAT, MANUEL L. SIN, JAIME CARDINAL SIN, RAMON L. ANCF. To prove possession, respondents presented
SIN, and CEFERINA S. VITA,Respondents. several tax declarations, the earliest of which was in
D E C I S I O N the year 1945.
LEONARDO-DE CASTRO, J.: On June 19, 2000, the MCTC rendered its Decision in
This is a Petition for Review assailing the favor of respondents, the dispositive portion of which
Decision1 of the Court of Appeals in CA-G.R. SP No. reads:
65244 dated February 24, 2003, which upheld the WHEREFORE, judgment is rendered declaring plaintiffs
Decisions of the Regional Trial Court (RTC) of Kalibo, [respondent heirs herein] the owner and possessor of
Aklan in Civil Case No. 6130 and the First Municipal the land in question in this case and for the
Circuit Trial Court (MCTC) of New Washington and Batan, defendants to cause the segregation of the same from
Aklan in Civil Case No. 1181, segregating from the the Civil Reservation of the Aklan National College of
Aklan National College of Fisheries (ANCF) reservation Fisheries, granted under Proclamation No. 2074 dated
the portion of land being claimed by respondents. March 31, 1981.
Petitioner in this case is the Republic of the It is further ordered, that defendants jointly and
Philippines, represented by ANCF and Dr. Elenita R. severally pay the plaintiffs actual damages for the
Andrade, in her capacity as Superintendent of ANCF. unearned yearly income from nipa plants uprooted by
Respondents claim that they are the lawful heirs of the defendants [on] the land in question when the same
the late Maxima Lachica Sin who was the owner of a has been converted by the defendants into a fishpond,
parcel of land situated at Barangay Tambac, New in the amount of Php3,500.00 yearly beginning the year
Washington, Aklan, and more particularly described as 1988 until plaintiffs are fully restored to the
follows: possession of the land in question.
A parcel of cocal, nipal and swampy land, located at It is finally ordered, that defendants jointly and
Barangay Tambac, New Washington, Aklan, containing an severally pay the plaintiffs the sum of Php10,000.00
approximate area of FIFTY[-]EIGHT THOUSAND SIX HUNDRED for attorney’s fees and costs of this suit.3
SIX (58,606) square meters, more or less, as per survey According to the MCTC, the sketch made by the Court
by Geodetic Engineer Reynaldo L. Lopez. Bounded on the Commissioner in his report (Exh. "LL") shows that the
North by Dumlog Creek; on the East by Adriano disputed property is an alienable and disposable land
Melocoton; on the South by Mabilo Creek; and on the of the public domain. Furthermore, the land covered by
West by Amado Cayetano and declared for taxation Civil Reservation under Proclamation No. 2074 was

52
Public Land V Govnt Land – Confirmation of Imperfect or incomplete title: Natural Resources Jennica Gyrl
Delfin
classified as timberland only on December 22, 1960 previous owners and that of Maxima Lachica Sin over
(Exh. "4-D"). The MCTC observed that the phrase "Block the disputed property, it does not tax ones imagination
II Alien or Disp. LC 2415" was printed on the Map of to conclude that the subject property had been
the Civil Reservation for ANCF established under privately possessed for more than 30 years before it
Proclamation No. 2074 (Exh. "6"), indicating that the was declared a timberland. This being the case, the
disputed land is an alienable and disposable land of said possession has ripened into an ownership against
the public domain. the State, albeit an imperfect one. Nonetheless, it is
The MCTC likewise cited a decision of this Court in our considered opinion that this should come under the
the 1976 case of Republic v. Court of Appeals4 where meaning of "private rights" under Proclamation No. 2074
it was pronounced that: which are deemed segregated from the mass of civil
Lands covered by reservation are not subject to entry, reservation granted to petitioner.7(Citation omitted.)
and no lawful settlement on them can be acquired. The Hence, this Petition for Review, anchored on the
claims of persons who have settled on, occupied, and following grounds:
improved a parcel of public land which is later I
included in a reservation are considered worthy of THE COURT OF APPEALS GRAVELY ERRED ON A QUESTION OF
protection and are usually respected, but where the LAW IN UPHOLDING RESPONDENTS’ CLAIM TO SUPPOSED
President, as authorized by law, issues a proclamation "PRIVATE RIGHTS" OVER SUBJECT LAND DESPITE THE DENR
reserving certain lands, and warning all persons to CERTIFICATION THAT IT IS CLASSIFIED AS TIMBERLAND.
depart therefrom, this terminates any rights II
previously acquired in such lands by a person who has THE COURT OF APPEALS GRAVELY ERRED ON A QUESTION OF
settled thereon in order to obtain a preferential right LAW IN AFFIRMING THE DECISIONS OF THE REGIONAL TRIAL
of purchase. And patents for lands which have been COURT AND THE MUNICIPAL CIRCUIT TRIAL COURTS RELEASING
previously granted, reserved from sale, or THE SUBJECT LAND BEING CLAIMED BY RESPONDENTS FROM THE
appropriated are void. (Underscoring from the MCTC, MASS OF PUBLIC DOMAIN AND AWARDING DAMAGES TO THEM.8
citations omitted.) The central dispute in the case at bar is the
Noting that there was no warning in Proclamation No. interpretation of the first paragraph of Proclamation
2074 requiring all persons to depart from the No. 2074:
reservation, the MCTC concluded that the reservation Upon recommendation of the Director of Forest
was subject to private rights if there are any. Development, approved by the Minister of Natural
The MCTC thus ruled that the claim of respondent heirs Resources and by virtue of the powers vested in me by
over the disputed land by virtue of their and their law, I, FERDINAND E. MARCOS, President of the
predecessors’ open, continuous, exclusive and Philippines, do hereby set aside as Civil Reservation
notorious possession amounts to an imperfect title, for Aklan National College of Fisheries, subject to
which should be respected and protected. private rights, if any there be, parcels of land,
Petitioner, through the Solicitor General, appealed to containing an aggregate area of 24.0551 hectares,
the RTC of Kalibo, Aklan, where the case was docketed situated in the Municipality of New Washington,
as Civil Case No. 6130. Province of Aklan, Philippines, designated Parcels I
On May 2, 2001, the RTC rendered its Decision affirming and II on the attached BFD Map CR-203, x x x [.]9
the MCTC judgment with modification: The MCTC, the RTC and the Court of Appeals unanimously
WHEREFORE, premises considered, the assailed decision held that respondents retain private rights to the
is modified absolving Appellant Ricardo Andres from disputed property, thus preventing the application of
the payment of damages and attorney’s fees. All other the above proclamation thereon. The private right
details of the appealed decision are affirmed in toto.5 referred to is an alleged imperfect title, which
The RTC stressed that Proclamation No. 2074 recognizes respondents supposedly acquired by possession of the
vested rights acquired by private individuals prior to subject property, through their predecessors-in-
its issuance on March 31, 1981. interest, for 30 years before it was declared as a
The RTC added that the findings of facts of the MCTC timberland on December 22, 1960.
may not be disturbed on appeal unless the court below At the outset, it must be noted that respondents have
has overlooked some facts of substance that may alter not filed an application for judicial confirmation of
the results of its findings. The RTC, however, absolved imperfect title under the Public Land Act or the
the Superintendent of the ANCF from liability as there Property Registration Decree. Nevertheless, the courts
was no showing on record that he acted with malice or a quo apparently treated respondents’ complaint for
in bad faith in the implementation of Proclamation No. recovery of possession, quieting of title and
2074.6 declaration of ownership as such an application and
Petitioner Republic, represented by the ANCF and Dr. proceeded to determine if respondents complied with
Elenita R. Andrade, in her capacity as the new the requirements therefor.
Superintendent of the ANCF, elevated the case to the The requirements for judicial confirmation of
Court of Appeals through a Petition for Review. The imperfect title are found in Section 48(b) of the
petition was docketed as CA-G.R. SP No. 65244. Public Land Act, as amended by Presidential Decree No.
On February 24, 2003, the Court of Appeals rendered 1073, as follows:
its Decision dismissing the petition for lack of merit. Sec. 48. The following described citizens of the
In addition to the findings of the MCTC and the RTC, Philippines, occupying lands of the public domain or
the Court of Appeals held: claiming to own any such lands or an interest therein,
Moreover, petitioner had not shown by competent but whose titles have not been perfected or completed,
evidence that the subject land was likewise declared a may apply to the Court of First Instance of the
timberland before its formal classification as such in province where the land is located for confirmation of
1960. Considering that lands adjoining to that of the their claims and the issuance of a certificate of title
private respondents, which are also within the therefor, under the Land Registration Act, to wit:
reservation area, have been issued original x x x x
certificates of title, the same affirms the conclusion (b) Those who by themselves or through their
that the area of the subject land was agricultural, predecessors in interest have been in the open,
and therefore disposable, before its declaration as a continuous, exclusive, and notorious possession and
timberland in 1960. occupation of alienable and disposable lands of the
It should be noted that Maxima Lachica Sin acquired, public domain, under a bona fide claim of acquisition
through purchase and sale, the subject property from or ownership, since June 12, 1945, or earlier,
its previous owners spouses Sotera Melocoton and Victor immediately preceding the filing of the application
Garcia on January 15, 1932, or 28 years before the said for confirmation of title except when prevented by war
landholding was declared a timberland on December 22, or force majeure. These shall be conclusively presumed
1960. Tacking, therefore, the possession of the to have performed all the conditions essential to a

53
Public Land V Govnt Land – Confirmation of Imperfect or incomplete title: Natural Resources Jennica Gyrl
Delfin
Government grant and shall be entitled to a certificate Yap,13 which presents a similar issue with respect to
of title under the provisions of this chapter. another area of the same province of Aklan. On November
An equivalent provision is found in Section 14(1) of 10, 1978, President Marcos issued Proclamation No. 1801
the Property Registration Decree, which provides: declaring Boracay Island, among other islands, caves
SECTION 14. Who may apply.— The following persons may and peninsulas of the Philippines, as tourist zones
file in the proper Court of First Instance an and marine reserves under the administration of the
application for registration of title to land, whether Philippine Tourism Authority (PTA). On September 3,
personally or through their duly authorized 1982, PTA Circular 3-82 was issued to implement
representatives: Proclamation No. 1801. The respondents-claimants in
(1) those who by themselves or through their said case filed a petition for declaratory relief with
predecessors-in- interest have been in open, the RTC of Kalibo, Aklan, claiming that Proclamation
continuous, exclusive and notorious possession and No. 1801 and PTA Circular 3-82 precluded them from
occupation of alienable and disposable lands of the filing an application for judicial confirmation of
public domain under a bona fide claim of ownership imperfect title or survey of land for titling purposes.
since June 12, 1945, or earlier. The respondents claim that through their predecessors-
This Court has thus held that there are two requisites in-interest, they have been in open, continuous,
for judicial confirmation of imperfect or incomplete exclusive and notorious possession and occupation of
title under CA No. 141, namely: (1) open, continuous, their lands in Boracay since June 12, 1945 or earlier
exclusive, and notorious possession and occupation of since time immemorial.
the subject land by himself or through his On May 22, 2006, during the pendency of the petition
predecessors-in-interest under a bona fide claim of for review of the above case with this Court, President
ownership since time immemorial or from June 12, 1945; Gloria Macapagal-Arroyo issued Proclamation No. 1064
and classifying Boracay Island into four hundred (400)
(2) the classification of the land as alienable and hectares of reserved forest land (protection purposes)
disposable land of the public domain.10 and six hundred twenty-eight and 96/100 (628.96)
With respect to the second requisite, the courts a quo hectares of agricultural land (alienable and
held that the disputed property was alienable and disposable). Petitioner-claimants and other landowners
disposable before 1960, citing petitioner’s failure to in Boracay filed with this Court an original petition
show competent evidence that the subject land was for prohibition, mandamus and nullification of
declared a timberland before its formal classification Proclamation No. 1064, alleging that it infringed on
as such on said year.11 Petitioner emphatically their "prior vested right" over portions of Boracay
objects, alleging that under the Regalian Doctrine, which they allege to have possessed since time
all lands of the public domain belong to the State and immemorial. This petition was consolidated with the
that lands not appearing to be clearly within private petition for review concerning Proclamation No. 1801
ownership are presumed to belong to the State. and PTA Circular 3- 82.
After a thorough review of the records, we agree with This Court, discussing the Regalian Doctrine vis-à-vis
petitioner. As this Court held in the fairly recent the right of the claimants to lands they claim to have
case of Valiao v. Republic12: possessed since time immemorial, held:
Under the Regalian doctrine, which is embodied in our A positive act declaring land as alienable and
Constitution, all lands of the public domain belong to disposable is required. In keeping with the presumption
the State, which is the source of any asserted right of State ownership, the Court has time and again
to any ownership of land. All lands not appearing to emphasized that there must be a positive act of the
be clearly within private ownership are presumed to government, such as an official proclamation,
belong to the State. Accordingly, public lands not declassifying inalienable public land into disposable
shown to have been reclassified or released as land for agricultural or other purposes. In fact,
alienable agricultural land or alienated to a private Section 8 of CA No. 141 limits alienable or disposable
person by the State remain part of the inalienable lands only to those lands which have been "officially
public domain. Unless public land is shown to have been delimited and classified."
reclassified as alienable or disposable to a private The burden of proof in overcoming the presumption of
person by the State, it remains part of the inalienable State ownership of the lands of the public domain is
public domain. Property of the public domain is beyond on the person applying for registration (or claiming
the commerce of man and not susceptible of private ownership), who must prove that the land subject of
appropriation and acquisitive prescription. Occupation the application is alienable or disposable. To overcome
thereof in the concept of owner no matter how long this presumption, incontrovertible evidence must be
cannot ripen into ownership and be registered as a established that the land subject of the application
title. The burden of proof in overcoming the (or claim) is alienable or disposable. There must still
presumption of State ownership of the lands of the be a positive act declaring land of the public domain
public domain is on the person applying for as alienable and disposable. To prove that the land
registration (or claiming ownership), who must prove subject of an application for registration is
that the land subject of the application is alienable alienable, the applicant must establish the existence
or disposable. To overcome this presumption, of a positive act of the government such as a
incontrovertible evidence must be established that the presidential proclamation or an executive order; an
land subject of the application (or claim) is alienable administrative action; investigation reports of Bureau
or disposable. of Lands investigators; and a legislative act or a
There must be a positive act declaring land of the statute. The applicant may also secure a certification
public domain as alienable and disposable.1âwphi1 To from the government that the land claimed to have been
prove that the land subject of an application for possessed for the required number of years is alienable
registration is alienable, the applicant must and disposable.
establish the existence of a positive act of the In the case at bar, no such proclamation, executive
government, such as a presidential proclamation or an order, administrative action, report, statute, or
executive order; an administrative action; certification was presented to the Court. The records
investigation reports of Bureau of Lands are bereft of evidence showing that, prior to 2006,
investigators; and a legislative act or a statute. The the portions of Boracay occupied by private claimants
applicant may also secure a certification from the were subject of a government proclamation that the land
government that the land claimed to have been possessed is alienable and disposable. Absent such well-nigh
for the required number of years is alienable and incontrovertible evidence, the Court cannot accept the
disposable. (Citations omitted.) submission that lands occupied by private claimants
This Court reached the same conclusion in Secretary of were already open to disposition before 2006. Matters
the Department of Environment and Natural Resources v. of land classification or reclassification cannot be

54
Public Land V Govnt Land – Confirmation of Imperfect or incomplete title: Natural Resources Jennica Gyrl
Delfin
assumed. They call for proof.14 (Emphases in the
original; citations omitted.)
Accordingly, in the case at bar, the failure of
petitioner Republic to show competent evidence that
the subject land was declared a timberland before its
formal classification as such in 1960 does not lead to
the presumption that said land was alienable and
disposable prior to said date. On the contrary, the
presumption is that unclassified lands are inalienable
public lands. Such was the conclusion of this Court in
Heirs of the Late Spouses Pedro S. Palanca and
Soterranea Rafols v. Republic,15 wherein we held:
While it is true that the land classification map does
not categorically state that the islands are public
forests, the fact that they were unclassified lands
leads to the same result. In the absence of the
classification as mineral or timber land, the land
remains unclassified land until released and rendered
open to disposition. x x x. (Emphasis supplied,
citation deleted.)
The requirements for judicial confirmation of
imperfect title in Section 48(b) of the Public Land
Act, as amended, and the equivalent provision in
Section 14(1) of the Property Registration Decree was
furthermore painstakingly debated upon by the members
of this Court in
Heirs of Mario Malabanan v. Republic.16 In Malabanan,
the members of this Court were in disagreement as to
whether lands declared alienable or disposable after
June 12, 1945 may be subject to judicial confirmation
of imperfect title. There was, however, no disagreement
that there must be a declaration to that effect.
In the case at bar, it is therefore the respondents
which have the burden to identify a positive act of
the government, such as an official proclamation,
declassifying inalienable public land into disposable
land for agricultural or other purposes. Since
respondents failed to do so, the alleged possession by
them and by their predecessors-in-interest is
inconsequential and could never ripen into ownership.
Accordingly, respondents cannot be considered to have
private rights within the purview of Proclamation No.
2074 as to prevent the application of said proclamation
to the subject property. We are thus constrained to
reverse the rulings of the courts a quo and grant the
prayer of petitioner Republic to dismiss Civil Case
No. 1181 (4390) for lack of merit.
WHEREFORE, premises considered, the Petition for
Review is GRANTED. The Decision of the Court of Appeals
in CA-G.R. SP No. 65244 dated February 24, 2003, which
upheld the Decisions of the Regional Trial Court of
Kalibo, Aklan in Civil Case No. 6130 and the First
Municipal Circuit Trial Court of New Washington and
Batan, Aklan in Civil Case No. 1181 (4390), segregating
from the Aklan National College of Fisheries
reservation the portion of land being claimed by
respondents is REVERSED and SET ASIDE. Civil Case No.
1181 (4390) of the First Municipal Circuit Trial Court
of New Washington and Batan, Aklan is hereby DISMISSED.
SO ORDERED.

55

You might also like